Você está na página 1de 308

Clculo II

Eliezer Batista
Elisa Zunko Toma
Mrcio Rodolfo Fernandes
Silvia Martini de Holanda Janesch

2 Edio
Florianpolis, 2012

Governo Federal
Presidente da Repblica: Dilma Vana Rousseff
Ministro de Educao: Alozio Mercadante
Coordenador Nacional da Universidade Aberta do Brasil: Celso Costa

Universidade Federal de Santa Catarina


Reitora: Roselane Neckel
Vice-Reitora: Lcia Helena Martins Pacheco
Pr-Reitoria de Graduao: Roselane Ftima Campos e
Rogrio Luiz de Souza

Pr-Reitoria de Ps-Graduao: Joana Maria Pedro e


Juarez Vieira do Nascimento

Pr-Reitoria de Pesquisa: Jamil Assereuy Filho e Heliete Nunes


Pr-Reitoria de Extenso: Edilson da Rosa e
Maristela Helena Zimmer Bortolini

Pr-Reitoria de Planejamento e Oramento: Luiz Alberton e Izabela Raquel


Pr-Reitoria de Administrao: Antnio Carlos Montezuma Brito e
Irvando Luiz Speranzini

Pr-Reitoria de Assuntos Estudantis: Beatriz Augusto de Paiva e


Simone Matos Machado

Centro de Cincias da Educao: Vera Lucia Bazzo


Centro de Cincias Fsicas e Matemticas: Tarciso Antnio Grandi

Cursos de Licenciaturas na Modalidade Distncia


Coordenao Acadmica Matemtica: Mrcio Rodolfo Fernandes
Coordenao de Ambientes Virtuais: Nereu Estanislau Burin

Comisso Editorial
Antnio Carlos Gardel Leito
Albertina Zatelli
Elisa Zunko Toma
Igor Mozolevski
Luiz Augusto Saeger
Roberto Corra da Silva
Ruy Coimbra Charo

Laboratrio de Novas Tecnologias - LANTEC/CED


Coordenao Pedaggica das Licenciaturas Distncia UFSC/CED/CFM
Coordenao Geral: Roseli Zen Cerny
Ncleo de Formao: Marina Bazzo de Espndola
Ncleo de Pesquisa e Avaliao: Andra Brando Lapa
Ncleo de Criao e Desenvolvimento de Materias: Juliana Cristina Faggion
Bergmann

Design Grfico
Coordenao: Cntia Cardoso, Cristiane Barbato Amaral, Talita vila Nunes
Projeto Grfico Original: Diogo Henrique Ropelato, Marta Cristina Goulart
Braga, Natal Anacleto Chicca Junior

Redesenho do Projeto Grfico: Laura Martins Rodrigues,


Thiago Rocha Oliveira

Diagramao: Cntia Cardoso, Joo Paulo Battisti de Abreu, Kallani Bonelli,


Laura Rodrigues, Roberto Colombo Gava, Talita vila Nunes

Ilustraes: Grazielle Xavier, Jean Henrique Menezes


Capa: Cntia Cardoso

Design Instrucional
Coordenao: Elizandro Maurcio Brick
Design Instrucional: Adriano Luiz dos Santos N, Niclio Jos Gesser,
Maria Carolina Machado Magnus

Reviso Gramatical: Contextuar


Copyright 2012, Universidade Federal de Santa Catarina/CFM/CED/UFSC
Nenhuma parte deste material poder ser reproduzida, transmitida e gravada, por qualquer
meio eletrnico, por fotocpia e outros, sem a prvia autorizao, por escrito, da Coordenao
Acadmica do Curso de Licenciatura em Matemtica na Modalidade a Distncia.

Ficha Catalogrfica
C144

Clculo II/ Eliezer Batista, Elisa Zunko Toma, Mrcio Rodolfo


Fernandes, Silvia Martini de Holanda Janesch - 2 ed. - Florianpolis:
UFSC/EAD/CED/CFM, 2012.
308 p.
Inclui bibliografia
UFSC. Licenciatura em Matemtica na Modalidade a Distncia
ISBN 978-85-8030-022-2
1. Clculo integral. 2. Sries (Matemtica). 3. Sries de potncia. 4.
Ensino a distncia. I. Batista, Eliezer.
CDU: 517.3

Catalogao na fonte pela Biblioteca Universitria da UFSC

Sumrio
Apresentao.....................................................................................7
1. Integrais.................................................................................. 9
1.1 Introduo.................................................................................... 11
1.2 Integrais inferior e superior funes integrveis............... 18
1.3 Integral como limite de somas.................................................. 26
1.4 Propriedades da integral............................................................ 35
1.5 Teorema fundamental do clculo............................................. 39
1.6 Integral indefinida...................................................................... 45
1.7 Tcnicas de integrao................................................................ 50
1.7.1 Mtodo da substituio ou mudana de varivel........... 50
1.7.2 Mtodo da integrao por partes...................................... 56
1.8 Clculo de reas...........................................................................61
1.9 Integrais imprprias................................................................... 67
1.10 Utilizao de pacotes computacionais.................................... 81
Resumo............................................................................................... 90

2. Mtodos de Integrao....................................................... 95
2.1 Integrais envolvendo funes trigonomtricas....................... 97
2.1.1 Funes trigonomtricas.................................................... 97
2.1.2 Integrais envolvendo potncias de senx e cosx............... 99
2.1.3 Integrais de potncias de tgx e cotgx..............................102
2.1.4 Integrais de potncias de secx e cossecx........................ 104
2.1.5 Integrais de produtos de potncias de tgx e secx......... 105
2.1.6 Integrais de funes envolvendo seno e cosseno de arcos
diferentes .......................................................................... 109
2.2 Substituio trigonomtrica.....................................................110
2.3 Integrao de funes racionais: mtodo das fraes
parciais....................................................................................... 120
2.4 Integrao de funes racionais de seno e cosseno
(substituio universal).............................................................132
Resumo............................................................................................. 138

3. Aplicaes de Integral...................................................... 147


3.1 Equaes diferenciais de primeira ordem com variveis
separveis....................................................................................149
3.2 Comprimento de arco de curvas planas................................ 158
3.3 Slidos e superfcies de revoluo...........................................162

3.3.1 Mtodo dos discos............................................................ 164


3.3.3 Mtodo das cascas cilndricas......................................... 171
3.3.4 reas de superfcies de revoluo.................................. 177
3.4 Centro de massa de regies planas.........................................182
3.5 Curvas e reas em coordenadas polares................................ 188

4. Sries Numricas............................................................... 203


4.1 Introduo.................................................................................. 205
4.2 Definies................................................................................... 208
4.3 Condies de convergncia e divergncia..............................219
4.4 Operaes sobre sries............................................................. 222
4.5 Sries de termos positivos ou nulos...................................... 225
4.6 Sries alternadas e sries absolutamente convergentes....... 247
4.6.1. Sries alternadas............................................................... 248
4.6.2 Sries absolutamente convergentes................................ 253

5. Sries de Potncia.............................................................. 261


5.1 Introduo.................................................................................. 263
5.2 Srie de potncias e convergncia........................................... 264
5.3 Representao de funes como sries de potncias........... 272
5.4 Srie de Taylor e srie de Maclaurin....................................... 278
5.4.1 Definies . ....................................................................... 279
5.4.2 Caso que a srie de Taylor

tem como

soma a funo f (x)..................................................................... 285


5.5 Aplicaes............................................................................. 296
5.5.1 Aplicaes de polinmios de Taylor............................... 296
5.5.2 Srie binominal................................................................. 298
5.5.3 Clculo de integrais aproximadas.................................. 302
Bibliografia bsica........................................................................... 307
Bibliografia complementar............................................................. 307

Apresentao
Caro estudante,
Estamos iniciando a disciplina de Clculo II!
O estudo dos contedos desta disciplina requer que voc tenha
conhecimento sobre limite, continuidade e derivada de funo de
uma varivel, conceitos estes estudados na disciplina de Clculo I.
O contedo deste material est divido em cinco captulos. No
Captulo 1 estudaremos o conceito de integral e suas propriedades. Demonstraremos o Teorema Fundamental do Clculo, um
resultado importante que relaciona a integral com a derivada, e
que simplifica consideravelmente a soluo de muitos problemas
envolvendo integrais. No Captulo 2 estudaremos algumas tcnicas de integrao. No Captulo 3, utilizaremos a integral definida para resolver problemas de clculo de reas, comprimento de
arcos, volume de slidos de revoluo e rea de superfcies de
revoluo. Os Captulos 4 e 5 so dedicados ao estudo de sries
numricas e sries de potncias. Vrios mtodos, ou testes, para
analisar a convergncia de sries numricas e sries de potncias
sero apresentados, alm de suas aplicaes ao clculo de integrais e representao de funes por sries de potncias.
Os Captulos 1 e 2 foram escritos pela Professora Svia, o Captulo 3 foi elaborado pelo Professor Eliezer e os Captulos 4 e 5 so
de responsabilidade da Professora Elisa. Em partes do texto so
apresentadas atividades complementares que envolvem o uso de
softwares matemticos. Essas atividades foram contribuies do
Professor Mrcio.
Esperamos que ao final da disciplina, voc tenha condies de
calcular e aplicar com adequado desembarao, integrais de funo de uma varivel e, alm disso, que saiba analisar a convergncia de sries numricas e de potncias, bem como representar
funes por sries de potncias. Tambm, esperamos que fique
bem compreendido o sentido de aproximar uma funo por
seus polinmios de Taylor.

Os autores

Captulo 1
Integrais

Captulo 1
Integrais
Neste captulo, estudaremos o conceito de integral e suas
propriedades. A integral tem muitas aplicaes na geometria (clculo de rea de regies planas, comprimento de
arco e clculo de volumes) e na fsica (clculo de trabalho, massa e momento de inrcia). Um dos resultados mais
importantes deste captulo o Teorema Fundamental do
Clculo, que relaciona a integral com a derivada, e simplifica consideravelmente a soluo de muitos problemas
envolvendo integrais.

1.1 Introduo
A principal motivao para estudar o conceito de integral est em
encontrar a rea de uma regio plana qualquer. O problema pode
ser formulado da seguinte forma:
Problema 1. Considere uma funo f :[a, b] contnua. Admitimos que f ( x) 0 para todo x [a, b]. Queremos encontrar a rea da
regio plana S que est limitada pelo grfico de f , pelas retas x = a
e x = b, e o eixo x.

y = f (x)

S = {( x, y) 2; 0 y f ( x) e a x b}
Figura 1.1

12

O problema de clculo de rea de uma regio plana antigo. Os


gregos, h aproximadamente 2.500 anos, j sabiam como encontrar
a rea de qualquer polgono. A ideia da tcnica empregada era dividir o polgono em tringulos ou retngulos e em seguida somar as
reas obtidas. No caso em que a regio plana era qualquer, o mtodo
utilizado era o da exausto, que consiste em inscrever ou circunscrever a figura com polgonos cujas reas eram conhecidas e melhorar a aproximao da rea desejada, aumentando o nmero de
polgonos inscritos ou circunscritos. As Figuras 1.2 e 1.3 ilustram o
mtodo aplicado pelos gregos.

S2

S1
a

S3
a

S7
b

Figura 1.2

S1
a

S2
b

S7

S3
a

a
Figura 1.3

13

Para encontrar a rea da regio S do Problema 1, em primeiro lugar precisamos dizer o que significa a rea de S, e depois tentar calcul-la. A ideia intuitiva de rea nos leva a dizer que a rea de uma
regio plana um nmero real no negativo. Mas como defini-lo?
Poderamos pensar em definir a rea de S como sendo o supremo
das reas dos retngulos contidos em S (ver Figura 1.2). Vamos
cham-lo de medida interna de S, que denotaremos por mint ( S ). De
forma semelhante, poderamos pensar em definir a rea de S como
nfimo das reas dos retngulos que contm S (ver Figura 1.3).
Vamos chamar este nmero de medida externa de S , e indicaremos por mext ( S ). Ao tentar definir a rea de S usando mint ( S ) ou
mext ( S ) surge a pergunta: esses nmeros so iguais? Quando a funo f contnua a resposta afirmativa. Assim, definimos a rea
de S como sendo o nmero real A no negativo tal que
A = mint ( S ) = mext ( S ) .
As noes de mint ( S ) e mext ( S ) levam os conceitos de integrais inferior e superior, respectivamente. No caso em que f :[a, b]
contnua e no negativa, os nmeros mint ( S ) e mext ( S ) coincidem
com as integrais inferior e superior, respectivamente. Na Seo 1.2,
trabalharemos com funo f :[a, b] limitada, mas no necessariamente contnua, e veremos que as integrais inferior e superior
nem sempre so iguais.
Antes de apresentar as definies de integrais inferior e superior,
vamos relembrar os conceitos de supremo e nfimo de um conjunto
e suas propriedades que sero teis para compreenso do texto.

Supremo e nfimo de um conjunto


Seja A um subconjunto de R, A . Dizemos que A limitado
superiormente quando existe b tal que x b, para todo x A.
Cada b R com essa propriedade chama-se cota superior de A.
Analogamente, dizemos que A limitado inferiormente quando
existe a tal que a x, para todo x A. Um elemento a com
essa propriedade chama-se cota inferior de A.
Quando o conjunto A limitado inferiormente e superiormente, dizemos que A limitado, isto , existem a e b tais que A [a, b].

14

Uma cota superior de A que pertence a A, chama-se mximo de


A e indica-se por max A. Uma cota inferior de A que pertence a A,
denomina-se mnimo de A e indica-se por min A.
Exemplo 1.1. Considere o conjunto A = {x | x 2}. Temos:
a) 2 , 1, -100 so cotas inferiores de A. O conjunto A limitado
inferiormente.
b) O conjunto A no possui cota superior, isto , no existe b
tal que b x para todo x A. Logo, A no limitado superiormente.
c) 2 uma cota inferior de A que pertence ao conjunto A. Logo,
min A = 2.
Exemplo 1.2. Seja B = {x | -3 x < 4}. Temos:
a) 10, - 5, - 3 so cotas inferiores de B e 4, 5, 7 so cotas superiores de B. Segue que B um conjunto limitado inferiormente e superiormente, logo B limitado.
b) - 3 uma cota inferior de B que pertence ao conjunto B. Logo,
min B = -3.
c) O conjunto B no tem mximo. De fato, para todo x B temos
x+4
x+4
B e x <
. Desta forma, para cada x B, existe outro
2
2
nmero em B que estritamente maior que x. Portanto, B no
admite mximo.
O conjunto B acima limitado superiormente e no admite mximo, mas tem uma cota superior que a menor de todas. Essa situao conduz a definio de supremo de um conjunto.
Definio 1.1. Seja A , A um subconjunto limitado superiormente. Um elemento b chama-se supremo de A, quando b
a menor das cotas superiores de A e indica-se por sup A = b.
O supremo de um conjunto pode ser caracterizado atravs das condies ( S1 ) e ( S 2 ) apresentadas na proposio abaixo.

15

Proposio 1.1. Seja A , A . Um elemento b o supremo


de A, se, e somente se, as duas condies seguintes so satisfeitas:
( S1 ) Para todo x A, tem-se x b;
( S 2 ) Dado e > 0 qualquer, existe x A tal que b - e < x.
(Ver Figura 1.4)
Demonstrao.
() ( S1 ) imediata, pois b cota superior de A.
Para provar ( S 2 ), suponhamos que existe um e 0 > 0 tal que b - e0 x
para todo x A. Assim, b - e0 uma cota superior de A. Como
b - e0 < b temos uma contradio, pois b a menor das cotas superiores. Logo, para todo e > 0, existe x A tal que b - e < x.
() De ( S1 ) temos que b cota superior de A. Falta mostrar que b
a menor das cotas superiores. Suponhamos que exista outra cota superior, digamos c, tal que c < b. Ento e = b - c > 0 e por ( S 2 ), existe
x A tal que b - (b - c) < x. Assim, existe x A tal que c < x, o que
absurdo, uma vez que c uma cota superior de A.

sup A

sup A = b
x A
Figura 1.4

Note que ( S1 ) diz que b cota superior de A e ( S 2 ) afirma que no


existe outra cota superior menor que b. De maneira anloga, definese o nfimo de um conjunto.
Definio 1.2. Seja A , A um subconjunto limitado inferiormente. Um elemento a chama-se nfimo de A, quando a a
maior das cotas inferiores de A e indica-se por inf A = a.

16

Proposio 1.2. Seja A , A . Um elemento a o nfimo


de A se, e somente se, as seguintes condies so satisfeitas:
( I1 ) Para todo x A, tem-se x a;
( I 2 ) Dado e > 0 qualquer, existe x A tal que x < a + e.
(Ver Figura 1.5)
Demonstrao. A prova anloga da proposio anterior. Fica
como exerccio.

inf A +

inf A = a
x A
Figura 1.5

Exemplo 1.3. Considere o conjunto B do Exemplo 1.2. Temos:


a) 4 a menor das cotas superiores de B, logo sup B = 4.
b) 3 a maior das cotas inferiores de B, portanto inf B = -3.
c) Note que o conjunto B possui min B = inf B, e B no admite
elemento mximo, mas possui supremo.
Em , todo subconjunto limitado superiormente possui supremo.
Esse fato apresentado no teorema abaixo.
Teorema 1.1. (Teorema do Supremo) Todo subconjunto de nmeros
reais, no vazio e limitado superiormente, admite supremo em .
De forma anloga ao Teorema do Supremo, temos um resultado que
diz que todo conjunto no vazio de nmeros reais, que limitado
inferiormente, admite nfimo.
Lembremos as seguintes propriedades de supremo e nfimo.

17

Proposio 1.3. Sejam A R, A e B R, B . Se B limitado


e A B, ento
i) sup A sup B e
ii) inf B inf A.
Demonstrao.
i) Por hiptese, B limitado, ento existe b R tal que x b para
todo x B. Como A B, temos x b para todo x A. Assim, A
limitado superiormente.
Sendo A e B limitados superiormente, A e B admitem supremos, digamos sup A = e sup B = . De sup B = , temos x
para todo x B. Como A B, ento vale x para todo x A.
Assim, uma cota superior do conjunto A. Mas, a menor das
cotas superiores de A. Logo, , ou seja, sup A sup B.
ii) Demonstrao anloga ao item i).

Exemplo 1.4. Determine, caso existam, o mximo, mnimo, supremo e nfimo do conjunto A = {x | x 2 4}.
Soluo. O conjunto A pode ser escrito como {x | -2 x 2}.
Temos min A = -2, max A = 2, inf A = -2 e sup A = 2.
Exemplo 1.5. Considere a funo f :[-2, 2] definida por
f ( x) = x 2 - 1. Encontre sup{ f ( x) | -2 x 2} e inf{ f ( x) | -2 x 2}.
Seja f :[a, b] R uma
funo contnua. Ento f
assume um valor mximo e
um valor mnimo. Isto ,
existem x1 , x2 [a, b] tais
que f ( x1 ) f ( x) f ( x2 )
para todo x [a, b].

Soluo. A funo f contnua no intervalo [-2, 2], pelo Teorema


de Weierstrass f assume em [-2, 2] valores mximo e mnimo que
so 3 e -1, respectivamente. Portanto,
sup{ f ( x) | -2 x 2} = 3 e inf{ f ( x) | -2 x 2} = -1.

Isso tambm fcil de ser verificado analisando o grfico da parbola


y = x 2 - 1.

18

1.2 Integrais inferior e superior


funes integrveis
Para definir integrais inferior e superior, precisamos de alguns conceitos relacionados partio de um intervalo.

Partio de um intervalo
Uma partio do intervalo [a, b] um conjunto finito P = {x0 , x1 ,..., xn }
onde a = x0 < x1 < ... < xn-1 < xn = b.
Os intervalos [ xi -1 , xi ] , i = 1,..., n sero chamados os intervalos da
partio P.

a = x0

x1

x2

xi 1

xi

xi +1

xn 1

xn = b

Figura 1.6

Os intervalos [ xi -1 , xi ] de P no precisam ter o mesmo comprimento. O nmero


| P |= max{( x1 - x0 ), ( x2 - x1 ),..., ( xn - xn -1 )}
chamado norma da partio P.
Sejam P e Q parties de [a, b]. Dizemos que Q mais fina do que
P, ou que Q um refinamento de P, se P Q.

Soma inferior e soma superior


Sejam f :[a, b] R uma funo limitada e P = {x0 , x1 ,..., xn } uma
partio de [a, b]. Definimos a soma inferior s ( f ; P) e a soma superior S ( f ; P) da funo f , referente partio P como sendo

Dizer que a funo


f :[a, b] limitada,
significa que existe c *+
tal que | f ( x) | c para todo
x [ a, b] .

19

s ( f ; P) = m1 ( x1 - x0 ) + m2 ( x2 - x1 ) + + mn ( xn - xn -1 )
n

= mi ( xi - xi -1 )
i =1

e
S ( f ; P) = M 1 ( x1 - x0 ) + M 2 ( x2 - x1 ) + + M n ( xn - xn -1 )
n

= M i ( xi - xi -1 )
i =1

onde

mi = inf{ f ( x); xi -1 x xi }, ou seja,


mi = nfimo dos valores f ( x) para x no intervalo [ xi -1 , xi ] ,
M i = sup{ f ( x); xi -1 x xi }, ou seja,
M i = supremo dos valores f ( x) para x no intervalo [ xi -1 , xi ] .

Note que
s( f ; P) =

m (x - x
i =1

i -1

M (x - x
i =1

i -1

) = S ( f ; P),

pois mi M i para todo i = 1, , n. Ou seja, a soma inferior de f menor ou igual soma superior de f relativa mesma partio.
Quando f contnua e no negativa em [a, b], podemos interpretar
a soma inferior s ( f ; P) como sendo uma soma de reas de retngulos inscritos ao grfico f , e assim um valor aproximado (por falta)
do que intuitivamente entendemos por rea da regio plana S, delimitada pelo grfico de f , pelas retas x = a e x = b, e pelo eixo x.
Similarmente, a soma superior S ( f ; P) pode ser interpretada como
uma soma de reas de retngulos circunscritos ao grfico de f , e
como um valor aproximado (por excesso) da rea da regio plana S.
A Figura 1.7 ilustra as observaes acima.

y = f ( x)

y = f ( x)

a = x0 x1 x2 x3 x4 = b

a = x0 x1 x2 x3 x4 = b
Figura 1.7

20

Exemplo 1.6. Calcular as somas inferior e superior para funo


1
f ( x) = x 2, definida no intervalo [0,1]. Usar a partio P = 0, ,1.
2
1
Soluo. Os intervalos da partio P so [ x0 , x1 ] = 0, e
2
1
[ x1 , x2 ] = ,1 . Temos
2
1

m1 = inf x 2 ;0 x = 0,
2

1
1
m2 = inf x 2 ; x 1 =
2
4

M 1 = sup x 2 ;0 x

1 1
= ,
2 4

1
e M 2 = sup x 2 ; x 1 = 1.
2

Segue que
s ( f ; P) = m1 ( x1 - x0 ) + m2 ( x2 - x1 )

1
1 1 1
= 0 - 0 + 1 - = ,
2
4 2 8
S ( f ; P) = M 1 ( x1 - x0 ) + M 2 ( x2 - x1 )
=

11
1 5
- 0 + 1 1 - = .
42
2 8

O que acontece com as somas inferior e superior quando acrescentamos um ponto partio P , ou em geral, quando refinamos P ? O
prximo teorema mostra que a soma inferior no diminui e a soma
superior no aumenta.
Teorema 1.2. Sejam f :[a, b] uma funo limitada,
P = {x0 , x1 ,..., xn } uma partio de [a, b] e Q um refinamento de P.
Ento
i) s ( f ; P) s ( f ; Q) e
ii) S ( f ; Q) S ( f ; P).
Demonstrao.
i) Vamos assumir inicialmente que Q obtida a partir de P acrescentando um ponto x , digamos Q = {x0 , x1 ,..., xi -1 , x , xi ,..., xn -1 , xn }.

21

Sejam mi , m ' e m '' os nfimos de f nos intervalos [ xi -1 , xi ],[ xi -1 , x ]


e [ x , xi ], respectivamente.
Temos
s ( f ; Q) = m1 ( x1 - x0 ) + m2 ( x2 - x1 ) + + mi -1 ( xi -1 - xi - 2 ) + m '( x - xi -1 ) +
m ''( xi - x ) + mi +1 ( xi +1 - xi ) + + mn ( xn - xn -1 )
e
s ( f ; P) = m1 ( x1 - x0 ) + m2 ( x2 - x1 ) + + mi -1 ( xi -1 - xi - 2 ) + mi ( xi - xi -1 ) +
mi +1 ( xi +1 - xi ) + + mn ( xn - xn -1 ).
Fazendo,
s ( f ; Q) - s ( f ; P) = m '( x - xi -1 ) + m ''( xi - x ) - mi ( xi - xi -1 )
= m '( x - xi -1 ) + m ''( xi - x ) - mi ( xi - x + x - xi -1 )
= m '( x - xi -1 ) - mi ( x - xi -1 ) + m ''( xi - x ) - mi ( xi - x )

= (m '- mi )( x - xi -1 ) + (m ''- mi )( xi - x ).
Como m ' mi e m '' mi , devido Proposio 1.3, temos
s ( f ; Q) - s ( f ; P) 0.

Portanto, se Q obtida a partir de P pelo acrscimo de um ponto,


temos s ( f ; P) s ( f ; Q).
Se Q possui vrios pontos a mais do que P, basta aplicar esse
resultado repetidamente e teremos s ( f ; P) s ( f ; Q).
ii) A demonstrao no caso das somas superiores muito parecida e
deixada como exerccio.

Como consequncia do teorema acima, temos que toda soma inferior menor ou igual a qualquer soma superior.
Corolrio. Seja f :[a, b] uma funo limitada. Para quaisquer
parties P, Q de [a, b] tem-se s ( f ; P) S ( f ; Q).
Demonstrao.
Consideremos a partio P Q. Temos que a partio P Q mais
fina do que P e Q . Pelo Teorema 1.2 segue que

22
s ( f ; P ) s ( f ; P Q) S ( f ; P Q) S ( f ; Q).

Portanto, s ( f ; P) S ( f ; Q).

Consideremos o conjunto das somas inferiores referentes a todas as


parties de [a, b]. Do corolrio acima, temos que qualquer soma superior uma cota superior para o conjunto. Segue que o conjunto
limitado superiormente, e pelo Teorema 1.1 possui supremo. Analogamente, qualquer soma inferior uma cota inferior para o conjunto
formado pelas somas superiores. Assim, faz sentido falar em nfimo
do conjunto formado pelas somas superiores. Essas observaes levam s definies de integrais inferior e superior.
Definio 1.3. Seja f :[a, b] uma funo limitada. Definimos a
integral inferior de f , denotada por
de f , denotada por

b
a

b
a

f ( x) dx, e a integral superior

, como sendo

f ( x) dx = sup s ( f ; P) e
P

f ( x) dx = inf S ( f , P ).
P

O supremo e o nfimo so tomados relativamente a todas as parties P do intervalo [a, b].

Quando a f ( x) dx = a f ( x) dx, dizemos que f integrvel em


[a, b]. Esse valor comum chamado de integral da funo f e

indicamos por

f ( x)dx.

Os nmeros a e b so respectivamente os limites inferior e superior


da integral, a funo f ( x) o integrando e o smbolo um sinal
b
de integrao. comum referir-se f ( x) dx como integral definia
da de f em [a, b].
b

A integral definida f ( x) dx um nmero. Podemos utilizar outras


a
letras para representar a varivel independente sem mudar o valor
da integral, ou seja,

f ( x) dx =

f (t ) dt =

f ( s ) ds.

23
Exemplo 1.7. (Exemplo de funo integrvel) Seja f :[a, b] R a
funo constante f ( x) = k . Mostre que f integrvel em [a, b] e que

f ( x) dx = k (b - a ).

Soluo. Seja P = {x0 , x1 ,..., xn } uma partio qualquer de [a, b].


Temos que
mi = inf{ f ( x); xi -1 x xi } = k e
M i = sup{ f ( x); xi -1 x xi } = k para todo i = 1,..., n.
Assim,
s ( f ; P ) = k (b - a )

e S ( f ; P) = k (b - a ).

Dado que P qualquer, temos

f ( x) dx = sup s ( f ; P ) = k (b - a ) e

f ( x) dx = inf S ( f , P) = k (b - a ).

Como as integrais inferior e superior so iguais, f integrvel em


[ a, b] e

f ( x) dx = k (b - a ).

Exemplo 1.8. (Exemplo de funo no integrvel) Seja f :[a, b] R


a funo definida por
1, se x racional
f ( x) =
-1, se x irracional .
Mostre que f no integrvel em [a, b].
Soluo. Seja P = {x0 , x1 ,..., xn } uma partio qualquer de [a, b]. Em
todo intervalo [ xi -1 , xi ] de P existem nmeros racionais e irracionais.
Assim,
mi = inf{ f ( x); xi -1 x xi } = -1
e
M i = sup{ f ( x); xi -1 x xi } = 1
para todo i = 1,..., n.
Segue que s ( f ; P) = -(b - a) e S ( f ; P) = (b - a) para qualquer partio P de [a, b]. Logo,

24

b
a

f ( x) dx = sup s ( f ; P ) = -(b - a )
P

f ( x) dx = inf S ( f ; P) = (b - a ).
P

Como

b
a

f ( x) dx f ( x) dx
a

temos que f no integrvel em [a, b] .


O Exemplo 1.8 mostra que nem toda funo limitada integrvel.
importante saber quais funes so integrveis. Os Teoremas 1.3 e
1.4, cujas demonstraes sero omitidas, garantem que um grande
nmero de funes integrvel.
Teorema 1.3. Se f :[a, b] uma funo contnua, ento f
integrvel.
Teorema 1.4. Seja f :[a, b] uma funo limitada, com um nmero finito de pontos de descontinuidade. Ento, f integrvel.

rea de uma regio plana


Seja f :[a, b] uma funo contnua tal que f ( x) 0 para todo
x [a, b]. Definimos a rea da regio plana S, limitada pelo grfico
de y = f ( x), as retas x = a, x = b e o eixo x, como sendo a integral de
f no intervalo [a, b] (ver figura abaixo). Escrevemos
b

rea S = f ( x)dx.
a

y = f (x)
S
a

b
Figura 1.8

Exemplo 1.9. Calcule a rea da regio S limitada pelo grfico da


funo f ( x) = 5, pelas retas x = 2 e x = 6, e o eixo do x.

25
Soluo. A Figura 1.9 mostra a regio S. Do Exemplo 1.7 temos que
a funo constante integrvel. Como f ( x) 0 para todo x [2, 6],,
segue que
6

rea S = 5 dx = 5(6 - 2) = 20 u.a. (unidades de rea).


2

y
f ( x) = 5

Figura 1.9

Exemplo 1.10. Calcule a


rea.

-3

9 - x 2 dx interpretando-a em termos de

Soluo. A funo f ( x) = 9 - x 2 contnua no intervalo [-3, 3] .


Pelo Teorema 1.3 f integrvel no intervalo [-3, 3] . Como f ( x) 0
para todo x [-3, 3], podemos interpretar a integral como rea sob a
curva y = 9 - x 2 , x de -3 at 3. Observe que y 2 = 9 - x 2, ou
x 2 + y 2 = 9 com y 0,
ou seja, o grfico da f a metade superior do crculo de raio 3.
Portanto,

-3

9 - x 2 dx =

32 9 .
=
2
2

1.2.1 Exerccios
1) Determine, caso existam, o mximo, mnimo, supremo e nfimo dos conjuntos.
n

; n .
a) A = {x ; - 2 x 5};
b) B =
n +1

2) Considere a funo f ( x) = - x 2 + 2 . Determine:


a) sup{ f ( x) ; - 3 x 1};

b) sup{ f ( x) ; - 3 x -1}.

26
1
no
x
intervalo [1, 3]. Usar a partio tal que o comprimento de cada

3) Calcular as somas inferior e superior para funo f ( x) =

intervalo da partio seja 1 .


2

-1, se 0 x 2
4) Considere a funo definida por f ( x) =
.
1, se 2 < x 4
A funo f integrvel em [0, 4]? Justifique.
5) Avalie a integral
rea.

4 - x 2 dx interpretando-a em termos de

6) Mostre o item ii) do Teorema 1.2.

1.3 Integral como limite de somas


Na seo anterior definimos a integral de uma funo usando a linguagem de supremo e nfimo de conjunto. Nosso objetivo agora
mostrar que a integral pode ser interpretada como limite de somas,
chamadas de somas de Riemann. Para isso precisamos estabelecer
alguns resultados.
Teorema 1.5. Seja f :[a, b] uma funo limitada. As seguintes
afirmaes so equivalentes:
i) f integrvel;
ii) Dado e > 0 qualquer, existem parties P e Q do intervalo
[a, b] tais que S ( f ; P ) - s ( f ; Q) < e;
iii) Dado e > 0 qualquer, existe uma partio R do intervalo [a, b]
tal que S ( f ; R) - s ( f ; R) < e.
Demonstrao. Mostraremos que (i) (ii) .
Temos que

b
a

f ( x) dx = sup s ( f ; P ) e
P

f ( x) dx = inf S ( f ; P).
P

Das propriedades de supremo e nfimo (Proposies 1.1 e 1.2), dado


e > 0, existem parties P e Q tais que

b
a

f ( x) dx -

e
< s( f ; Q) e
2

f ( x) dx +

e
> S ( f ; P ).
2

27
Segue das relaes acima e de f ser integrvel em [a, b] que
b
b
e
e
e e
S ( f ; P) < + f ( x) dx = + f ( x) dx < + + s ( f ; Q).
a
a
2
2
2 2
Portanto,
S ( f ; P ) - s ( f ; Q) < e.
Agora, vamos mostrar que (ii) (iii) .
Seja ee > 0 . Por hiptese, existem parties P e Q do intervalo [a, b]
tais que
S ( f ; P) - s( f ; Q) < e .
Tome a partio R = P Q de [a, b]. Ento, pelo Teorema 1.2, segue que
s( f ; Q) s( f ; R) e S ( f ; R) S ( f ; P).

Portanto, S ( f ; R) - s ( f ; R) < e.
Falta mostrar que (iii) (i) . Para quaisquer parties P e Q de
[a, b] temos, usando o Corolrio do Teorema 1.2, que
s( f ; P) S ( f ; Q) .

Ento, da definio de supremo podemos escrever que


sup s ( f ; P) S ( f ; Q) para qualquer partio Q de [a, b].
P

Segue que, sup s ( f ; P) uma cota inferior do conjunto formado peP

las somas superiores S ( f , Q) e, assim,


sup s ( f ; P) inf S ( f ; P).
P

Agora, vamos mostrar que sup s ( f ; P) = inf S ( f ; P).


P

Se fosse sup s ( f ; P) < inf S ( f ; P) ento


P

inf S ( f ; P) - sup s ( f ; P) > 0.


P

Tome e = inf S ( f ; P) - sup s ( f ; P). Por hiptese, para este e existe


P

uma partio R do intervalo [a, b] tal que


S ( f ; R ) - s ( f ; R ) < e .

Como

inf S ( f ; P) S ( f ; R) e s ( f ; R ) sup s ( f ; P )
P

(1)

28

podemos escrever
S ( f ; R) - s ( f ; R) inf S ( f ; P) - sup s ( f ; P) = e,
P

o que contraria a desigualdade (1) . Assim,


sup s ( f ; P) = inf S ( f ; P) ,
P

ou seja, a f ( x) dx = f ( x) dx . Portanto, f integrvel.


a

Lema 1.1. Sejam f :[a, b] uma funo limitada e P = { x0 , x1 ,..., xn }


uma partio de [a, b]. Se Q um refinamento de P onde Q = P {x }
ento S ( f ; P) - S ( f ; Q) 2 M | P | onde M = sup{| f ( x) |; x [a, b]} e
P a norma da partio P .
Demonstrao. Suponhamos que x esteja entre xi-1 e xi . Sejam
M i , M ' e M '' os supremos da f em [ xi -1 , xi ] , [ xi -1 , x ] e [ x , xi ] , respectivamente. Assim,
S ( f ; P) - S ( f ; Q) = M i ( xi - xi -1 ) - M '( x - xi -1 ) - M ''( xi - x )
= M i ( xi - x + x - xi -1 ) - M '( x - xi -1 ) - M ''( xi - x )
= M i ( xi - x ) + M i ( x - xi -1 ) - M '( x - xi -1 ) - M ''( xi - x )
= ( M i - M '')( xi - x ) + ( M i - M ')( x - xi -1 )
2 M ( xi - x ) + 2 M ( x - xi -1 )
= 2 M ( xi - xi -1 )
2M | P | .

Observao: Generalizando o Lema 1.1, se Q uma partio de


[a, b] obtida pelo acrscimo de n pontos partio P, ento
S ( f ; P) - S ( f ; Q) 2nM | P |.
Teorema 1.6. A integral superior de uma funo limitada
f :[a, b] o limite das somas superiores S ( f ; P) quando a norma da partio P tende a zero, ou seja,

f ( x) dx = lim S ( f ; P).
| P| 0

29

Demonstrao. Dado > 0. Temos que mostrar que existe > 0 tal que
b

0 < | P | < - + f ( x) dx < S ( f ; P) < + f ( x) dx .


a

Da definio de integral superior temos

f ( x) dx S ( f ; P) para qualquer partio P de [a, b].

Observe que
b

- + f ( x) dx < S ( f ; P) para qualquer partio P de [a, b].


a

Ainda, da definio de integral superior, para o > 0 dado existe uma


partio Q = {x0 , x1 ,..., xn } de [a, b] tal que
b

S ( f ; Q) < f ( x) dx + .
a
2

Tomemos =
, onde M = sup{| f ( x) |; x [a, b]}. Seja
4(n - 1) M
P uma partio arbitrria com 0 < | P | < . Considere a partio
R = P Q. Note que a partio R (um refinamento de Q ) obtida
a partir de P pelo acrscimo de no mximo n - 1 pontos, pois Q
possui n + 1 pontos onde x0 = a e xn = b. Da observao feita aps o
Lema 1.1, temos

S ( f ; P) - S ( f ; R) 2 M (n - 1) | P |.
Segue que
S ( f ; P) S ( f ; R) + 2 M (n - 1) | P |
< S ( f ; Q) + 2 M (n - 1)

4(n - 1) M

b

< f ( x) dx + + , sempre que 0 < | P | < .
a
2 2

Portanto,

lim S ( f ; P) = f ( x) dx.

| P| 0

De forma anloga, mostra-se que

b
a

f ( x) dx = lim s ( f ; P ).
| P| 0

30

Soma de Riemann
Sejam f :[a, b] uma funo limitada e P = {x0 , x1 ,..., xn } uma
partio de [a, b]. Uma soma de Riemann de f em relao partio P qualquer expresso S n ( f ) da forma
n

S n ( f ) = f (ci )( xi - xi -1 ),
i =1

onde ci um nmero em [ xi -1 , xi ] para i = 1, 2,..., n.


Se f (ci ) > 0 ento f (ci )( xi - xi -1 ) representa a rea do retngulo de
base ( xi - xi -1 ) e altura f (ci ) (ver Figura 1.10).

a = x0 c1 x1 c2 x2

xi 1ci xi xn 1 cn xn = b

Figura 1.10

Observe que independentemente da escolha de ci [ xi -1 , xi ] temos


s ( f ; P) S n ( f ) S ( f ; P),

ou seja,
n

m (x - x
i =1

i -1

i =1

i =1

) f (ci )( xi - xi -1 ) M i ( xi - xi -1 ),

pois mi f (ci ) M i .
Exemplo 1.11. Determine a soma de Riemann para f ( x) = 2 - x 2,
0 x 2 e P a partio de [0, 2] em 4 subintervalos de mesmo comprimento. Escolha ci como sendo o extremo direito do subintervalo
[ xi -1 , xi ].
Soluo. O nmero de subintervalos 4 , ou seja, n = 4. O comprimento dos subintervalos

31
b-a 2-0 1
=
= .
n
4
2

Os subintervalos so

1 1 3
3
0, 2 , 2 ,1 , 1, 2 e 2 , 2 .

Assim, c1 =

1
3
, c2 = 1 , c3 = e c4 = 2. Logo, a soma de Riemann
2
2
4

S 4 ( f ) = f (ci )( xi - xi -1 )
i =1

1
1
1 1
3 1
= f + f (1) + f + f (2)
2
2
2 2
2 2
=

1 7
1

+ 1 - - 2

2 4
4

1
= .
4
Teorema 1.7. Seja f :[a, b] uma funo limitada. As afirmaes
so equivalentes:
i) f integrvel;
n

ii) Existe o lim f (ci )( xi - xi -1 ), independentemente da escolha


| P| 0

i =1

de ci [ xi -1 , xi ] . Neste caso, lim f (ci )( xi - xi -1 ) = f ( x) dx .


| P| 0

i =1

Demonstrao.
i) ii) Do Teorema 1.6 e f integrvel temos
b

lim s ( f ; P) = f ( x) dx = lim S ( f ; P).


a

| P| 0

| P| 0

Observe que valem as relaes


n

s ( f ; P) f (ci )( xi - xi -1 ) S ( f ; P)
i =1

Sejam f, g e h funes com


o mesmo domnio D, sendo
f ( x ) g ( x ) h( x ) .
f
Se ( x) e h( x) tm o
mesmo limite L com x a
ento g ( x) tambm tem
limite L com x a .

independentemente da escolha de ci [ xi -1 , xi ].
Aplicando o limite nas desigualdades quando | P | 0 e o Teorema do
Confronto, conclumos que
n

lim f (ci )( xi - xi -1 ) = f ( x) dx.

| P| 0

i =1

32

ii) i) Para provar que f integrvel em [a, b], mostraremos que


para todo > 0, dado arbitrariamente, existe uma partio P de
[a, b] tal que S ( f ; P) - s ( f ; P) < .
n

Seja I = lim f (ci )( xi - xi -1 ). Da definio de limite, dado > 0


| P| 0

i =1

existe um > 0 tal que


n

f (c )( x - x

0<| P|<

i =1

i -1

)-I <

(2)

independentemente da escolha de ci [ xi -1 , xi ].
Fixemos P = {x0 , x1 ,..., xn } com 0 < | P | < e tomemos ci [ xi -1 , xi ]
de duas maneiras. Primeiramente, vamos escolher ci [ xi -1 , xi ] tal

que f (ci ) < mi +


, onde mi = inf{ f ( x); x [ xi -1 , xi ]},
4n( xi - xi -1 )
para cada i = 1, , n. Assim,
n

i =1

f (ci )( xi - xi -1 ) < mi ( xi - xi -1 ) +
i =1

= s( f , P) + ,
4
4

ou seja,
n

f (c )( x - x
i

i =1

i -1

)-

< s ( f , P ) .
4

(3)

Agora, vamos escolher ci [ xi -1 , xi ] tal que


f (ci ) > M i -

, onde M i = sup{ f ( x); x [ xi -1 , xi ]}.


4n( xi - xi -1 )

Assim,
n

f (c )( x - x
i

i =1

i -1

) > M i ( xi - xi -1 ) i =1

= S ( f , P) - ,
4
4

isto ,
n

f (c ) ( x - x
i =1

i -1

)+

> S ( f , P ) .
4

(4)

Das desigualdades (3) e (4) resultam que


n

f (c )( x - x
i =1

i -1

)-

< s( f ; P) S ( f ; P) <
4

f (c )( x - x
i =1

i -1

)+ .
4

33

De (2), temos que as somas de Riemann


n

i =1

f (ci )( xi - xi -1 ) e

f (c )( x - x
i =1

i -1

esto no intervalo I - , I + . Logo, s ( f ; P) e S ( f ; P) perten4


4

cem ao intervalo I - , I + , e assim S ( f ; P) - s ( f ; P) < . Por2


2

tanto, f integrvel.

O teorema acima garante que se f integrvel em [a, b], ento


o valor do limite
n

lim f (ci )( xi - xi -1 )

| P| 0

i =1

o mesmo, independentemente da escolha de ci , e igual a

f ( x) dx. Se, para uma escolha particular dos ci , encontrarn

mos lim f (ci )( xi - xi -1 ) = L, ento teremos


| P| 0

i =1

f ( x) dx = L.

Usaremos essa observao no prximo exemplo.

Exemplo 1.12. Considere a funo f :[a, b] definida por f ( x) = x.


A funo f integrvel em [a, b]? Justifique. Caso afirmativo, determine

x dx como limite de somas de Riemann.

Soluo. A funo f contnua em [a, b], ento pelo Teorema 1.3


temos f integrvel em [a, b].
Para determinar

x dx dividiremos o intervalo [a, b] em n subin-

tervalos de mesmo comprimento, formaremos as somas de Riemann


n

S n ( f ) = f (ci )( xi - xi -1 ) onde escolheremos ci como sendo o extrei =1

mo direito dos subintervalos da partio P, e calcularemos lim S n ( f ) ,


que ser

| P| 0

x dx. Com efeito, para cada n , consideremos

(b - a )
2(b - a )
(n - 1)(b - a )
n(b - a)

P = a, a +
,a+
,..., a +
,a+

n
n
n
n

34

a partio que consiste em dividir [a, b] em n partes iguais, cada


b-a
uma com comprimento
. Para cada subintervalo
n
(b - a )
(b - a )

a + (i - 1) n , a + i n ,
temos ci = a + i

(b - a )
(b - a )
e f (ci ) = a + i
.
n
n

A soma de Riemann
Sn ( f ) =

f (c )( x - x
i =1

i -1

(b - a )
(b - a )
2(b - a )
(n - 1)(b - a )
n(b - a )
a+
+a+
+ + a +
+a+

n
n
n
n
n

(b - a )
(b - a )

[1 + 2 + + (n - 1) + n] (usar soma de P.A. com n termos)


na +
n
n

= a (b - a ) +

(b - a ) 2 (1 + n)n
n 2 2

= ab - a 2 +

(b - a ) 2 (b - a ) 2
+
2n
2

b 2 - a 2 (b - a ) 2
.
+
2
2n

Como os comprimentos dos intervalos da partio P so iguais a


b-a
, fazer P tender a zero equivale a fazer n tender a . Logo,
n
b 2 - a 2 (b - a ) 2 b 2 - a 2
.
+
lim S n ( f ) = lim
=
n
| P| 0
2
n
2
Portanto,

x dx =

b2 - a 2
.
2

1.3.1 Exerccios
1) Determine a soma de Riemann S n ( f ) da funo f ( x) = 3 x - 2,
onde P = {x0 , x1 , x2 , x3 , x4 } a partio do intervalo [1,5] em
quatro subintervalos de mesmo comprimento, e:
a) ci o extremo direito de intervalo [ xi -1 , xi ] ;

35

b) ci o extremo esquerdo do intervalo [ xi -1 , xi ] ;


c) ci o ponto mdio de [ xi -1 , xi ].
2) Considere a funo f :[0,1] definida por f ( x) = x 2.
i) f integrvel em [0,1]? Justifique.
ii) Encontre

x 2 dx como limite de somas de Riemann.

Sugesto. Dividir o intervalo [0,1] em n partes


iguais, escolher ci como sendo o extremo direito dos
subintervalos, e usar a relao
n(n + 1)(2n + 1)
.
12 + 22 + 32 + + n 2 =
6
3) Considere a funo f :[a, b] R definida por f ( x) = e x.
i) f integrvel em [a, b]? Justifique.
ii) Encontre

e x dx como limite de somas de Riemann.

1.4 Propriedades da integral


b

Na definio f ( x) dx, assumimos a < b. Nos casos em que a = b e


a
a > b definimos as integrais como sendo

f ( x) dx = 0 e

f ( x) dx = - f ( x) dx,
b

respectivamente.
Teorema 1.8. Sejam f , g funes integrveis em [a, b] e k . Ento:
a) k f integrvel em [a, b] e

k f ( x) dx = k f ( x) dx;
a

b) f + g integrvel em [a, b] e

[ f ( x) + g ( x)] dx = f ( x)dx + g ( x)dx ;

c) Se f ( x) 0 para todo x [a, b], ento

d) Se f ( x) g ( x) para todo x [a, b], ento

f ( x) dx 0;

f ( x) dx g ( x) dx.
a

Demonstrao. Vamos provar os itens a), c) e d). O item b) deixamos


como exerccio.

36

a) Seja P = {x0 , x1 ,..., xn } uma partio de [a, b] . Toda soma de


Riemann da funo k f da forma
n

k f (c )( x - x
i

i =1

i -1

) onde ci [ xi -1 , xi ].

Como f integrvel em [a, b], ento existe


n

lim f (ci )( xi - xi -1 )

| P| 0

i =1

independentemente da escolha de ci [ xi -1 , xi ], e igual a

f ( x) dx.

Assim, usando as propriedades de limite de funes, obtemos


n

lim k f (ci )( xi - xi -1 ) = k lim k f (ci )( xi - xi -1 )


P 0

P 0

i =1

i =1

= k f ( x)dx.
b

Portanto, k f integrvel em [a, b] e k f ( x) dx = k f ( x) dx.


c) Seja P = { x0 , x1 ,..., xn } uma partio de [a, b] . Como f integrvel em [a, b] , ento existe
n

lim f (ci )( xi - xi -1 )

| P| 0

i =1

independentemente da escolha de ci em [ xi -1 , xi ], e
b
f ( x) dx.
a

Como f (ci ) 0 para todo ci [ xi -1 , xi ] segue que


n

f (c )( x - x
i

i =1

i -1

) 0.

Portanto, por propriedade de limite de funes,


n

lim f (ci )(xi - xi -1 ) 0, ou seja,

| P| 0

i =1

f ( x) dx 0 .

d) De f ( x) g ( x) para todo x [a, b] segue que


( f - g )( x) 0 para todo x [a, b].
Dos itens a) e b) temos que f - g integrvel em [a, b] e

( f - g )( x)dx = f ( x) dx - g ( x) dx.

Aplicando o item (c) funo f - g, temos

a
b

( f - g )( x) dx = f ( x) dx - g ( x) dx 0 .

Portanto,

f ( x) dx g ( x) dx.
a

37

Exemplo 1.13. Calcule a integral

(3 x + 4) dx .

Soluo. Dos Exemplos 1.7 e 1.12, temos

4dx = 4(5 - 1) = 16 e

x dx =

25 - 1
= 12.
2

Pelos itens a) e b) do Teorema 1.8, podemos escrever

(3 x + 4) dx = 3 x dx + 4dx
1

= 3 12 + 16 = 52.

Exemplo 1.14. Sejam f uma funo integrvel em [a, b] e m, M R.


Mostre que, se m f ( x) M para todo x [a, b] , ento
b

m(b - a ) f ( x) dx M (b - a ) .
a

Soluo. Considere as funes g e h definidas por g ( x) = m e


h( x) = M para todo x [a, b]. Temos que g e h so integrveis em
[a, b] , pois so funes constantes, e

mdx = m(b - a ) e

Mdx = M (b - a ).

Como m f ( x) M para todo x [a, b] , segue pelo item (d) do


Teorema 1.8 que
b

m(b - a ) f ( x) dx M (b - a ).
a

Teorema 1.9. Se a < c < b e f integrvel em [a, c], em [c, b] e em


[a, b] , ento

f ( x) dx = f ( x) dx + f ( x) dx.

Demonstrao. Seja P = {x0 , x1 ,..., xn } uma partio de [a, b]. Como


c (a, b) ento ou c um ponto da partio P, isto , c = xi para
algum i, ou c est no interior de algum subintervalo da partio P,
ou seja, c ( xi -1 , xi ). Considere a partio P ' de [a, b] formada da
seguinte maneira: se c for um ponto da partio P ento P ' ser
a prpria P. Se c ( xi -1 , xi ) para algum i, ento P ' ser a partio formada por todos os pontos de P mais o ponto c . Assim, os
subintervalos da partio P ' sero os mesmos de P, com exceo
do subintervalo [ xi -1 , xi ] que poder ser dividido em [ xi -1 , c] e [c, xi ].
Dessa forma teremos P ' P .

38

Suponhamos que na partio P ' o intervalo [a, c] foi dividido em l


subintervalos e o intervalo [c, b] foi dividido em n - l subintervalos.
Sendo a funo f integrvel em [a, b], podemos escrever

f ( x) dx = lim f (ci )( xi - xi -1 )
| P| 0

i =1

n
l

= lim f (ci )(xi - xi -1 ) + f (ci )( xi - xi -1 )


| P| 0
i =l +1
i =1

= lim f (ci )( xi - xi -1 ) + lim


| P| 0

| P| 0

i =1

i =l +1

f (ci )( xi - xi -1 ) .

Como P ' P , temos que P ' 0 quando P 0. Assim,

f ( x) dx = lim f (ci )( xi - xi -1 ) + lim


| P '| 0

| P '| 0

i =1

f (c )( x - x
i

i =l +1

i -1

).

Como a funo f integrvel em [a, c] e [c, b] temos que os limites


na igualdade acima so respectivamente

f ( x) dx e

f ( x) dx .

Portanto,

f ( x) dx = f ( x) dx + f ( x) dx.

2, se 0 x 2
Exemplo 1.15. Considere a funo f ( x) =
.
x, se 2 < x 5

5
Calcular a integral f ( x) dx.
0

Soluo. A funo f integrvel em [0,5], pois contnua


(Teorema 1.3).
Temos que f integrvel em [0, 2] e, pelo Exemplo 1.7, obtemos

f ( x) dx = 2dx = 2(2 - 0) = 4.
0

f integrvel em [2,5] e, pelo Exemplo 1.12, obtemos

f ( x) dx =

52 - 22 21
= .
x dx =
2
2

39
Logo, pelo Teorema 1.9, temos

f ( x) dx = 2dx + x dx

= 4+

21 29
= .
2
2

1.4.1 Exerccios

1) Calcule a integral

x sen x 2 dx.

2) Escreva a integral como uma nica integral da forma


5

a)

a)

3) Se

2
3

10

f ( x) dx.

f ( x) dx + f ( x) dx + f ( x) dx .
3

f ( x) dx - f ( x) dx + f ( x) dx .
f (t ) dt = 1,

Encontre

f (t ) dt = -2 e

f (t ) dt = 2.

2 f (t ) dt .

4) Mostre o item (b) do Teorema 1.8.

1.5 Teorema fundamental do clculo


Este teorema
foi estabelecido
independentemente por Sir
Isaac Newton (1642-1727)
na Inglaterra e Gottfried
Leibniz (1646-1716) na
Alemanha.

Calcular integrais definidas usando somas de Riemann trabalhoso, mesmo para funes simples. Nesta seo vamos demonstrar
o Teorema Fundamental do Clculo que estabelece uma conexo
entre as operaes de derivao e integrao. Este teorema permite
encontrar a integral definida, para uma certa classe de funes, de
maneira rpida e simples sem utilizar limites de somas. Para isso,
introduzimos o conceito de primitiva de uma funo.
Definio 1.4. Sejam I um intervalo e f : I uma funo. Uma
primitiva de f em I uma funo derivvel F
f : I tal que
F '( x) = f ( x) para todo x I .
Exemplo 1.16.
a) Se f ( x) = 2 x, ento F ( x) = x 2 uma primitiva de f , pois
F '( x) = f ( x).

40

b) Se f ( x) = 2e 2 x, ento F ( x) = e 2 x uma primitiva de f , pois


F '( x) = f ( x).
c) Se f ( x) = sen x, ento F ( x) = - cos x uma primitiva de f , pois
F '( x) = f ( x).
sen (2 x)
d) Se f ( x) = cos(2 x), ento F ( x) =
uma primitiva de f ,
2
pois F '( x) = f ( x).
3x
e) Se f ( x) = 3x, ento F ( x) =
uma primitiva de f , pois
ln 3
F '( x) = f ( x).
x5
4
f) Se f ( x) = x , ento F ( x) =
uma primitiva de f , pois
5
F '( x) = f ( x).
x5
g) Para toda constante c (c R), F ( x) = + c uma primitiva da
5
funo f ( x) = x 4.
Note que se F ( x) uma primitiva de f em [a, b] ento para toda
constante c (c R), a funo G ( x) = F ( x) + c tambm primitiva de
f em [a, b].
A proposio a seguir, estabelece que se F ( x) uma primitiva particular de f ento toda primitiva de f da forma F ( x) + c.
Proposio 1.4. Se F , G :[a, b] so primitivas da funo
f :[a, b] , ento existe uma constante c R tal que G ( x) = F ( x) + c
para todo x [a, b].
Demonstrao. Considere a funo H :[a, b] definida por
H ( x) = G ( x) - F ( x) . Temos que
H '( x) = ( G ( x) - F ( x) ) ' = G '( x) - F '( x) = f ( x) - f ( x) = 0

para todo x (a, b).


Como a funo H possui derivada nula em todos os pontos de (a, b),
segue de um resultado visto no Clculo I, que H uma funo
constante. Portanto, existe c tal que G ( x) - F ( x) = c, ou seja,
G ( x) = F ( x) + c para todo x [a, b].

Observao. A Proposio 1.4 vale para um intervalo I qualquer.

Se uma funo contnua


f :[a, b] possui
derivada nula em todos os
pontos c (a, b) , ento f
constante. (Pgina 223 do
material de Clculo I)

41
Teorema 1.10. (Teorema Fundamental do Clculo TFC)
Se f :[a, b] uma funo integrvel e F :[a, b] uma
primitiva de f ento

f ( x) dx = F (b) - F (a ) .

Demonstrao. Seja P = {x0 , x1 ,..., xn } uma partio de [a, b] .


Temos que F derivvel em [a, b] , e consequentemente F contnua em [a, b] . Segue que F contnua e derivvel em cada intervalo
[ xi -1 , xi ] da partio P. Aplicando o Teorema do Valor Mdio em cada
intervalo de P, existe ci ( xi -1 , xi ) tal que

Seja uma funo contnua


em [a, b] e derivvel em
. Ento existe
tal que
.

F '(ci ) =

F ( xi ) - F ( xi -1 )
para todo i = 1,..., n.
xi - xi -1

Como F '(ci ) = f (ci ), resulta que


F ( xi ) - F ( xi -1 ) = f (ci )( xi - xi -1 ).
Podemos escrever
n

S n ( f ) = f (ci )( xi - xi -1 ) ( ci escolhido como acima)


i =1
n

= [ F ( xi ) - F ( xi -1 )]
i =1

= F (b) - F (a ) .

Assim,

lim S n ( f ) = F (b) - F (a ).

| P| 0

Como f integrvel, o valor deste limite o mesmo, independenteb


mente da escolha dos ci e, portanto, f ( x) dx = F (b) - F (a ).
a

comum expressar a diferena F (b) - F (a ) por F ( x)

b
.
a

Exemplo 1.17. Calcular as seguintes integrais definidas:


a)

x 2 dx;

b) (2 x3 + 3) dx ;
-1

c) (sen x + x) dx.
0

42
Soluo.
x3
uma primitiva da funo f ( x) = x 2. Logo,
3
3
2
x 2
2
1 x dx = 3 1

a) A funo F ( x) =

23 13 7
- = .
3 3 3

b) Aplicando as propriedades da integral e o Teorema Fundamental


do Clculo, temos

-1

-1

-1

(2 x 3 + 3) dx = 2 x 3 dx + 3dx

=2

1
x4 1
+ 3x
-1
4 -1

14 (-1) 4
= 2 + 3[1 - (-1)]
4
4
= 6.

c) (sen x + x) dx = sen x dx + x dx
0

= - cos x

x 2 2
2 0

1
= - cos - (- cos 0) +
2

2 2
= 1+

Exemplo 1.18. Calcule

-2

2 2 + 8
.
=
8
8

| x | dx.

Soluo. A funo f ( x) = | x | pode ser escrita como


- x, se - 2 x 0
f ( x) =
.
x, se 0 < x 2
Das propriedades da integral e o Teorema Fundamental do Clculo,
temos
2
0
2
|
x
|
dx
=
x
dx
+

x dx
-2

-2

=-

x2 0 x2 2
+
2 -2 2 0

= 2+2 = 4.

43

O Teorema Fundamental do Clculo pode ser aplicado para


calcular a integral definida de uma funo f, quando se conhece uma primitiva de f. No que segue, mostraremos que
toda funo contnua em [a, b] possui uma primitiva.

Se uma funo f contnua em [a, b], ento f integrvel no intervalo [a, x] para qualquer x [a, b]. Para cada x [a, b] a integral

f (t ) dt um nmero e nico. Assim podemos definir uma fun-

o G que a cada x [a, b] associa esse nmero. Mostraremos que


G uma primitiva de f .
Teorema 1.11. Se f :[a, b] uma funo contnua ento a funo
x
G :[a, b] , definida por G ( x) = f (t ) dt , derivvel e G '( x) = f ( x)
a
para todo x [a, b].
Demonstrao. Seja c [a, b]. Para encontrar a derivada da funo
G determinaremos as derivadas laterais da G no ponto c . Inicialmente mostraremos que
G (c + h ) - G (c )
= f (c).
h

lim+

h 0

Vamos assumir h > 0 e admitir que c + h [a, b], isto , c [a, b) .


Pela definio da G e o Teorema 1.9, temos
G (c + h ) - G (c ) =

c+h

f (t ) dt - f (t ) dt
a

c+h

= f (t ) dt +
=

c+h

f (t ) dt - f (t ) dt
a

f (t ) dt .

A funo f contnua em [c, c + h], ento pelo Teorema de Weierstrass existem x1 , x2 [c, c + h] tais que
f ( x1 ) f (t ) f ( x2 ) para todo t [c, c + h] .
Pelo Exemplo 1.14, podemos escrever
f ( x1 )h

c+h

f (t ) dt f ( x2 ) h.

44
Como h > 0 e
f ( x1 )

c+h

f (t ) dt = G (c + h) - G (c), temos

G (c + h ) - G (c )
f ( x2 ).
h

(5)

Note que se h 0+ ento x1 c + e x2 c +, pois x1 e x2 esto


entre c e c + h . Da continuidade de f podemos escrever
lim f ( x1 ) = lim+ f ( x1 ) = f (c) e

h 0+

x1 c

lim f ( x2 ) = lim+ f ( x2 ) = f (c).

h 0+

x2 c

Fazendo h 0+ na desigualdade (5) e aplicando o Teorema do Confronto, obtemos


lim+

h 0

G (c + h ) - G (c )
= f ( c ),
h

ou seja, para c tal que c [a, b) , temos


G'+ (c) existe e G+' (c) = f (c).

De forma anloga, mostra-se que G-' (c) = f (c) para c (a, b] e assim
G ' (c) = f (c). Observe que, para a e b, temos apenas G+' (a ) = f (a )
e G-' (b) = f (b). Segue que G derivvel e G '( x) = f ( x) para todo
x [a, b].

Exemplo 1.19. Encontre a derivada da funo G ( x) = sen t 2 dt.


0

Soluo. Como f (t ) = sen t 2 contnua, ento, pelo Teorema 1.11,


temos que G '( x) = f ( x) = sen x 2.
Exemplo 1.20. Calcule

d x2 t
e dt .
dx 1

Soluo. Vamos aplicar o Teorema 1.11 juntamente com a Regra da


Cadeia. Fazendo u = x 2, temos
d x2 t
d u
e dt = et dt

dx 1
dx 1
d u t du
=
e dt
dx
du 1
= eu 2 x
2

= 2 xe x .

45

1.5.1 Exerccios
1) Calcular as integrais abaixo:
a)

-2

(2 x + x 4 ) dx ;

b) cos 2 x dx ;
0

c)

3e

d)

3 dt.

2x

dx;

2) Achar as derivadas das seguintes funes:


a)

;
3

b) F ( x) = ln t 2 dt ;
x

c) H ( x) =

-x

(Sugesto. Escreva

ln t 2 dt = - ln t 2 dt )
3

cos t dt .

3) Calcule a integral definida

-3

| x - 4 | dx.

1.6 Integral indefinida


O Teorema Fundamental do Clculo estabelece uma relao entre
primitiva e integral definida. Para representar a famlia de todas
as primitivas de uma funo f , introduzimos a notao f ( x) dx,
que de acordo com a definio abaixo ser chamada de integral
indefinida.
Definio 1.5. Se F ( x) uma primitiva de f ( x) em um intervalo I , a
expresso F ( x) + c, onde c uma constante arbitrria, chamada integral indefinida da funo f ( x) e denotada por f ( x) dx = F ( x) + c.
Da definio acima, temos

f ( x) dx = F ( x) + c

F '( x) = f ( x).

Note que a integral indefinida f ( x) dx representa uma famlia de


funes (a famlia de todas as primitivas), enquanto a integral defib
nida f ( x) dx um nmero.
a

46

Propriedades da integral indefinida


Teorema 1.12. Sejam f , g : I funes definidas em um intervalo I , que possuem primitivas, e k uma constante no nula. Ento:
a)

k f ( x) dx = k f ( x) dx;

b)

[ f ( x) + g ( x)] dx = f ( x) dx + g ( x) dx .
Demonstrao.
a) Seja F uma primitiva de f . Temos k F uma primitiva da funo k f , pois
(kF ( x))' = kF '( x) = k f ( x) para todo x I .
Portanto,

k f ( x) dx = k F ( x) + c
c

= k F ( x) +
k

= k [ F ( x) + c1 ]

= k f ( x) dx.
b) Deixamos como exerccio.

Exemplo 1.21.
x 4
x4
a) x dx = + c, pois = x3.
4
4
e 2 x
e2 x
2x
b) e 2 x dx =
+ c, pois
=e .
2
2

c) sen x dx = - cos x + c, pois (- cos x) ' = sen x.


Podemos construir uma tabela de integrais a partir das derivadas
das funes elementares. Chamamos essas integrais de imediatas.
No que segue listamos algumas integrais imediatas.

47

Tabela de integrais imediatas


1) dx = x + c
2) x dx =
3)

x +1
+ c, ( constante e -1)
+1

x dx = ln | x | +c

4) e x dx = e x + c
5) a x dx =

ax
+ c, a > 0 e a 1
ln a

6) sen x dx = - cos x + c
7) cos x dx = sen x + c
8) sec 2 x dx = tg x + c
9) cossec 2 x dx = - cotg x + c
10) sec x tg x dx = sec x + c
11) cossec x cotg x dx = - cossec x + c
12)

13)

14)

1
dx = arc tg x + c
+1

1
1 - x2

dx = arcsen x + c

1
x2 -1

dx = arc sec | x | +c.

48
1

Exemplo 1.22. Calcular a integral indefinida + sen x dx.


x

Soluo.
1

x + sen x dx = x dx + sen x dx

(propriedade da integral)

= ln | x | +c1 - cos x + c2 (c1 e c2 constantes arbitrrias)


= ln | x | - cos x + c (c = c1 + c2).

x3 + 3x 2 + 4
Exemplo 1.23. Calcular a integral indefinida
dx.
x

Soluo.
x3 + 3x 2 + 4
4

dx = x 2 + 3 x + dx

x
x

1
= x 2 dx + 3 x dx + 4 dx (propriedades da integral)
x
3
2
x
x
= + c1 + 3 + c2 + 4 ln | x | + c3
3
2
3
2
x
x
= + 3 + 4 ln | x | +c ( c = c1 + c2 + c3 ).
3
2

Observao. Quando tivermos uma soma de vrias integrais indefinidas, escreveremos uma nica constante para indicar a soma das
constantes de integrao.
2
sen x
+ 3 dx.
Exemplo 1.24. Calcule a integral indefinida
2
cos x x

Soluo.
2
sen x
1
sen x
+ 3 dx =
dx + 2 3 dx
2
2
x x
cos x
x

cos

= tg x sec x dx + 2 x -3 dx
2 x -2
+c
-2
1
= sec x - 2 + c.
x
= sec x +

Exemplo 1.25. Calcule a integral definida

2
dx .
x +1
2

49

Soluo.

1 dx
2dx
= 2 2
2
0
x +1
x +1

= 2 arc tg x

1
(tabela - item 12 e TFC)
0


= 2 - 0 = .
4
2

1.6.1 Exerccios
1) Calcule as integrais indefinidas:
a) (3 x 2 + x 4 + 1) dx;

d)

x 2 ln x
ln x 2 dx;

t3 + 9
dt ;
t2

e)

1 - sen

c) e - x dx;

f)

b)

sen x
2

dx;

4
dx.
1- x 2

2) Verifique por diferenciao que a igualdade est correta.


a)

x
1
dx = ln(1 + x 2 ) + c;
2
1+ x
2

b) x 2 e x dx = e x ( x 2 - 2 x + 2) + c;
1

c)

2 3

(4 - x )

dx =

x
4 4 - x2

+ c.

3) Determinar a funo f ( x) tal que:


1
f ( x) dx = 3sen x + 2 x - 3x6 + c.
4) Calcule as integrais definidas:
2

a) (e 2 x + x 2 ) dx;
0

b) 2 (3cos + 2) d .
0

50

1.7 Tcnicas de integrao


Muitas vezes, para calcular uma integral indefinida precisamos
usar certos artifcios matemticos para transform-la em outra integral mais simples de ser obtida. Nesta seo vamos apresentar duas
tcnicas bsicas para calcular integrais indefinidas, que so:
Mtodo da Substituio ou Mudana de Varivel, e
Mtodo da Integrao por Partes.
No prximo captulo, veremos outras tcnicas de integrao.

1.7.1 Mtodo da substituio ou mudana


de varivel
Sejam f e F funes tais que F = f . Suponhamos que g seja outra
funo derivvel tal que a imagem da g esteja no domnio de F .
Podemos considerar a funo composta F G
g. Aplicando a Regra da
Cadeia e usando o fato F = f temos
[ F ( g ( x))] = F ( g ( x)) g ( x)
= f ( g ( x)) g ( x).
Assim, usando a Definio 1.5, obtemos a frmula de integrao
pelo mtodo da substituio

f ( g ( x)) g ( x) dx = F ( g ( x)) + c.
Se fizermos a mudana de varivel u = g ( x) e substituirmos g '( x) dx
pela diferencial du, ento

f ( g ( x)).g ( x) dx = f (u ) du = F (u ) + c.
A tcnica da mudana de varivel uma ferramenta poderosa para
calcular integrais indefinidas, que permite substituir uma integral
relativamente complicada por uma mais simples. Vejamos alguns
exemplos.

Diferenciais foram vistas


no Clculo 1. Se u = g ( x)
uma funo diferencivel,
ento du = g ( x) dx .

51

Exemplo 1.26. Calcular as integrais indefinidas:


a) 2 x e x dx;
2

b) cos (3 x + 2) dx ;
c) 3 x 2 ( x 3 + 2)10 dx .
Soluo.
a) Para calcular a integral
varivel

2x e

x2

dx, faremos a mudana de

u = x 2 e obtemos du = 2 x dx.
Logo,

2x e

x2

dx = eu du
= eu + c (voltando varivel inicial x )
2

= e x + c.
b) Para calcular a integral cos(3 x + 2) dx, faremos a substituio
du
u = 3 x + 2 e obtemos du = 3dx ou
= dx.
3
Assim,
cos u
cos(3x + 2) dx = 3 du
1
= cos u du
3
1
= sen u + c
3
1
= sen(3 x + 2) + c.
3
c) Encontraremos a 3 x 2 ( x3 + 2)10 dx fazendo a mudana de varivel
u = x3 + 2. Segue que du = 3 x 2 dx.
Logo,

3x ( x
2

+ 2)10 dx = u10 du
=

u11
+c
11

( x 3 + 2)11
+ c.
11

52

Exemplo 1.27. Use o mtodo da substituio para mostrar que

tg x dx = ln | sec x | +c.
Soluo.
sen x

tg x dx = cos x dx.
Fazendo u = cos x obtemos du = - sen x dx.
Assim,
du
tg x dx = - u
= - ln | u | +c
= ln

1
+c
cos x

= ln | sec x | +c.
Exemplo 1.28. Calcule a integral indefinida

1+ x dx.

Soluo. Fazendo t 2 = 1 + x com t 0, obtemos 2t dt = dx.


Assim,

1 + x dx = (t 2 - 1) 2 t 2t dt

= 2 (t 2 - 1) 2 t 2 dt
= 2 (t 4 - 2t 2 + 1) t 2 dt
= 2 (t 6 - 2t 4 + t 2 ) dt
t 7 2t 5 t 3
= 2 + +c
5
3
7
t 4 2t 2 1
= 2t 3 + +c
5 3
7
(1 + x) 2 2(1 + x) 1
= 2(1 + x) (1 + x)
+ + c.
5
3
7
Exemplo 1.29. Calcule a integral
Soluo.

dx
dx
=
- 2x + 5
( x - 1) 2 + 4

dx
.
- 2x + 5

53

Fazendo u = x - 1 obtemos du = dx . Assim,

dx
1
= 2
du
- 2x + 5
u +4
1
= 2 4 du
u +4
4
1
du
=
4 u 2
+1
2

1 2dt
u
(fazendo t = obtemos 2dt = du )
2

4 t +1
2

1 dt
(tabela)
2 t2 +1

1
arc tg t + c
2

1
u
arc tg + c
2
2

1
( x - 1)
arc tg
+ c.
2
2

O mtodo da substituio de varivel pode ser usado para calcular


integrais definidas. Podemos utiliz-lo de duas formas:
1) Mudamos os limites de integrao ao fazer a mudana de varivel e aplicamos o Teorema Fundamental do Clculo. Nesse
caso, a frmula da integrao torna-se:

f ( g ( x)) g '( x) dx =

g (b )

g (a)

(u = g ( x))

f (u ) du .

2) Calculamos a integral indefinida correspondente e em seguida


aplicamos o Teorema Fundamental do Clculo.
Exemplo 1.30. Calcule a integral definida

4x
dx.
x +1
2

Soluo. Fazendo u = x 2 + 1 obtemos du = 2 x dx. Para encontrar os


novos limites de integrao, notemos que

54
se x = 0 ento u = 1;
se x = 2 ento u = 5.
Assim,

5 du
4x
dx = 2
1 u
x +1
2

= 2 ln | u |

5
1

= 2 ln 5 - 2 ln1
= 2 ln 5.
Outra maneira de calcular a integral definida obter primeiramente
a integral indefinida e, em seguida, aplicar o Teorema Fundamental
do Clculo. Vejamos:

4x
du
(u = x 2 + 1, temos du = 2 x dx)
dx = 2
+1
u

= 2 ln | u | +c
= 2 ln( x 2 + 1) + c.
Aplicando o TFC, temos

4x
dx = 2 ln ( x 2 + 1)
2
x -1
0
= 2 ln 5 - 2 ln1
= 2 ln 5.

Exemplo 1.31. Encontre a rea da regio S limitada pelo grfico da

funo f ( x) = sen 2 x , pelas retas x = 0 e x = , e o eixo dos x.


2
Soluo. A regio S est ilustrada na Figura 1.11.

1
S

Figura 1.11

55

Como f ( x) 0 para todo x 0, , a rea da regio S (ver definio
2
na Seo 1.2) dada por

rea S = sen 2 x dx.


0

Fazendo u = 2 x, obtemos du = 2dx. Os novos limites de integrao


so:
se x = 0, ento u = 0;

se x = , ento u = .
2
Logo,

rea S = sen 2 x dx
0

1
sen u du
2 0

1
= - cos u
0
2
1
= - (cos - cos 0)
2
= 1u.a.

1.7.1 Exerccios
1) Calcular as integrais indefinidas.
a) 3 3 x - 1 dx;

b) cos(5 x + 2) dx;
x

c)

x +4

dx;

ln x
dx ;
x

d)

e)

cotg x dx;

f)

dx
.
+ 4 x + 20

2) Calcular as integrais definidas usando o mtodo da mudana


de varivel.
a)

b)

x2
dx;
x3 + 1

c)

3dx
;
x ln 2 3 x

d)

x e3 x dx ;
2

2 x 3x dx .

56
3) Calcule a integral sec x dx.
Sugesto. Escreva sec x = sec x
4) Mostre que

(sec x + tg x)
.
(sec x + tg x)

dx
1
x
= arc tg + c, onde a 0.
2
+a
a
a

1.7.2 Mtodo da integrao por partes


Sejam f e g funes derivveis num mesmo intervalo I . Pela regra
da derivada do produto, temos:
[ f ( x) g ( x)]' = f '( x) g ( x) + f ( x) g '( x) .
Note que [ f ( x) g ( x)] uma primitiva de [ f ( x) g ( x)]. Assim, podemos escrever

[ f ( x) g ( x) + f ( x) g ( x)] dx = f ( x) g ( x) + c ,
1

ou ainda

f ( x) g ( x) dx = f ( x) g ( x) - f ( x) g ( x) dx + c .
1

Observe que ao desenvolver a integral no segundo membro surgir


outra constante de integrao. Supriremos a constante c1 na frmula
acima e no final do processo introduziremos uma constante c para
representar todas as constantes de integrao envolvidas. Desse
modo podemos escrever:

f ( x) g ( x) dx = f ( x) g ( x) - f ( x) g ( x) dx,
que a frmula de integrao por partes.
Vamos reescrever a frmula da integrao por partes, usando uma
notao que se torna fcil de ser memorizada. Fazendo
u = f ( x) e v = g ( x), temos du = f '( x)dx e dv = g '( x) dx.
Ento a frmula da integrao por partes pode ser escrita como

u dv

= u v - v du.

57
Exemplo 1.32. Calcular a integral

x cos x dx.

Soluo. Vamos aplicar o mtodo da integrao por partes para calcular a integral. Para isso, devemos escolher u e dv . Fazendo
u=x

e dv = cos x dx , temos

du = dx e v = sen x.

Aplicando a frmula da integrao por partes, obtemos

x cos x dx = x sen x - sen x dx


= x sen x + cos x + c .

Exemplo 1.33. Calcular a integral ln x dx.


Soluo. Fazendo
u = ln x

du =

Logo,

e dv = dx , temos

1
dx e
x

v = x.

ln x dx = x ln x - x x dx
= x ln x - dx
= x ln x - x + c.

Exemplo 1.34. Calcular a integral arcsen x dx.


Soluo. Fazendo
u = arcsen x

du =

1
1 - x2

dv = dx , obtemos

dx e

v = x.

Aplicando a frmula de integrao por partes, temos:


x
arcsen x dx = x arcsen x - 1 - x 2 dx.
x
Para calcular a integral
faremos a mudana de varivel,
1- x 2
t = 1 - x 2 e obtemos dt = -2 x dx.

58
Assim,
x

1- x

=-

1 1
1 dt
2 t2
1

1 t2
=+ c1
2 1
2

Portanto,

= - 1 - x 2 + c1 .

arcsen x dx = x arcsen x +
Exemplo 1.35. Calcular a integral

1 - x 2 + c, onde (c = -c1 ).

cos 2 x dx .

Soluo. Fazendo
u = x 2 e dv = cos 2 x dx obtemos
1
du = 2 x dx e v = cos 2 x dx = sen2 x.
2
Assim,
2
x cos 2 x dx =

x2
sen 2 x - x sen 2 x dx .
2

Para calcular a x sen 2 x dx devemos aplicar novamente o mtodo


da integrao por partes.
Fazendo
e dv = sen 2 x dx obtemos
1
du = dx e v = - cos 2 x.
2
Segue que,
u=x

x sen 2 x dx = - 2 cos 2 x + 2 cos 2 x dx


Portanto,

x
1
= - cos 2 x + sen 2 x + c1.
2
4

x2
x
1
x cos 2 x dx = 2 sen 2 x + 2 cos 2 x - 4 sen 2 x + c.
2

59
Exemplo 1.36. Calcular a integral e3 x cos x dx.
Soluo. Fazendo
u = e3 x

dv = cos x dx temos

du = 3e3 x dx e

v = sen x .

Assim,

3x

cos x dx = e3 x sen x - 3 e3 x sen x dx.

Para calcular a integral e3 x sen x dx aplicamos novamente a integrao por partes. Fazendo
u = e3 x

dv = sen x dx temos

du = 3e3 x dx

v = - cos x.

Segue que

3x

sen x dx = -e3 x cos x + 3 e3 x cos x dx.

3x

cos x dx = e3 x sen x + 3e3 x cos x - 9 e3 x cos x dx.

Logo,

Note que a integral do segundo membro igual integral que queremos calcular.
Chamando I = e3 x cos x dx podemos escrever
I = e3 x sen x + 3e3 x cos x - 9 I,
ou seja,
I=

1 3x
[e sen x + 3e3 x cos x].
10

Portanto,
3x
e cos x dx =

e3 x
[sen x + 3cos x] + c.
10

Podemos calcular integrais definidas usando integrao por partes.


Combinando a frmula de integrao por partes com o Teorema
Fundamental do Clculo, obtemos

b b
f ( x) g '( x) dx = f ( x) g ( x) - f '( x) g ( x) dx ,
a a

que a frmula de integrao por partes para integral definida.

60

Exemplo 1.37. Avalie a integral

x ln x dx.

Soluo. Fazendo
u = f ( x) = ln x
du = f '( x) dx =

1
dx
x

dv = g '( x) dx = x dx obtemos

e
e

v = g ( x) =

x2
.
2

Usando a frmula da integrao por partes para integral definida,


temos
2
2 2x
x2
ln
ln x - dx
x
x
dx
=
1
1 1 2
2
22
x2 2
12
= ln 2 - ln1 2
2
4 1
22 1
= 2 ln 2 - -
4 4
3
= 2 ln 2 - .
4

1.7.2 Exerccios
1) Calcule as integrais indefinidas:
a) x 2 e x dx ;
b) ( x - 3) sec 2 x dx;
c) x 4 ln x dx;
d) arc tg x dx;
e) ln ( x 2 + 1) dx;
Sugesto. Aplicar o mtodo da integrao por partes e esx2
1
= 1- 2
crever 2
.
x +1
x +1
f) sec3 x dx ;
Sugesto. Escreva sec3 x = sec x sec 2 x e use sec 2 x = tg 2 x + 1.

61

2) Calcular as integrais definidas:


2

a) x e x dx;
1

b) ( x + 1) cos 2 x dx;
0

c)

d)

sen 3 x dx;

ln x
dx.
x2

3) Use o mtodo da integrao por partes para mostrar que

sen

1
n -1
x dx = - sen n -1 x cos x +
sen n - 2 x dx.

n
n

Sugesto. Expresse sen n x = sen x sen n -1 x.


4) Suponha que f '' contnua em [a, b]. Mostre que
b

f (b) = f (a ) + f '(a )(b - a ) + (b - x) f "( x) dx.


a

1.8 Clculo de reas


Vimos na Seo 1.2 que, se f :[a, b] uma funo contnua tal
que f ( x) 0 para todo x [a, b], a rea da regio plana S limitada
pelas retas x = a , x = b e y = 0, e pelo grfico de f definida por
b

rea S = f ( x) dx.
a

y = f ( x)
S
a

b
Figura 1.12

Podemos estender o clculo de rea para uma classe mais ampla de


regies planas. Vamos assumir que as funes envolvidas so funes contnuas em [a, b]. Vejamos:
1) Se f ( x) 0 para todo x [a, b], ento - f ( x) 0 e, assim,
b

rea S = - f ( x) dx .
a

62

b
S
y = f (x)

S = {( x, y ) 2 ; a x b e f ( x) y 0}
Figura 1.13

Observao. Se a regio S descrita como na Figura 1.14, ento


c

rea S = f ( x) dx - f ( x) dx + f ( x) dx .

S o conjunto hachurado
Figura 1.14

2) Se a regio plana est entre os grficos de duas funes y = f ( x)


e y = g ( x), e as retas x = a e x = b, com f ( x) g ( x) para todo
x [a, b], ento:
b

rea S = f ( x) dx - g ( x) dx.

y
y = f ( x)
S
y = g (x )
a

S = {( x, y ) 2 ; a x b e g ( x) y f ( x)}
Figura 1.15

63

A Figura 1.15 ilustra o caso onde S a regio limitada pelas retas


x = a e x = b, e pelos grficos das funes f e g com f ( x) g ( x) 0
para todo x [a, b]. Note que f ( x) - g ( x) 0 para todo x [a, b]. Se
for mantido f ( x) g ( x) para todo x [a, b], mesmo que f e g no
satisfaam a condio f ( x) 0 e/ou g ( x) 0 para todo x [a, b], a
frmula para o clculo da rea do conjunto S continua o mesmo. As
Figuras 1.16 (a) e 1.16 (b) ajudam a visualizar o clculo da rea.

S
S
a

b
e

k constante
Figura 1.16

Concluso. Se f ( x) g ( x) para todo x [a, b] , ento


b

rea S = f ( x) dx - g ( x) dx = [ f ( x) - g ( x)] dx.


Exemplo 1.38. Calcular a rea da regio S limitada pelas retas
3
x = 1, x = 2 e y = 0 e pela curva y = x + 2.
Soluo. A regio S est ilustrada na Figura 1.17.
2

rea S = ( x 3 + 2) dx
1

x4
2
= + 2x
4
1
=

23
u.a.
4

64

y = x3 + 2

2
S

2
Figura 1.17

Observao. Em alguns casos x = a e/ou x = b precisam ser determinados.


Exemplo 1.39. Encontre a rea da regio S limitada pelo eixo x e
pela parbola y = x 2 + x - 2 .
Soluo. A parbola y = x 2 + x - 2 corta o eixo dos x nos pontos
em
x = -2 e x = 1, (ver Figura 1.18). Como
[-2,1] , resulta que
1

rea S = - ( x 2 + x - 2) dx
-2

x3 x 2
1
= - + - 2x
3 2
-2
9 9
= - - = u.a.
2 2

y = x2 + x 2

Figura 1.18

65
1
Exemplo 1.40. Calcular a rea da regio S limitada pelas retas x =
2
e y = - x + 2 e pela curva x = y .
Soluo. As curvas y = - x + 2 e x =
de abscissa 1 (ver Figura 1.19).

y interceptam-se no ponto

rea S = 1 [- x + 2 - x 2 ] dx
2

x2
x3 1
= - + 2 x - 1
3 2
2
1
= u.a.
3

x=

S
y = x + 2
1

Figura 1.19

Exemplo 1.41. Encontre a rea da regio S limitada pelas retas


y = x + 2, y = 2, y = -1 e pela curva x = y 2.
Soluo. As retas y = x + 2 e y = -1 interceptam-se no ponto de
abscissa -3. As curvas y 2 = x e y = 2 interceptam-se no ponto de
abscissa 4. As curvas y 2 = x e y = -1 interceptam-se no ponto de
abscissa 1. A Figura 1.20 indica os pontos de interseo das curvas.
0

-3

rea S = [ x + 2 - (-1)] dx + (2 - x ) dx + [- x - (-1)] dx


x2
=
2

2 3
2 3
+ 3x + 2 x - x 2 - x 2 + x
3 0 3 0
-3
0
0
-3

15
u.a.
2

66

y = x+2
y2 = x

2
3 2

S
1

Figura 1.20

Outra maneira para encontrar a rea acima calcular a integral


2
rea S = [ y 2 - ( y - 2)] dy.
-1

1.8.1 Exerccios
1) Esboce a regio limitada pelas curvas dadas e encontre a rea
da regio:
3
a) y = x e y = x ;

x
b) y = 6 + x, y = x3 e y = - ;
2
1
c) y = , y = x , x = 2 e y = 0;
x
1
d) y = 2 , x = 1, x = 2 e y = 0;
x
e) y = cos x, y = sen 2 x e x = 0 no primeiro quadrante;
f) y = x - 1 e y 2 = 2 x + 6.
2) Considere a regio descrita pelo conjunto S dado e calcule sua
rea.
a) S = {( x, y ) 2 ; 0 x 1, x 2 y x };
b) S = {( x, y ) 2 ; 1 x 2, x y x 2 };
c) S = {( x, y ) 2 ; x 0 , x 3 - x y - x 2 + 5 x}.

67

1.9 Integrais imprprias

Na definio da integral

f ( x) dx, assumimos f uma funo li-

mitada no intervalo fechado e limitado [a, b]. Agora, vamos estender o conceito de integral para os demais tipos de intervalos. Isto ,
para intervalos da forma [a, b), (a, b], (a, b), [a, +), (-, b], (a, +),
(-, b) e (-, ).
Definio 1.6. (Integrais imprprias em intervalos limitados)
a) Seja f :[a, b) R uma funo integrvel em [a, t ], para todo
t

t em (a, b). Se lim- f ( x) dx existir ento este limite chamaa

t b

do integral imprpria de f no intervalo [a, b), e escrevemos

f ( x) dx = lim- f ( x) dx.
a

t b

b) Seja f : (a, b] uma funo integrvel em [t , b], para todo


t em (a, b). Definimos a integral imprpria de f no intervalo
b

(a, b] como sendo lim+ f ( x) dx, se este limite existir, e escrevet

t a

mos

f ( x) dx = lim+ f ( x) dx .
t a

c) Sejam f : (a, b) uma funo e c (a, b). Se

f ( x) dx e

f ( x) dx existem ento definimos a integral imprpria de f


b

em (a, b) por

f ( x) dx = f ( x) dx + f ( x) dx.

Observaes:
a) Com a notao do item (c) da definio acima, pode-se provar
que, se

f ( x) dx e

f ( x) dx e

f ( x) dx existem para algum, ento

f ( x) dx existem para todo . Logo, pela contrapo-

sitiva, para mostrar que

um ponto c (a, b) tal que

f ( x) dx no existe, basta encontrar

f ( x) dx ou

b) Costuma-se chamar a expresso

f ( x) dx no exista.

f ( x) dx de integral, mes-

mo que o domnio da f seja [a, b), (a, b] ou (a, b). E escrevese a integral

f ( x) dx igual ao limite correspondente, mes-

68

mo antes de calcular o limite. Deixando subentendido que a


b

integral f ( x) dx s existe quando o limite correspondente


a

existe. Por exemplo, se f est definida em [a, b) escrevemos

f ( x) dx = lim- f ( x) dx , mesmo antes de calcular o limite.


a

t b

Quando a integral

f ( x) dx existe, dizemos que essa integral con-

vergente. Caso contrrio, dizemos que divergente.

Exemplo 1.42. Determine se cada integral convergente ou divergente.


2
1
dx ;
a)
0
2- x
1
dx ;
0 x

b)

c)

1
dx, onde p > 0.
xp

Soluo.
a) A funo f ( x) =

1
contnua em [0, t ] para 0 < t < 2. Logo,
2- x

f integrvel em [0, t ] para todo t (0, 2). Da Definio 1.6, temos:

t
1
1
dx = lim-
dx
0

t
2
2- x
2- x

= lim- - 2 2 - x
t 2

t
0

= lim- (-2 2 - t + 2 2)
t 2

= 2 2.
Como lim-
t 2

vergente.

1
dx = 2 2 , a integral
2- x

1
dx con2- x

1
contnua em [t ,1] para todo 0 < t < 1. Logo, f
x
integrvel em [t ,1] para todo 0 < t < 1. Assim,

b) A funo f ( x) =

69

11
1
lim
dx
=
0 x t 0+ t x dx
1

= lim+ ln x
t 0

1
t

= lim+ [ln1 - ln t ] = +.
t 0

Como lim+ t
t 0

1
dx no existe, a integral
x

1
dx divergente.
0 x

1
contnua em [t ,1] para todo t (0,1). Note
xp
11
que, para p = 1, a integral 0 dx , que divergente (item (b)).
x

c) A funo f ( x) =

Vamos analisar a convergncia da integral para p 1. Neste caso,

1 1
1
dx = lim+ p dx
p
t 0 t x
x
1

= lim+ x - p dx
t 0

x - p +1
= lim+
t 0 - p + 1
t

1
t - p +1
= lim+
.
t 0
1- p 1- p
Se p > 1 ento - p + 1 < 0 , e, quando t 0+, temos t - p +1 + .
Segue que
1 1
lim+ p dx no existe.
t 0 t x
1 1
Logo, a integral 0 p dx com p > 1 divergente.
x
Se 0 < p < 1, ento - p + 1 > 0, e, quando t 0+, temos t - p +1 0
Logo,
lim+

t 0

1
1
dx =
, isto , a integral
p
1- p
x

1
dx
xp

convergente.
Portanto, a integral
gente para p 1.

1
dx convergente para 0 < p < 1 e diverxp

70
Exemplo 1.43. Avalie a integral

x ln x dx .

Soluo. A funo f ( x) = x ln x no est definida na origem, e


contnua em [t ,1] para todo 0 < t < 1. Assim,

x ln x dx = lim+ x ln x dx .
t 0

Para obter

x ln x dx , vamos usar integrao por partes. Fazendo

u = ln x

1
dx e
x

du =

dv = x dx , obtemos
x2
v= .
2

Temos

x2
1 1
x ln x dx = ln x - x dx
2
2 t
t
1

t2
1
= - ln t - x 2
2
4 t
=-

t2
1 t2
ln t - + .
2
4 4

Assim,
t2
1 t2
=
+
x
ln
x
dx
lim
ln
t
.
2
0
t 0+
4 4

Vamos usar a Regra de L Hospital para calcular o limite do primeiro


termo.
lim+ -

t 0

t2
- ln t
ln t = lim+
t 0
2
2
t2

1
= lim+ t
t 0 -4
t3
-

= lim+
t 0

Portanto,

t2
= 0.
4

1
x ln x dx = - .
4

71

Exemplo 1.44. Calcule a integral

-1

1 - x2

dx.

est definida em (-1,1) e cont1 - x2


nua nos intervalos [t , 0] para todo -1 < t < 0 e [0, s ] para todo 0 < s < 1.
Assim,
1
0
1
x
x
x
-1 1 - x 2 dx = -1 1 - x 2 dx + 0 1 - x 2 dx ,
Soluo. A funo f ( x) =

desde que as integrais do segundo membro sejam convergentes.


Vamos calcular

-1

Para obter

x
1 - x2

dx = lim+

1- x 2

1 - x2

t -1

dx.

dx , aplicaremos o mtodo da substituio.

Fazendo u = 1 - x 2, obtemos du = -2 xdx .


Assim,
0
x
-1 1 du
dx
=
t 1 - x 2
2 1-t 2 u
=- u

1
1-t 2

= -1 + 1 - t 2 .

Logo,

-1

dx = lim+ -1 + 1 - t 2

t -1
1- x
= -1.
2

Agora,
0

x
1- x

dx = lim-
t 1

1 - x2

= lim- - u
t 1

dx .

1-t 2
1

= lim- - 1 - t 2 + 1

t 1
= 1.

Portanto,

-1

x
1 - x2

dx = -1 + 1 = 0.

72
Exemplo 1.45. Calcule a integral

-1

1
dx .
x -1
2

1
est definida em (-1,1), e conx -1
tnua nos intervalos [t , 0] para todo -1 < t < 0 e [0, s ] para todo
0 < s < 1. Assim,

Soluo. A funo f ( x) =

0
1
1
1
1
dx = 2 dx + 2 dx ,
0 x -1
-1 x - 1
-1 x - 1

desde que as integrais do segundo membro sejam convergentes. Calcularemos


0
1
1
lim
dx
=
-1 x 2 - 1 t -1+ t x 2 - 1 dx.
0

1
pode ser escrita como
x -1

A funo f ( x) =

1 1
1 1
.
f ( x) =
-
2 x -1 2 x +1
Ento

1
1 0 1
1 0 1
=
dx
dx
dx
2 t x - 1
2 t x + 1
x2 -1
0
0
1
1
= ln x - 1 t - ln x + 1 t
2
2

1
1
= - ln t - 1 + ln t + 1 .
2
2

Segue que

1
1
1

dx = lim+ - ln t - 1 + ln t + 1
-1 x - 1
t -1 2
2

= -.
Como a integral
divergente.

1
dx divergente, conclumos que
-1 x - 1

1
dx
-1 x - 1

Se a integral f ( x) dx existir e f no negativa no seu domnio [a, b),


a
(a, b] ou (a, b), ento esta integral pode ser interpretada como a rea
da regio plana S sob grfico de f e acima do eixo x, e entre retas
x = a e x = b (ver Figura 1.21).

73
y

S
a

b x

Figura 1.21

Definio 1.7. (Integrais imprprias em intervalos ilimitados)


a) Seja f :[a, +) R uma funo integrvel em [a, t ], para todo
t

t > a. Se lim f ( x) dx existir, ento este limite chamado de


t + a

integral imprpria de f no intervalo [a, +). Neste caso usamos a notao

f ( x) dx = lim f ( x) dx .
t + a

Analogamente, se f : (-, b] R uma funo integrvel em


[t , b] para todo t < b, definimos a integral imprpria de f em
(-, b] por

f ( x) dx = lim f ( x) dx, quando o limite existir.


t - t

b) Seja f : (a, +) R uma funo. Se para algum c (a, +)


existirem

f ( x) dx e

pria de f em (a, +) por

f ( x) dx, definimos a integral impr-

f ( x) dx = f ( x) dx +

f ( x) dx.

De forma anloga, definem-se as integrais imprprias para


funes definidas em intervalos da forma (-, b) e (-, +).
Observaes:
a) Costuma-se chamar as expresses

-
b
-

f ( x) dx ,

f ( x) dx de integrais. E escreve-se a integral


f ( x) dx ou

f ( x) dx e

f ( x) dx,

f ( x) dx igual ao limite correspondente, mes-

mo antes de calcular o limite. Deixando subentendido que a


integral s existe quando o limite existe.

74
b) Com a notao do item (b) da definio acima pode-se provar
que, se
ento

f ( x) dx e

f ( x) dx e

c
+

f ( x) dx existem para algum c (a, +),


f ( x) dx existem para todo (a, +). O

mesmo vale para integrais definidas em (-, b) e (-, +).

Quando a integral

f ( x) dx existe dizemos que esta integral

convergente. Caso contrrio, dizemos que divergente. O mesmo


vale para as integrais

f ( x) dx e

f ( x) dx.

Exemplo 1.46. Determine se cada integral convergente ou divergente.


+

a) e - x dx;
0

b)

c)

1
dx;
x
1
dx, onde p > 0.
xp

Soluo.
a) Da Definio 1.7, temos

e - x dx = lim e - x dx
t + 0

= lim - e - x
t +

t
0

= lim (-e - t + e0 )
t +

1
= lim - t + 1
t +
e

= 1.
t

Como lim e - x dx = 1, a integral


t + 0

b)

t 1
1
dx = lim dx
t + 1 x
x

= lim ln x
t +

t
1

e - x dx convergente.

75

= lim [ln t - ln1]


t +

= lim ln t = + .
t +

Como o limite no existe, a integral

1
dx divergente.
x

+ 1
c) Para analisar a convergncia da integral
dx, estudaremos os
1
xp
casos em que p = 1 e p 1.
+ 1
dx divergente (item (b)).
Se p = 1 ento a integral 1
x
Para p 1 , temos

t
1
dx = lim x - p dx
p
t + 1
x
t

x - p +1
= lim
t + - p + 1
1

t1- p
1
= lim
.
t + 1 - p
1
p

Se p > 1, ento 1 - p < 0, e quando t + temos t1- p 0. Logo,


t 1
1
, isto ,
lim p dx =
t + 1 x
p -1
a integral

1
dx com p > 1 convergente.
xp

Se 0 < p < 1 ento 1 - p > 0 , e quando t + temos t1- p +.


Neste caso,
t

lim x - p dx no existe,

t + 1

1
dx com p < 1 divergente.
xp
+ 1
Portanto, a integral
dx convergente para p > 1 e diver1
xp
gente para 0 < p 1.

e assim a integral

Exemplo 1.47. Calcule

e - x sen x dx .

Soluo. Da Definio 1.7, temos

e - x sen x dx = lim e - x sen x dx .


t + 0

76
Para calcular a integral
partes. Fazendo

e
0

-x

sen x dx aplicaremos a integrao por

u = e- x

dv = sen x dx, obtemos

du = -e - x dx

v = - cos x.

Assim,

e - x sen x dx = -e - x cos x - e - x cos x dx .

Para calcular

-x

cos x dx , aplicamos novamente a integrao por

partes. Fazendo
u = e- x

dv = cos x dx, obtemos

du = -e - x dx

v = sen x.

Logo,
t
-x

t
-x
-t
=
+
+ e - x sen x dx
e
sen
x
dx
(
e
cos
t
cos
0)
e
sen
x

0
0

0
ou seja,
t
1
-x
-t
-t
0 e sen x dx = 2 [-e cos t + 1 - e sen t ] .
Portanto,
t

1 cos t
sen t
e - x sen x dx = lim - t + 1 - t
t + 2
e
e
1
= . (Note que cos t e sen t so limitadas e
2
1
0 quando t + )
et

Exemplo 1.48. Calcule

1
dx.
x +9
2

Soluo. Podemos escrever

0
+
1
1
1
dx = 2
dx +
dx ,
2
- x + 9
0
x +9
x +9
2

desde que ambas as integrais do segundo membro sejam convergentes. Calculando,

t
1
1
lim
=
dx
2
2

0
t
+
x +9
x +9
t

1
x
= lim arc tg
t + 3
30

(Seo 1.7.2 Exerccio 4)

Teorema (Clculo I).


Sejam f , g :[a, b]
funes, e x0 (a, b) .
Se

g limitada, ento
lim f ( x) g ( x) = 0.

x x0

77
t 1
1

= lim arc tg - arc tg 0


t + 3
3 3

1
t
= lim arc tg
t + 3
3

= .
6
De forma anloga, mostra-se que

= .
x +9 6
2

Como ambas as integrais so convergentes, a integral dada conver+


1

gente e
dx = .
- x 2 + 9
3
Observao. Qualquer uma das integrais imprprias definidas acima pode ser interpretada como uma rea, desde que a funo seja no negativa e a integral convergente. Por exemplo, se f ( x) 0 para todo x a e a integral
+
f ( x)dx existe, ento definimos a rea da regio corresa

pondente
rea S =

S = {( x, y ) R 2 ; x a, 0 y f ( x)}
f ( x)dx .

S
a

x
Figura 1.22

Exemplo 1.49. Esboce a regio correspondente


S = {( x, y ) R 2 ; x 1, 0 y e x }
e determine a rea de S .
Soluo. A regio S est ilustrada na Figura 1.23.
1

rea S = e x dx
-

= lim e x dx
t - t

como

sendo

78

= lim e x
t -

1
t

= lim (e - et )
t -

= e u.a .

y = ex

1
1

Figura 1.23

Em algumas situaes, estamos interessados somente em saber se a


integral convergente ou divergente. O prximo resultado (Critrio
da Comparao) permite afirmar a convergncia ou divergncia de
uma integral comparando-a com outra. A demonstrao ser omitida.
Teorema 1.13. (Critrio da Comparao) Sejam f , g : I R funes
tais que 0 f ( x) g ( x) para todo x I , onde I um intervalo da
forma [a, b), (a, b], (a, b), [a, +), (a, +), (-, b], (-, b) ou (-, +).
a) Se a integral de g em I convergente, ento a integral de f
em I convergente;
b) Se a integral de f em I divergente, ento a integral de g em
I divergente.
Note que o item b) no teorema acima a contrapositiva do item a).
Observaes:
a) O Critrio da Comparao pode ser aplicado quando f ( x) e
g ( x) so ambas no positivas. Sejam f e g funes tais que
f ( x) g ( x) 0 para todo x I ( I um dos intervalos mostrados anteriormente).

79
i) Se a integral de f em I converge, ento a integral de g em
I converge.
ii) Se a integral de g em I diverge, ento a integral de f em
I diverge.
b) No Critrio da Comparao, a hiptese de f ( x) e g ( x) serem ambas no negativas (ou no positivas) essencial. Se
esta hiptese removida, podemos ter problema. Vejamos:
-1
Considere as funes f , g : (0,1] R definidas por f ( x) =
e
x
g ( x) = 1. Temos g ( x) f ( x), pois f ( x) 0 para todo x (0,1] .
Calculando

g ( x) dx = dx = lim+ dx = lim+ x t = lim(1


- t) = 1
+
0

t 0

t 0

t 0

11
1
-1
dx = lim+ - dx = lim+ - ln x t = lim+ - (ln1 - ln t ) = -.
0 x
t x
t 0
t 0
t 0

f ( x) dx =

11
1
-1
dx = lim+ - dx = lim+ - ln x t = lim+ - (ln1 - ln t ) = -.
0 x
t x
t 0
t 0
t 0
Logo,

f ( x) dx =

g ( x) dx converge, mas

f ( x) dx diverge.

Exemplo 1.50. Verifique que a integral imprpria


convergente.

x
dx
x +1
4

Soluo. Temos que


x 4 < x 4 + 1 para x 1.
Segue que
1
1
< 4
4
x +1 x

para x 1.

Multiplicado por x a desigualdade acima temos,


1
x
0< 4
< 3
para x 1.
x +1 x
+ 1
Como
dx convergente (Exemplo 1.46), segue pelo Critrio da
1
x3
+
x
Comparao que a integral
dx convergente.
4
1
x +1

80

Exemplo 1.51. Mostre que a integral

1 + e- x
dx divergente.
x

Soluo. Para x 1 temos


1 + e- x 1
> > 0.
1 + e- x > 1 e
x
x
+ 1
Como
dx divergente, segue pelo Critrio da Comparao que
1
x
-x
+ 1 + e
a integral
dx divergente.
1
x
Exemplo 1.52. Verifique que

cos 2 x
dx convergente.
x2 + 9

Soluo. Para x 0 temos cos 2 x 1 . Podemos escrever


0

cos 2 x
1
para todo x 0.
2
2
x +9 x +9

1
dx convergente (Exemplo 1.48) segue, pelo Critrio
0 x +9
2
+ cos x
da Comparao, que a integral
dx convergente.
0
x2 + 9

Como

Exemplo 1.53. Aplique o Critrio da Comparao para verificar que


x
1 e
0 x dx convergente.
Soluo. Para 0 < x 1 temos 0 < e x e . Segue que
ex
e
0<

para 0 < x 1.
x
x
1 e
1 1
dx tambm condx convergente, temos que
Como
0
0
x
x
x
1 e
dx convergente, pelo critrio da comparao.
vergente. Logo,
0
x

1.9.1 Exerccios
1) Verifique se a integral convergente ou divergente e avalie
aquelas que so convergentes.
a)

x
dx ;
2
x +1

e)

x e - x dx ;

81

2
dt ;
- 1 + 4t 2

b)

ln z dz;

g)

dx
, ( a > 0 );
x + a2

h)

c)

( x - 2)

d)

dx ;

f)

1
dx;
x + 5x + 6
2

e - x cos 2 x dx .

2) Use o critrio da comparao para determinar se a integral


convergente.
a)

b) 0
c)

dx
;
3
x (1 + e x )

d)

e- x
dx;
x

e)

1
dx ;
x -x

f)

sen 2 x
dx ;
x2 + 1
1
x5 + 1

dx;

1+ x
dx .
x

3) Se f (t ) contnua para t 0, a transformada de Laplace de f


+
a funo F definida por F ( s ) = f (t ) e - st dt , e o domnio de
0
F o conjunto de todos os nmeros s para os quais a integral
converge. Calcule a transformada de Laplace das seguintes
funes:
a) f (t ) = 1;

c) f (t ) = e at;

b) f (t ) = cos t ;

d) f (t ) = t .

1.10 Utilizao de pacotes


computacionais
Os computadores surgiram como ferramentas para facilitar a realizao de grandes quantidades de operaes de forma automtica.
Atualmente, essa no a nica de suas utilidades.

82

A manipulao de expresses simblicas um dos mais recentes


usos dos computadores, produzindo resultados algbricos para problemas algbricos.
Nesse contexto, o pacote Gnu/Maxima um timo representante
podendo ser utilizado para a realizao de clculos ntegro-diferenciais, bastante teis para auxiliar o aluno que est estudando disciplinas de Clculo. Alm disso, trata-se de um software livre (gratuito) que pode ser encontrado em http://maxima.sourceforge.net/.
A seguir, apresentaremos alguns exemplos das potencialidades do
Maxima no clculo integral e diferencial.
5

Exemplo 1.54. Calcular (3 x + 4) dx .


1

Soluo. Podemos resolver esta integral atravs


do clculo da rea delimitada pelas curvas dadas por f ( x) = 3 x + 4, x = 1, x = 5 e pelo eixo-x.
Para facilitar, podemos desenhar o grfico da
respectiva regio (Figura 1.24). Para isso, podemos usar o seguinte comando do pacote computacional Maxima:
plot2d(3*x + 4, [x, 1, 5])

20
18
16
14
12
10
8
6
4
2

onde o primeiro argumento representa a funo,


e o segundo argumento se divide em trs partes:
a varivel independente e os extremos inferior
e superior do intervalo onde a funo deve ser
representada.

B
b
0,5 1 1,5 2 2,5 3 3,5 4 4,5 5

Figura 1.24: Esboo do grfico de f ( x) = 3 x + 4 e determinao de rea no intervalo [1,5].

Sabemos que a regio hachurada no grfico um trapzio cuja rea


dada por
( B + b).a (19 + 7).4
=
= 52 u.a.
2
2
Tal integral definida tambm pode ser calculada atravs do comando

integrate(3*x + 4, x,1, 5)

83

onde o primeiro argumento representa a funo a ser integrada, o


segundo representa a varivel de integrao e os dois ltimos representam os extremos inferior e superior do intervalo de integrao.
Exemplo 1.55. Encontrar a primitiva das integrais abaixo utilizando
o Maxima:
a) F ( x) = 2 x dx;

b) F ( x) = 3e3 x dx;

c) F ( x) = sen( x) dx;
d) F ( x) = x 4 dx.

Soluo. Em todos os itens, pode-se utilizar o comando


integrate(f(x), x).
Repare que agora ele s tem dois argumentos: o primeiro que deve
ser trocado pela respectiva funo a ser integrada e o segundo que
representa a varivel de integrao. Em
a) temos o comando integrate(2x,x);
b) integrate(3*exp(3*x),x);
c) integrate(sin(x),x) e em
d) integrate(x^4,x).
Ao realizar esses testes, note que aos resultados podem ser adicionadas constantes de integrao para representar toda a famlia de
primitivas.
Exemplo 1.56. Calcular a rea definida pelo grfico da curva
f ( x) =

x
x2 +1

, em [0,1].

Soluo. O grfico da respectiva regio (Figura 1.25) pode ser obtido


atravs do comando plot2d(x/(x^2+1)^(1/2), [x, 0, 1]).

84
y
0,7
0,6
0,5
0,4
0,3
0,2
0,1
0

0 0,1 0,2 0,3 0,4 0,5 0,6 0,7 0,8 0,9 1

Figura 1.25: Esboo do grfico de f ( x) =

x
2

x +1

determinao de rea no intervalo [0,1].

A rea definida pelo grfico da funo f , no intervalo especificado,


pode ser calculada atravs da integral
1

x
2

x +1

dx

que o software Maxima pode calcular usando o comando


integrate(x/(x^2+1)^(1/2), x,0, 1),
resultando 2 - 1 u.a. Para obtermos somente a primitiva
podemos usar o comando

x2 + 1 ,

integrate((x^2+1)^(1/2) , x).
Exemplo 1.57. Calcular a rea delimitada pelo grfico das curvas
y = x 2 e y = 2 x - x 2.
Soluo. Para esboar os grficos das duas funes em um mesmo
sistema de eixos coordenados, entre x = -0, 5 e x = 1,5 (Figura 1.26),
utilizamos o comando
plot2d([x^2, 2*x-x^2] , [x, -0.5, 1.5])
onde o primeiro argumento se divide em duas partes representando
cada uma das funes, e o segundo argumento se divide em trs
partes: a varivel independente e os extremos inferior e superior do
intervalo onde a funo deve ser representada.

85
y
1
0,5
0
0,5
1
0

0,5

0,5

1,5

2
2
Figura 1.26: Esboo dos grficos de y1 = x e y2 = 2 x - x
e determinao de rea em [0,1] .

As abscissas dos pontos onde as duas curvas se interceptam so 0 e


1 . Assim, a rea definida pelo grfico pode ser calculada atravs da
1

integral (2 x - x 2 ) - x 2 dx , usando
0

integrate(2*x, x,0, 1),

1
que resulta em u.a .
3
0

Exemplo 1.58. Calcular

-x

dx.

Soluo. Neste caso, trata-se de uma integral imprpria:


0

-x

e - x dx = lim F (b).
dx = blim
-
b -
b

Vamos calcular a primitiva utilizando o comando


integrate(exp(-x), x,0,b)
que resulta em F (b) = -e - b + 1. Em seguida calculamos o respectivo
limite no infinito atravs de
limit (-exp(-b)+1, b, inf)
onde o primeiro argumento representa a funo F , o segundo representa a varivel independente b e o ltimo representa para onde
tende tal varivel. Nesse caso, obtemos o resultado 1.

86
0

Exemplo 1.59. Calcular

-x

dx.

Soluo. Agora, temos que calcular lim e - x dx = lim F (b).


b -
b -
b
O comando
integrate(exp(-x), x,b,0)
resulta em F (b) = e - b - 1 enquanto o limite calculado por
limit (exp(-b)-1, b, -inf)
resulta + , indicado pelo software por inf, mostrando que a integral
divergente.

1.10.1 Exerccios
1) Calcule a integral

e - x dx .

2) Mostre que se f contnua em [-1, 3], ento

-1

-1

f ( x) dx + f ( x) dx + f ( x) dx + f ( x) dx = 0.
1

3) Considere a funo f :[-1,1] definida por


1

sen , se x 0
.
f ( x) =
x
0, se x = 0

A funo f integrvel em [-1,1]? Justifique.


4) Considere as integrais

f (t ) dt = 3,

Encontre

f ( s ) ds = 8 e

f (u ) du = -1.

f ( x) dx .

5) Se f contnua em [a, b], mostre que

f ( x) dx

f ( x) dx .

Sugesto. Use - | f ( x) | f ( x) | f ( x) | e Teorema 1.8.


1 1
6) Encontre uma funo f tal que f ( x) - - 2 = 0 e f (1) = 1.
x x

87
7) Seja f funo contnua em [-a, a ] . Mostre que:
a) Se f funo par, ento

-a

b) Se f funo mpar, ento

f ( x) dx = 2 f ( x) dx.

-a

f ( x) dx = 0.

8) Use o Exerccio 7 acima para mostrar que, se f :[0,1] uma


1
funo contnua, ento x f ( x 2 ) dx = 0.
-1

9) Mostre que se g :[-1,1] uma funo contnua, ento

g (sen x) cos x dx = 0.
x3

10) Encontre a derivada da funo G ( x) = cos t dt .


2

Sugesto. Faa u = x3 e use a Regra de Cadeia.


11) Verifique por diferenciao que a frmula est correta.
x-2
x-2
6
dx = 2 x - 2 + c;
arc tg
x +1
3
3

a)

b)

c)

dx
1
x+a
=
ln
+ c;
2
-x
2a x - a

x2
x2 + 4

dx =

1
2
x x 2 + 4 - ln
6
3

x2 + 4 + x
+ c.
2

12) Calcule as integrais indefinidas:


a)

2
( x + sen x) dx ;

x4 - 3 3 x + 2
dx ;

x
x2 + 1
c) 2 dx ;
x
x2
d) 2 dx ,
x +1

b)

ecos x
dx
g)
cossec x ;
h)

cos x sen (sen x) dx ;

( x - 1)
dx;
- 2x + 5
dx
j) 2
,
x - 2x + 5

i)

Sugesto. Dividir x 2 por ( x 2 + 1) ; Sugesto. Completar o quadrado;


2 - sen x

e)

2 x + cos x dx;

f)

tg x dx;

k)

( x + sec

5 x) dx ;

88

l)

cos x dx,
2

Sugesto. Use cos 2 x =


m)

log x dx ;

n)

1 + cos 2 x
.
2

sen 2 x
dx ;
cos x

o)

p)

cos x dx ;

q)

e x dx ;

r)

2x

sen 3 x dx ;

ln x dx.

13) Sabendo que f (0) = g (0) = 0 , mostre que

f ( x) g "( x) dx = f (a ) g '(a ) - f '(a ) g (a ) + f "( x ) g ( x ) dx.


0

14) Considere a funo


2

x + 1, se | x | 1
.
f ( x) = x
<

2
,
se
1
x
2

Determine

-1

f ( x) dx.

15) Calcule as integrais definidas:


ln 3 x
1 x dx ;
1
1
b)
dt ;
0 1+ t2
2

a)

c)
d)

ln x dx;

e)

f)

g)

( x - x 2 ) dx ;

h)

-2

1
2

cos x sen 5 x dx;

| x 2 - 1| dx;
arcsen x dx;

(sen x + | cos x |) dx .

16) Use integrao por partes para mostrar as frmulas de reduo.


1
n -1
cos n-2 x dx.
a) cos n x dx = cos n-1 x sen x +
n
n

b)

(ln x) dx = x(ln x)
n

- n (ln x) n -1 dx.

17) Calcule a rea entre as curvas y = x 2 e y = x 3.

89

18) Calcule a rea entre a curva y = ( x + 1)( x - 1)( x + 2) e o eixo dos x.


19) Calcule a rea entre as curvas x = y + 1 e x =

y2
- 3.
2

20) Esboce a regio limitada pelas curvas dadas e encontre a rea


da regio.
a) y = x + 6, y = x 3 e y = - x ;
b) x = y 2 - 2, x = e y, y = 1 e y = -1.
21) Determine a rea da regio limitada pelos grficos de y = sec x,

e x= .
y = x,
4
22) Verifique se a integral convergente ou divergente, e avalie
aquelas que so convergentes.
a)

b)

x sen x dx;
dx
;
x + 2x + 2
2

1
dx ;
0 x - 2x
+
1
dx .
d)
3
x(ln x) 4

c)

23) Use o critrio da comparao para determinar se a integral


convergente.
a)

| sen x |
dx;
x3

24) a) Mostre que

b)

1
dx;
2
x + 10

c)

1 + e- x
dx.
x

x dx divergente.
+t

b) Verifique que lim x dx = 0 . Note que


t + - t

x dx lim x dx.
t + - t

25) Seja f :[a, b] funo contnua. Mostre que existe c (a, b)


tal que

f ( x) dx = f (c)(b - a ).

Esse resultado chamado de Teorema do Valor Mdio para


Integrais.
Sugesto. Aplicar o Teorema de Weierstrass e o Teorema do
Valor Intermedirio.

90

Resumo
Os principais assuntos estudados neste captulo foram:
A definio de integral e suas propriedades;
O Teorema Fundamental do Clculo;
Mtodo da Substituio;
Mtodo da Integrao por Partes;
Aplicao da integral definida no clculo de rea;
As definies de Integrais Imprprias. importante saber calcular limite de funes.

Respostas dos exerccios


1.2.1 Exerccios
1) a) max A = sup A = 5; min A = inf A = -2;
1
b) min B = inf B = ; sup B = 1.
2
2) 2; 1.
3)

57
;
60

77
.
60

4) Sim,

limitada em [0, 4] e descontnua somente no ponto 2.

5) .

1.3.1 Exerccios
1) 34;

22;

28 .

2)

1.4.1 Exerccios
1) 0.
2) a)

10

f ( x) dx ; b)

f ( x) dx .

91

3) -10.

1.5.1 Exerccios
1) a)

3 2
(e - 1) ;
2

c)

b) 0;

d)

d)

1 3
x + c;
6

2) a)

b) - ln x 2;
c) -e - x cos e - x.
3)

1.6.1 Exerccios
1) a) x

t2
b)
2

x5
+ + x + c ;
5
9
- + c;
t

c) -e - x + c;
3) 3cos x +
4) a)

e) sec x + c ;
f)

1
2
+ 7.
x x

3e 4 + 13
;
6

b) 3 +

2
.
4

1.7.1 Exerccios
1
1) a) 3 (3 x - 1) 4 + c ;
4
1
b) sen (5 x + 2) + c ;
5

c)

x 2 + 4 + c ;

2) a)

b) -

3
3
;
+
ln 6 ln 3

d)

ln 2 x
+ c;
2

e) ln | sen x | +c;

1
x -1
arc tg
+ c;
2
2
1
c) [e12 - e3 ] ;
6
f)

d)

39 - 1
.
ln 3

92

3) ln | sec x + tg x | + c.

1.7.2 Exerccios
1) a) e x ( x 2 - 2 x + 2) + c ;
b) ( x - 3) tg x + ln | cos x | +c ;
c)

x5
1
ln x - + c ;

5
5

2) a) e 2;
1
b) - ;
2

1
d) x arc tg x - ln (1 + x 2 ) + c ;
2
2
e) x ln ( x + 1) - 2 x + 2 arc tg x + c ;

f)

1
[sec x tg x + ln | sec x + tg x |] + c .
2

c)

d)

1 1
- ln 2.
2 2

1.8.1 Exerccios
1) a)

1
u.a.
2

d)

1
u.a.
2

b)

e)

1
u.a.
4

c) ln 2 + u.a.
2

f) 18 u.a.

2) a)
b)
c)

16
u.a.
3

1.9.1 Exerccios
1) a) Diverge;

e) 0;

b) ;

f) -1;

c) 1;

d) ;
2a

2
g) - ln ;
3
1
h) .
5

2) a) Converge;

b) Converge;

93

c) Diverge;

d) Converge;

e) Converge;

f) Diverge.

3) a)

1
, s > 0;
s

b)

s
, s > 0;
s +1

c)

1
, s > a;
s-a

d)

1
, s > 0.
s2

1.10.1 Exerccios
1) 0.
3) Sim.

limitada e possui em nico ponto de descontinuidade.

4) 6.
1
6) f ( x) = ln x - + 2 .
x

10) 3 x 2 cos x 3.
x3
- cos x + c;
3
x4
b)
- 9 3 x + 2ln | x | +c;
4
1
c) x - + c;
x

1
x -1
arc tg
+ c;
2
2
x2 1
k)
+ tg 5 x + c;
2 5
1
1
l) x + sen 2 x + c;
2
4

d)

m) x log x - x log e + c;

12) a)

e)

j)

n) -2cos x + c;
e2 x
(2sen 3 x - 3cos 3 x) + c ;
13

f) ln | sen x | +c ;

o)

g) -ecos x + c;

p) x 2 sen x + 2 x cos x - 2sen x + c;

h) - cos (sen x) + c ;

q) x 2e x - 2 xe x + 2e x + c;

94

1
i) ln x 2 - 2 x + 5 + c;
2

14)

x3
x3
r)
ln - + c;
3
9

8 2
.
+
3 ln 2

15) a)

b)

;
4

c) 2ln 2 - 1 ;
d) ;

1
;
384
28
f)
;
3

3
g)
+
-1;
12 2

e)

h) 4.

17)
18)
19) 18 u.a.
20) a) 19

10

b) e - e -1 + u.a.
3

2 +1
21) ln
u.a.
2
1

22) a) Diverge;
b) Converge para ;
23) a) Converge ;
b) Converge ;
c) Diverge.

c) Diverge;
d) Converge para

1
.
3ln 3 3

Captulo 2
Mtodos de Integrao

Captulo 2
Mtodos de Integrao
Neste captulo, estudaremos mtodos para calcular integrais indefinidas cujos integrandos envolvem:
Funes trigonomtricas;
Funes com expresses da forma a 2 - x 2 ,
ou x 2 - a 2 , onde a > 0 ;

x2 + a2

Funes racionais o mtodo das fraes parciais, e


Funes racionais de seno e cosseno.
Para facilitar a leitura do texto, no final deste captulo
inclumos tabelas de derivadas, integrais e identidades
trigonomtricas.

2.1 Integrais envolvendo funes


trigonomtricas
2.1.1 Funes trigonomtricas
Vamos comear relembrando que as integrais

sen x dx = - cos x + c e cos x dx = sen x + c so imediatas.


A partir dessas integrais e da aplicao do mtodo da substituio, podemos obter as integrais de outras funes trigonomtricas.
Exemplo 2.1. Use o mtodo da substituio para mostrar que:
a) tg x dx = ln | sec x | +c ;
b) sec x dx = ln | sec x + tg x | +c .

98
Soluo.
a)

sen x

tg x dx = cos x dx .
Aplicando o mtodo da substituio, fazemos
u = cos x . Ento du = - sen xdx .
Assim,
du
u
= - ln | u | +c

tg x dx = -

= - ln | cos x | +c
= ln | sec x | + c .
b) Para calcular a integral sec x dx , escreveremos
sec x = sec x

(sec x + tg x)
(sec x + tg x)

e aplicaremos o mtodo da substituio. Temos,


(sec x + tg x)

sec x dx = sec x (sec x + tg x) dx


=

sec 2 x + sec x tg x
dx .
sec x + tg x

Fazendo u = sec x + tg x ento du = (sec x tg x + sec 2 x) dx .


Assim,
du
sec x dx = u
= ln | u | + c
= ln | sec x + tg x | +c .
De modo semelhante ao Exemplo 2.1, calculamos as integrais

cotg x dx = ln | sen x | +c

cossec x dx = ln | cossec x - cotg x | +c .

Exemplo 2.2. Calcule as integrais.


a) x tg x 2 dx ;
3x + 1
b) sec
dx .
2

99
Soluo.
1
du = x dx . Assim,
2
1
2
x tg x dx = 2 tg u du
1
= ln | sec x 2 | +c .
2

2
a) Fazendo u = x temos

2
3x + 1
b) Fazendo u =
temos dx = du . Ento
3
2
2
3x + 1
sec 2 dx = 3 sec u du
2
= ln | sec u + tg u | +c
3
2
3x + 1
3x + 1
= ln sec
+ tg
+c.
3
2
2

2.1.2 Integrais envolvendo potncias


de senx e cosx
Para calcular integrais da forma sen n x dx e cos n x dx , onde n 2
um nmero natural, usaremos as identidades trigonomtricas:

sen
2 x + cos 2 x = 1

sen 2 x =

1 - cos 2 x

2

(2)

cos 2 x =

1 + cos 2 x
,
2

(3)

(1)

e aplicaremos o mtodo da substituio.


Outro processo para calcular as integrais sen n x dx e cos n x dx utiliza frmulas de recorrncia, que so obtidas usando o mtodo da
integrao por partes (veja Exerccio 3 da Seo 1.7.4). Vamos ilustrar essa tcnica considerando exemplos que envolvem potncias
mpar e par.
Exemplo 2.3. Calcular as integrais abaixo:
a) sen 3 x dx ;

b)

cos x dx .
4

100
Soluo.
a) Temos sen 3 x = sen 2 x sen x
= (1 - cos 2 x) sen x
= sen x - cos 2 x sen x .
Assim,

sen x dx = (sen x - cos x sen x)dx


= sen x dx - cos x sen x dx .
3

A primeira integral imediata, e para calcular a segunda faremos


u = cos x e du = - sen x dx .
Portanto,

sen x dx = - cos x + u
3

du

u3
= - cos x + + c
3
cos3 x
= - cos x +
+c.
3
O mesmo raciocnio aplicado para resolver este exemplo vlido
para obter as integrais sen n x dx ou cos n x dx , quando n mpar.
Note, a ideia escrever o integrando de forma que aparea somente um fator seno (e o resto da expresso em termos de cosseno) ou
apenas um fator cosseno (e o resto da expresso em termos de seno).
b) Neste exemplo n par e no podemos aplicar a ideia acima. Vamos reescrever o integrando usando a identidade (3).
cos 4 x = (cos 2 x) 2
1 + cos 2 x
=

1 + 2 cos 2 x + cos 2 2 x
4
1 1
1
= + cos 2 x + cos 2 2 x
4 2
4
=

1 1
1 1 + cos 4 x
+ cos 2 x +

4 2
4
2

101

1 1
1 1
+ cos 2 x + + cos 4 x
4 2
8 8

3 1
1
= + cos 2 x + cos 4 x .
8 2
8
Assim,
3

cos x dx = 8 + 2 cos 2 x + 8 cos 4 x dx


4

3
1
1
= dx + cos 2 x dx + cos 4 x dx
8
2
8
3
1
1
= x + sen 2 x + sen 4 x + c .
8
4
32
O mesmo raciocnio aplicado para resolver este exemplo vlido
para calcular as integrais sen n x dx ou cos n x dx com n par. A ideia
escrever o integrando de forma que aparea potncia de ordem
2, e usar as identidades trigonomtricas que relacionam ngulo e
metade de ngulo.
Para calcular integral que envolve produto de potncias de seno e
m
n
cosseno, ou seja, integral do tipo sen x cos x dx com m e n inteiros positivos, empregamos as mesmas tcnicas usadas anteriormente. Se m e n so nmeros pares, ento recorremos s identidades (2)
e (3), e aplicamos o mtodo da substituio. No caso em que m ou n
mpar usamos a identidade (1) e aplicamos o mtodo da mudana
de varivel.
Exemplo 2.4. Calcular as integrais.
a) sen 2 x cos5 x dx ;
b)

sen

x cos 2 x dx .

Soluo.
a) Neste caso, o cosseno tem potncia mpar. Vamos preparar o integrando, mantendo um fator de cosseno, e usaremos a identidade (1).
sen 2 x cos5 x = sen 2 x cos x cos 4 x
= sen 2 x cos x (cos 2 x) 2
= sen 2 x cos x (1 - sen 2 x) 2

102

= sen 2 x cos x (1 - 2sen 2 x + sen 4 x)


= sen 2 x cos x - 2sen 4 x cos x + sen 6 x cos x .
Assim,

sen

x cos5 x dx = sen 2 x cos x dx - 2 sen 4 x cos x dx + sen 6 x cos x dx ( u = sen x )


= u 2 du - 2 u 4 du + u 6 du
1
2
1
= sen 3 x - sen 5 x + sen 7 x + c .
3
5
7

b) Neste exemplo m e n so nmeros pares. Ento vamos reescrever


o integrando usando as identidades (2) e (3).
1 - cos 2 x 1 + cos 2 x
sen 2 x cos 2 x =

2
2

1
= (1 - cos 2 2 x)
4
=

1 1 + cos 4 x
1-

4
2

1 1
= - cos 4 x .
8 8
Portanto,

sen

1
1
x cos 2 x dx = dx - cos 4 x dx
8
8
1
1
= x - sen 4 x + c .
8
32

Observao. s vezes conveniente usar a identidade sen 2 x = 2sen x cos x para calcular integral do tipo
n
m
sen x cos x dx , quando n = m e so nmeros pares.

2.1.3 Integrais de potncias de tgx e cotgx


Para calcular integrais do tipo tg n x dx e cotg n x dx , sendo n 2
um nmero natural, vamos usar as identidades

sec 2 x - tg 2 x = 1

(4)

103

cossec 2 x - cotg 2 x = 1 .

(5)

No caso do integrando ser potncia da tangente, devemos usar (4)


e lembrar que se u = tg x ento du = sec 2 x dx para resolver fazendo
uma mudana de varivel. Se o integrando envolver a cotangente,
aplicamos (5) e lembremos que se u = cotg x ento du = - cossec 2 x dx .
Exemplo 2.5. Calcule as integrais.
a) tg 4 x dx ;
b) cotg 3 x dx .
Soluo.
a) Vamos reescrever o integrando usando a identidade (4). Temos,
tg 4 x = tg 2 x tg 2 x
= tg 2 x(-1 + sec 2 x)
= - tg 2 x + tg 2 x sec 2 x
= -(-1 + sec 2 x) + tg 2 x sec 2 x
= tg 2 x sec 2 x - sec 2 x + 1 .
Assim,

tg x dx = tg x sec x dx - sec x dx + dx .
Para calcular tg x sec x dx faremos
4

u = tg x e du = sec 2 x dx .
Portanto,

tg x dx = u
4

du - tg x + x

tg 3 x
- tg x + x + c .
3

b) Preparando o integrando temos,


cotg 3 x = cotg x cotg 2 x
= cotg x (-1 + cossec 2 x)
= cotg x cossec 2 x - cotg x .

104

Assim,

cotg x = cotg x cossec x dx - cotg x dx .


Para encontrar a integral cotg x cossec x dx faremos
3

u = cotg x e du = - cossec 2 x dx .
Portanto,

cotg x dx = - u du - ln | sen x |
3

- cotg 2 x
+ ln | cossec x | +c .
2

2.1.4 Integrais de potncias de sec x e cossec x


Para calcular integrais do tipo sec n x dx ou cossec n x dx , onde n
um nmero natural maior do que 2 , devemos analisar os casos em
que n um nmero par e n um nmero mpar.
1) Se n par ento devemos expressar o integrando como produto de potncias de forma que um fator seja sec 2 x ou cossec 2 x ,
usar as identidades (4) ou (5) e aplicar o mtodo da substituio da varivel.
2) Se n for mpar, ento devemos aplicar o mtodo da integrao
por partes.
Exemplo 2.6. Calcule as integrais.
a) sec 4 x dx ;
b) cossec3 x dx .
Soluo.
a) Temos sec 4 x = sec 2 x sec 2 x
= sec 2 x (tg 2 x + 1)
= sec 2 x tg 2 x + sec 2 x .
Assim,

sec x dx = sec x tg
4

x dx + sec 2 x dx .

105
Para calcular a integral sec 2 x tg 2 x dx faremos
2
u = tg x e du = sec x dx .

Portanto,

sec x dx = u
4

du + tg x

tg 3 x
+ tg x + c .
3

b) Temos,

cossec x dx = cossec x cossec x dx .


3

Fazendo
u = cossec x e dv = cossec 2 x dx , temos
du = - cossec x cotg x dx e v = - cotg x .
Assim,

cossec x dx = - cossec x cotg x - cotg x cossec x dx .


3

Usando a identidade
cotg 2 x = -1 + cossec 2 x
temos,

cossec x dx = - cossec x cotg x - (-1 + cossec x) cossec x dx


= - cossec x cotg x + cossec x dx - cossec xdx .
3

Portanto,

cossec x dx = 2 (- cossec x cotg x + ln | cossec x - cotg x |) + c .


3

2.1.5 Integrais de produtos de potncias


de tgx e sec x
Vamos considerar integral da forma
inteiros positivos.

tg

x sec n xdx , onde m, n so

Se m = n = 1 , ento a integral imediata.


Se n = 2 e m qualquer usamos o mtodo da substituio.
Se n for par, n > 2 e m qualquer, reescreva apenas
sec n x = sec 2 x sec n - 2 x , use a identidade (4) para expressar sec n - 2 x
em termos da tg x , e faa a mudana de varivel u = tg x .

106
Se m e n forem mpares, ento reescreva o integrando de
modo que um fator seja tg x sec x , use a identidade (4) para
expressar tg m -1 x em termos da sec x e faa a mudana de varivel u = sec x .
Se m for par e n mpar, ento use a identidade (4) para expressar tg m x em termos de sec x . A integral original se transformar numa soma de integrais de potncias de sec x . Use a tcnica anterior (Seo 2.1.4) para calcular as integrais resultantes.
Exemplo 2.7. Calcule as integrais.
a) tg 6 x sec 4 x dx ;
b) tg 3 x sec5 x dx .
Soluo.
a) O expoente da sec x par. Assim, vamos preparar o integrando de
2
modo a ter um fator sec x . Temos,
tg 6 x sec 4 x = tg 6 x sec 2 x sec 2 x
= tg 6 x (tg 2 x + 1) sec 2 x
= tg8 x sec 2 x + tg 6 x sec 2 x .
Assim,

tg x sec
6

x dx = tg8 x sec 2 x dx + tg 6 x sec 2 x dx .

Fazendo
u = tg x temos du = sec 2 x dx .
Portanto,

tg x sec
6

x dx = u 8 du + u 6 du
=

tg 9 x tg 7 x
+
+c.
9
7

b) O expoente da tg x mpar. Neste caso, vamos preparar o integrando usando a identidade (4) de modo a ter um fator tg x sec x .
Assim,
tg 3 x sec5 x = tg x sec x tg 2 x sec 4 x
= tg x sec x (sec 2 x - 1) sec 4 x
= sec6 x tg x sec x - sec 4 x tg x sec x .

107
Portanto,

tg x sec
3

x dx = sec6 x tg x sec x dx - sec 4 x tg x sec x dx


= u 6 du - u 4 du

sec7 x sec5 x
+c,
7
5
pois se u = sec x ento du = sec x tg xdx .
=

De forma semelhante tcnica apresentada acima, podemos obter


integrais de produtos de potncias cotg x e cossec x .
Outro processo para calcular integrais que envolvem potncias de
funes trigonomtricas dado no teorema abaixo. O processo consiste em expressar a integral que se deseja obter em outra que envolva um expoente menor.
Teorema 2.1. (Frmulas de Recorrncia). Para qualquer inteiro positivo n , temos:
-1 n -1
n -1
1) sen n x dx =
sen x cos x +
sen n - 2 x dx ;

n
n
1
n -1
2) cos n dx = cos n -1 x sen x +
cos n - 2 x dx ;
n
n
1
3) tg n x dx =
tg n -1 x - tg n - 2 x dx, n 1 ;
n -1
1
4) cotg n x dx = cotg n -1 x - cotg n - 2 x dx, n 1 ;
n -1
1
n-2
5) sec n x dx =
sec n - 2 x tg x +
sec n - 2 x dx, n 1 ;

n -1
n -1
-1
n-2
6) cossec n x dx =
cossec n - 2 x cotg x +
cossec n - 2 xdx, n 1.
n -1
n -1
Demonstrao. Provaremos os itens (3) e (5), os demais sero deixados como exerccio.
Para provar o item (3), escreveremos

tg

xdx = tg 2 x tg n - 2 xdx
= (-1 + sec 2 x) tg n - 2 xdx
= sec 2 x tg n - 2 xdx - tg n - 2 xdx .

108

Para calcular a primeira integral, faremos a substituio


u = tg x

du = sec 2 xdx .

Logo,

tg

xdx = u n - 2 du - tg n - 2 xdx
=

u n -1
- tg n - 2 xdx ,
n -1

1
tg n -1 x - tg n - 2 xdx .
n -1

(n 1)

Agora, provaremos o item (5). Vamos escrever


I = sec n x dx = sec n - 2 x sec 2 x dx .
Para usar integrao por partes faremos,
u = sec n - 2 x

dv = sec 2 x dx

du = (n - 2) sec n - 2 x tg x dx

v = tg x .

Logo,
I = sec n - 2 x tg x - (n - 2) sec n - 2 x tg 2 x dx .
Usando a identidade
tg 2 x = -1 + sec 2 x ,
segue que
I = sec n - 2 x tg x - (n - 2) sec n - 2 x (sec 2 x - 1) dx .
Assim,
I = sec n - 2 x tg x - (n - 2) sec n x dx + (n - 2) sec n - 2 x dx
= sec n - 2 x tg x - (n - 2) I + (n - 2) sec n - 2 x dx ,
ou seja,
(n - 1) I = sec n - 2 x tg x + (n - 2) sec n - 2 x dx .
Portanto,
I =

1
n-2
sec n - 2 x tg x +
sec n - 2 x dx , n 1 .

n -1
n -1

109

Exemplo 2.8. Aplique a frmula de recorrncia para calcular a integral sec3 x dx .


Soluo.
1

sec x dx = 2 sec x tg x + 2 sec x dx


3

1
= (sec x tg x + ln | tg x + sec x |) + c .
2

2.1.6 Integrais de funes envolvendo seno e


cosseno de arcos diferentes
Para calcular integrais do tipo

sen ax cos bx dx , sen ax sen bx dx e cos ax cos bx dx ,


onde a, b R , devemos usar a identidade correspondente:


1
[sen(a - b) x + sen(a + b) x ] ;
2
1
sen ax sen bx = [ cos(a - b) x - cos(a + b) x ] ;
2
1
cos ax cos bx = [ cos(a - b) x + cos(a + b) x ] .
2
sen ax cos bx =

(6)
(7)
(8)

As integrais acima tambm podem ser obtidas usando integrao


por partes.
Exemplo 2.9. Calcular a integral sen 5 x cos 2 x dx .
Soluo. Recorrendo a identidade (6) temos
1

sen 5 x cos 2 x dx = 2 (sen 3x + sen 7 x) dx


1 cos 3 x cos 7 x
=-
+
+c.
2 3
7

2.1.1 Exerccios
1) Calcular as integrais.
a) sen 5 xdx ;

b) cos 4 2x dx ;

110

d) sec3 ( x + 2) dx ;

c) e x tg 4 e x dx ;

e) sen 3 2 x cos 4 2 x dx ;
g) sen 4 x cos 5 x dx ;

f)

tg x sec

h)

cos3 x
sen 4 x dx .

x dx ;

2) Mostre que

-1
n-2
cossec n - 2 x cotg x +
cossec n - 2 xdx,
n -1
n -1
onde n um inteiro positivo e n 1 .

cossec x dx =
n

3) Verifique que

sen mx cos nx dx = 0 , para m, n Z .

2.2 Substituio trigonomtrica


Se o integrando envolve uma expresso da forma
a2 - x2 ,

x 2 + a 2 ou

x 2 - a 2 , com a > 0 ,

possvel fazer uma mudana de varivel (uma substituio trigonomtrica) e com o auxlio das identidades,
sen 2 + cos 2 = 1

ou

tg 2 + 1 = sec 2 ,

reescrever o integrando sem o radical. Vejamos:


(1) O integrando envolve a expresso

a 2 - x 2 com a > 0 .

Para este caso, vamos fazer a mudana de varivel


x = a sen (ver Figura 2.1).
Temos,
a 2 - x 2 = a 2 - (a sen ) 2
= a 2 (1 - sen 2 )
= a 2 cos 2
= a cos 2
= a | cos | .

111

a2 x2
Figura 2.1

-
Para facilitar os clculos, vamos supor
,
. Dessa forma,
2 2
cos 0 e | cos | = cos , logo a 2 - x 2 = a cos .

Exemplo 2.10. Calcular

4 - x2
dx .
x2

Soluo. Vamos fazer a mudana de varivel x = 2sen , ento


dx = 2 cos d , e
-
4 - x 2 = 2 cos com
.
,
2 2
Assim,
4 - x2
2 cos 2 cos d
x 2 dx = 4sen 2
=

cos 2 d
sen 2

= cotg 2 d
= (cossec 2 - 1) d
= - cotg - + c .
Agora, devemos retornar varivel original x . Temos x = 2sen com
-
. Nesse intervalo a funo seno inversvel, ento

,
2 2
x
= arcsen .
2
Para expressar a cotg em termos da varivel x , basta observar a
Figura 2.2. Logo,
cos
4 - x2
cotg =
=
.
sen
x

112

22 x 2
Figura 2.2

Portanto,

4 - x2
4 - x2
x
dx = - arcsen + c .
2
x
x
2

Exemplo 2.11. Calcular

1
2

9 - x2

dx .

ento
2
2
9 - x 2 = 3cos .

Soluo. Fazendo x = 3sen com dx = 3cos d

Assim,

1
2

9 - x2

3cos
d
9sen 2 3cos

dx =

1
d

9 sen 2
1
= cossec 2 d
9
1
= - cotg + c .
9
=

9 - x2
.
9 - x = 3cos ento, cotg =
x

Temos x = 3sen e

Portanto,

1
2

9 - x2

9 - x2
+c.
9x

dx = -

Exemplo 2.12. Calcular

a 2 - x 2 dx , a > 0 .

Soluo. Fazendo x = a sen com dx = a cos d

. Ento
2
2

a 2 - x 2 = a cos .

113

Mudando os limites de integrao, quando


x = 0 temos = 0 e quando x = a temos =

Portanto,

.
2

a 2 - x 2 dx = 2 (a cos )(a cos ) d


0

=a

=a

2
0

cos 2 d

2
0

1 + cos 2
d
2

a 2 2
= d + 2 cos 2 d
0
0
2

1
a2
=
2 + sen 2 2

2
2
0
0
=

a2 1

+ (sen - sen 0)

2 2 2

a2
.
4

Observe que, nesse exemplo, foi calculado um quarto da rea do crculo de raio a .
x 2 + a 2 com a > 0 .

(2) O integrando envolve a expresso

Neste caso, faremos a mudana de varivel


x = a tg ( ver Figura 2.3).

x2 + a2
x

a
Figura 2.3

114

< < . Note que com neste intervalo, a


2
2
funo tangente inversvel e a funo secante positiva. Ento
Vamos supor que -

x 2 + a 2 = a 2 tg 2 + a 2
= a 2 (tg 2 + 1)
= a 2 sec 2
= a | sec |
= a sec .
Exemplo 2.13. Calcular

x2 + 4
dx .
4

< < , temos


2
2
2
x + 4 = 2sec .

Soluo. Fazendo x = 2 tg com dx = 2sec 2 d

Assim,

x 2 + 4 dx 1
= (2sec )(2sec 2 ) d
4
4
= sec3 d .

Vamos calcular sec3 d usando a frmula de recorrncia (5).


1

sec d = 2 sec tg + 2 sec x dx


3

1
1
= sec tg + ln | sec + tg | +c .
2
2
Para retornar varivel original x , temos
tg =

x
e sec =
2

x2 + 4
.
2

Portanto,

x2 + 4
1
1 x + x2 + 4
dx = x x 2 + 4 + ln
+ c1
4
8
2
2
1
1
1 1
= x x 2 + 4 + ln x + x 2 + 4 + ln + c1
8
2
2 2
1
1
1 1
= x x 2 + 4 + ln x + x 2 + 4 + c onde c = ln + c1 .
2 2
8
2

115
Exemplo 2.14.
a) Mostre que

dx
2

a +x

= ln x + x 2 + a 2 + c , com a > 0 ;

b) Use o item a) para calcular

dx

x 2 + 2 x + 10

Soluo.

a) Para calcular a integral


varivel

x = a tg com -

dx
a2 + x2

, vamos fazer a mudana de

<< .
2
2

Ento
dx = a sec 2 d

a 2 + x 2 = a sec .

Assim,

dx
a2 + x2

a sec 2
d
a sec

= sec d
= ln | tg + sec | +c1 .
Devemos voltar varivel original x . Temos
x = a tg e
tg =

a 2 + x 2 = a sec ento

x
e sec =
a

x2 + a2
.
a

Portanto,

dx
a2 + x2

= ln

x
x2 + a2
+
+ c1
a
a

= ln x + x 2 + a 2 + ln

1
+ c1
a

= ln x + x 2 + a 2 + c , onde c = ln

b) Para calcular
Escrevemos

dx
x 2 + 2 x + 10
dx

x 2 + 2 x + 10

1
+ c1 .
a

, vamos completar quadrados.


dx
( x + 1) 2 + 9

116
Fazendo a mudana de varivel t = x + 1 , obtemos dt = dx . Assim,

dx
2

( x + 1) + 9

dt
t2 + 9

= ln t + t 2 + 9 + c

(pelo item a))

= ln ( x + 1) + ( x + 1) 2 + 9 + c .
Portanto,

dx
2

x + 2 x + 10

Exemplo 2.15. Calcule

= ln ( x + 1) + x 2 + 2 x + 10 + c .

du
9 + 4u 2

Soluo. Para calcular a integral

x = 2u e obtemos dx = 2du .

du
9 + 4u 2

faremos a substituio

Assim,
1
dx

2 x 9 + x2
9 + 4u
2
dx
=
.
x 9 + x2
Agora, fazendo

x = 3 tg com - < < ento


2
2
dx = 3sec 2 d e 9 + x 2 = 3sec .

du

Segue que

dx
9 + x2

3sec 2
d
3 tg 3sec

1 sec
d
3 tg

1
cossec d
3

1
= ln cossec - cotg + c1 .
3
Devemos escrever este resultado em termos da varivel original u .
Inicialmente, escrevemos em termos da varivel x e em seguida de
u . Como

117

9 + x2
x
ento
tg = e sec =
3
3
3
9 + x2
cotg = e cossec =
.
x
x
Portanto,

du

1
= ln cossec - cotg + c1
9 + 4u 2 3
1
= ln
3

9 + x2 3
- + c1
x
x

1
= ln
3

9 + 4u 2 - 3
+ c1
2u

1
= ln
3

9 + 4u 2 - 3
1 1
+ c , onde c = ln + c1 .
u
3 2

(3) O integrando envolve a expresso

x 2 - a 2 com a > 0 .

Neste caso, vamos fazer a mudana de varivel


x = a sec (ver Figura 2.4).
Temos,
x 2 - a 2 = a 2 sec 2 - a 2
= a 2 (sec 2 - 1)
= a tg 2
= a | tg | .
x

x2 a2

a
Figura 2.4

Para garantir que tg 0 e que a funo secante seja inversvel,


3
vamos supor 0, , .
2 2

118
Assim,
x 2 - a 2 = a | tg |
= a tg .
Exemplo 2.16. Calcular as integrais abaixo:
a)

b)

dx

com a > 0 ;

x2 - a2
dx
4 x2 - 5

Soluo.
a) Para calcular a integral

dx
x2 - a2

faremos a mudana de varivel

3
x = a sec onde 0, , . Ento
2 2
dx = a tg sec d

x 2 - a 2 = a tg .

Assim,

dx
2

x -a

a tg sec d
a tg

= sec d
= ln | tg + sec | +c1 .
Devemos voltar varivel original x . Temos,
x 2 - a 2 = a tg ento

x = a sec e
sec =

x
e tg =
a

x2 - a2
.
a

Portanto,

dx
x2 - a2

= ln

x2 - a2 x
+ + c1
a
a

= ln

x 2 - a 2 + x + c , onde c = ln

b) Para calcular a integral

dx
4x2 - 5

1
+ c1 .
a

faremos a mudana de varivel

t = 2 x e obtemos dt = 2dx .

119
Assim,
dx

4x - 5

1
dt

2
t2 - 5

1
= ln t 2 - 5 + t + c (pelo item a))
2
1
= ln
2
Exemplo 2.17. Calcular

4x2 - 5 + 2 x + c .

dx
2

x 2 - 16

Soluo. Fazendo a mudana de varivel

3
, ento
x = 4sec , onde 0 < ou <
2
2
dx = 4 tg sec d e

x 2 - 16 = 4 tg .

Assim,

dx
2

x - 16

4 tg sec
d
16sec 2 4 tg

1
1
d

16 sec

1
cos d
16

1
sen + c .
16

Vamos retornar varivel original x . Como,


tg =

x 2 - 16
tg
x
e sec = ento sen =
=
4
4
sec

x 2 - 16
.
x

Portanto:

dx
2

x 2 - 16

x 2 - 16
+c.
16 x

2.2.1 Exerccios
1) Calcular as integrais.
a)

9 - x2
dx ;
x2

b)

x2
x2 + 4

dx ;

120

c)

x2
x2 - 4

dx ;

2) Avalie as integrais abaixo:


3
dx
a)
;
0
9 + x2

d)

b)

9 - 4x 2 .
dx
x

dx
16 - x 2

2.3 Integrao de funes racionais:


mtodo das fraes parciais
Nesta seo estudaremos como calcular a integral de qualquer funo racional (quociente de polinmios) usando o mtodo das fraes
parciais. Este consiste em escrever a funo racional como soma de
fraes mais simples.
Para apresentar o mtodo, vamos considerar uma funo racional
f ( x) =

p( x)
,
q( x)

onde p ( x) e q ( x) so polinmios. Se o grau de p ( x) menor que o


grau de q ( x) , ento f ( x) uma funo racional prpria; caso contrrio, f ( x) chamada imprpria.
Uma funo racional imprpria pode ser escrita como soma de um
polinmio e uma funo racional prpria. Por exemplo:
x3 + x
2
= ( x 2 + x + 2) +
.
x -1
x -1
Para calcular a integral da funo f ( x) , devemos verificar inicialmente se f ( x) prpria ou imprpria. Se f ( x) prpria, ento
devemos escrever a funo como soma de fraes parciais mais simples (um resultado da lgebra garante que sempre possvel fazer
isso), e a partir desta soma encontramos a integral da funo f ( x) .
Se f ( x) imprpria, ento devemos dividir p ( x) por q ( x) e teremos f ( x) escrita como soma de um polinmio e uma funo racional prpria. Neste caso para obter a integral da f ( x) , basta encontrar a integral do polinmio (que se obtm facilmente) e a integral
da funo racional prpria.

121
A decomposio de uma funo racional prpria em uma soma de
fraes parciais mais simples est vinculada ao modo que o polinmio do denominador q ( x) se decompe em fatores lineares (da
forma ax + b ) e quadrticos irredutveis (da forma ax 2 + bx + c , onde
b 2 - 4ac < 0 ). Por exemplo:
a) Se q ( x) = 2 x 4 - 8 x 2 , ento q ( x) pode ser escrito como
q ( x) = 2 x x ( x - 2) ( x + 2) .
b) Se q ( x) = x 3 - 2 x 2 + 4 x - 8 , ento q ( x) pode ser escrito como
q ( x) = ( x 2 + 4)( x - 2) .
Para efetuar a decomposio da funo racional prpria, devemos
decompor q ( x) em fatores lineares e quadrticos e considerar os
fatores envolvidos na decomposio. Dependendo da natureza dos
fatores associamos a cada fator um tipo de frao parcial (as formas
das fraes parciais so garantidas por resultados da lgebra). Vejamos os quatro casos que ocorrem:
1 Caso: Fatores lineares distintos
Se q ( x) = (a1 x + b1 ) (a2 x + b2 ) (an x + bn ) , onde nenhum fator linear
p( x)
repetido, ento a funo racional prpria
corresponde a uma
q( x)
soma de n fraes parciais da forma:
An
A1
A2
p( x)
=
+
+ +
,
q( x)
a1 x + b1
a2 x + b2
an x + bn
onde A1 , A2 , , An so constantes que devem ser determinadas mediante a tcnica de coeficientes indeterminados ou a de substituio
de valores.
Exemplo 2.18. Calcular

x +8
dx .
+ x-2

Soluo. A funo racional f ( x) =

x +8
prpria.
x + x-2
2

Decompondo o polinmio do denominador temos


q ( x) = x 2 + x - 2 = ( x - 1) ( x + 2) .
Como q ( x) tem fatores lineares distintos, ento a funo racional
corresponde a uma soma de duas fraes parciais da forma

122

A
A
x +8
= 1 + 2 .
x + x - 2 x -1 x + 2
2

Para determinar A1 e A2 , vamos multiplicar ambos os lados da equao acima por ( x - 1) ( x + 2) , e assim obtemos
x + 8 = A1 ( x + 2) + A2 ( x - 1)
= ( A1 + A2 ) x + (2 A1 - A2 ) .
Igualando os coeficientes das mesmas potncias de x , segue que
A1 + A2 = 1
.

2 A1 - A2 = 8
Resolvendo o sistema temos
A1 = 3 e A2 = -2 .
Logo, a decomposio em fraes parciais
x +8
3
2
=
.
x + x - 2 x -1 x + 2
2

Portanto,

x+8
3
-2
dx =
dx +
dx
x + x-2
x -1
x+2
2

= 3ln | x - 1| -2 ln | x + 2 | +c .
O resultado acima tambm poderia ser obtido atravs do software
Maxima com o seguinte comando
partfrac ( (x + 8)/(x^2 + x - 2), x)
onde o primeiro argumento representa a funo a ser decomposta
em fraes parciais; e o segundo argumento, a varivel independente.
Observao: Existe outra maneira prtica para determinar os valores das constantes A1 e A2 . A equao
x + 8 = A1 ( x + 2) + A2 ( x - 1) satisfeita para todo x real.
Em particular vlida para x = 1 e x = -2 . Assim,
para x = 1 temos 9 = 3A1 , e
para x = -2 temos 6 = -3A2 .

123
Logo,
A1 = 3 e A2 = -2 .
Note que os valores de x considerados so os valores que anulam
os denominadores das fraes parciais, isto , so as razes do denominador q ( x) .
Exemplo 2.19. Calcular

x -1
dx .
+ x2 - 4x - 4

Soluo. O integrando uma funo racional prpria. As razes da


equao x 3 + x 2 - 4 x - 4 = 0 so
x = 2, x = -1 e x = -2 .
Assim,

A3
A1
A2
x -1
=
+
+
2
x - x - 4 x - 4 ( x - 2) ( x + 1) ( x + 2) .
3

Multiplicando a equao acima por ( x - 2) ( x + 1) ( x + 2) , obtemos


x - 1 = A1 ( x + 1) ( x + 2) + A2 ( x - 2) ( x + 2) + A3 ( x - 2) ( x + 1) .
Vamos determinar A1 , A2 e A3 usando o mtodo prtico. Substituindo
x = 2, x = -1 e x = -2 na equao acima, temos
1 = 12 A1

-2 = -3 A2 ,
-3 = 4 A
3

ou seja,
A1 =

1
,
12

A2 =

2
3

e A3 = -

3
.
4

Logo, a decomposio em fraes parciais


x -1
1
2
3
.
=
+
2
x + x - 4 x - 4 12( x - 2) 3( x + 1) 4( x + 2)
3

Portanto,

x -1
1
dx
2 dx 3 dx
dx =
+
2
+ x - 4x - x
12 x - 2 3 x + 1 4 x + 2
=

Exemplo 2.20. Calcular I =

1
2
3
ln | x - 2 | + ln | x + 1| - ln | x + 2 | + c.
12
3
4

x 4 - x3 - 3x 2 - 2 x + 2
dx .
x3 + x 2 - 2 x

124
Soluo. O integrando uma funo racional imprpria. Neste caso,
devemos fazer a diviso de polinmios. Temos,
x 4 - x3 - 3x 2 - 2 x + 2
x2 - 6x + 2
=
(
x
2)
+
.
x3 + x 2 - 2 x
x3 + x 2 - 2 x
Assim,
I = ( x - 2) dx +

x2 - 6x + 2
dx .
x3 + x 2 - 2 x

A primeira integral no apresenta dificuldade. Precisamos calcular a


segunda integral, para isso vamos decompor o integrando em fraes
parciais. Temos,
A
A
x 2 - 6 x + 2 A1
= + 2 + 3 .
3
2
x + x - 2x x x -1 x + 2
Multiplicando a equao acima por x( x - 1) ( x + 2) obtemos,
x 2 - 6 x + 2 = A1 ( x - 1) ( x + 2) + A2 x ( x + 2) + A3 x( x - 1) .
Agora,
para x = 0 temos 2 = -2A1 ,
para x = 1 temos - 3 = 3A2 ,
para x = -2 temos 18 = 6A3 ,
ou seja,
A1 = -1 ,

A2 = -1 e

A3 = 3 .

Logo,
I = ( x - 2) dx -

dx
dx
dx
-
+ 3
x
x -1
x+2

x2
= - 2 x - ln | x | - ln | x - 1| +3ln | x + 2 | +c .
2
2 Caso: Fatores lineares repetidos
Se um fator linear (ax + b) de q ( x) tem multiplicidade k , a esse fator
corresponder uma soma de k fraes parciais da forma
Ak
A1
A2
,
+
+ +
2
(ax + b) k
ax + b (ax + b)
onde A1 , A2 ,..., Ak so constantes a determinar.
Exemplo 2.21. Calcular

x2 + x + 1
x3 - 3x 2 + 3x - 1 dx .

125
Soluo. A equao q ( x) = x 3 - 3 x 2 + 3 x - 1 = 0 tem apenas uma raiz
real, x = 1 , com multiplicidade 3. Assim,
A3
A1
A2
x2 + x + 1
.
=
+
+
3
2
2
x - 3 x + 3 x - 1 ( x - 1) ( x - 1) ( x - 1)3
Multiplicando a equao por ( x - 1)3 , temos
x 2 + x + 1 = ( x - 1) 2 A1 + ( x - 1) A2 + A3 .
Substituindo x = 1 , x = 0 e x = -1 na equao acima, obtemos
A3 = 3

A1 - A2 + A3 = 1 ,
4 A - 2 A + A = 1
2
3
1
ou seja,
A1 = 1,

A2 = 3 e

A3 = 3 .

Tambm podemos escolher quaisquer outros valores de x para formar o sistema. Os valores considerados simplificam as contas.
Portanto,
x2 + x + 1
dx
dx
dx
x3 - 3x 2 + 3x - 1 dx = x - 1 + 3 ( x - 1)2 + 3 ( x - 1)3
= ln | x - 1| -

Exemplo 2.22. Calcular

3
3
+c .
( x - 1) 2( x - 1) 2

x +1
dx .
+ 4x2 + 4x

Soluo. As razes de q ( x) = x 3 + 4 x 2 + 4 x = 0 so x = 0 e x = -2 ,
sendo que x = -2 tem multiplicidade 2. Neste caso, o integrando por
ser escrito na forma
A3 .
A
A
x +1
= 1 + 2 +
2
x + 4 x + 4 x x - 0 x + 2 ( x + 2) 2
3

Multiplicando ambos os membros da equao por x ( x + 2) 2 , temos


x + 1 = A1 ( x + 2) 2 + A2 x ( x + 2) + A3 x .
Substituindo x = 0, x = -2 e x = -1 na equao acima obtemos,
respectivamente:

126

4 A1 = 1

.
-2 A3 = -1
A - A - A = 0
2
3
1
Resolvendo o sistema encontramos
1
1
1
A1 = , A2 = e A3 = .
4
4
2
Portanto,
x +1
1 dx 1 dx 1
dx
x3 + 4 x 2 + 4 x dx = 4 x - 4 x + 2 + 2 ( x + 2)2
1
1
1
= ln | x | - ln | x + 2 | +c.
4
4
2( x + 2)

3 Caso: Fatores quadrticos irredutveis distintos


A cada fator quadrtico irredutvel ax 2 + bx + c que aparece uma vez
em q ( x) , corresponder a uma frao parcial da forma
Ax + B
,
ax + bx + c
2

onde A e B so constantes a determinar.


Exemplo 2.23. Calcular

x2 - 2 x - 3
x3 + x 2 - 2 dx .

Soluo. A equao q ( x) = x 3 + x 2 - 2 = 0 tem somente uma raiz


real, x = 1 . Assim, o polinmio q ( x) decomposto como
q ( x) = ( x - 1) ( x 2 + 2 x + 2) ,

( x 2 + 2 x + 2 fator irredutvel)

e o integrando corresponde a uma soma de duas fraes parciais da


forma
x2 - 2 x - 3
A
Bx + C
=
+ 2
.
3
2
x + x - 2 x -1 x + 2x + 2
Multiplicado a equao acima por ( x - 1) ( x 2 + 2 x + 2) , temos
x 2 - 2 x - 3 = A( x 2 + 2 x + 2) + ( Bx + C ) ( x - 1)
= ( A + B ) x 2 + (2 A - B + C ) x + (2 A - C ) .

127

Igualando os coeficientes das mesmas potncias de x , obtemos


A + B =1

2 A - B + C = -2 .
2 A - C = -3

Resolvendo o sistema, encontramos


4
9
7
A=- , B=
e C= .
5
5
5
Logo,
x2 - 2 x - 3
4 dx 1
9x + 7
x3 - x 2 - 2 dx = - 5 x - 1 + 5 x 2 + 2 x + 2 dx
4
1 9x + 9 - 2
= - ln | x - 1| + 2
dx
5
5 x + 2x + 2
4
9
x +1
2
dx
= - ln | x - 1| + 2
dx - 2
.
5
5 x + 2x + 2
5 x + 2x + 2
Na primeira integral faremos a mudana de varivel
u = x 2 + 2 x + 2 ento du = (2 x + 2)dx .
Assim,

x +1
1 du
dx =
2 u
+ 2x + 2
1
= ln | x 2 + 2 x + 2 | +c1 .
2

Na segunda integral completamos o quadrado no denominador e obtemos


dx
dx
x 2 + 2 x + 2 = ( x + 1)2 + 1
= arctg( x + 1) + c2 .
Portanto,
x2 - 2 x - 3
4
9
2
2
x3 + x 2 - 2 dx = - 5 ln | x - 1| + 10 ln | x + 2 x + 2 | - 5 arctg( x + 1) + c.
x 2 - x - 21
dx .
Exemplo 2.24. Calcular 3
2 x - x2 + 8x - 4
Soluo. O polinmio q ( x) = 2 x 3 - x 2 + 8 x - 4 pode ser escrito como
q ( x) = (2 x - 1) ( x 2 + 4) .

128
Assim, como fator x 2 + 4 irredutvel, temos
x 2 - x - 21
A
Bx + C
=
+ 2
.
3
2
2 x - x + 8x - 4 2 x -1 x + 4
Multiplicando a equao por (2 x - 1) ( x 2 + 4) , temos
x 2 - x - 21 = A( x 2 + 4) + ( Bx + C ) (2 x - 1)
= ( A + 2 B ) x 2 + (- B + 2C ) x + (4 A - C )
Igualando os coeficientes das mesmas potncias de x , segue
A + 2B = 1

- B + 2C = -1 ,
4 A - C = -21

ou seja,
A = -5,

B=3 e

C = 1.

Portanto,
x 2 - x - 21
dx
3x + 1
2 x3 - x 2 + 8 x - 4 dx = -5 2 x - 1 + x 2 + 4 dx
5
x
dx
= - ln | 2 x - 1| +3 2
dx + 2
2
x +4
x + 22
5
3
1
x
= - ln | 2 x - 1| + ln( x 2 + 4) + arctg + c.
2
2
2
2
4 Caso: Fatores quadrticos irredutveis repetidos
Se um fator quadrtico irredutvel ax 2 + bx + c de q ( x) tem multiplicidade k , a esse fator corresponder uma soma de fraes parciais
da forma
Ak x + Bk
A1 x + B1
A2 x + B2
+
+ +
,
2
2
2
ax + bx + c
(ax + bx + c)
(ax 2 + bx + c) k
onde A1 , A2 ,..., Ak , B1 , B2 ,..., Bk so constantes a determinar.
Exemplo 2.25. Calcular

5 x3 - 3x 2 + 7 x - 3
x 4 + 2 x 2 + 1 dx .

Soluo. Decompondo o polinmio do denominador, temos


q ( x) = x 4 + 2 x 2 + 1 = ( x 2 + 1) ( x 2 + 1) .

129

Como q ( x) tem um fator quadrtico irredutvel com multiplicidade


2, ento
5 x3 - 3 x 2 + 7 x - 3 A1 x + B1 A2 x + B2
= 2
+ 2
.
x4 + 2x2 + 1
x +1
( x + 1) 2
Multiplicando a equao acima por ( x 2 + 1) 2 , temos
5 x3 - 3 x 2 + 7 x - 3 = ( A1 x + B1 ) ( x 2 + 1) + ( A2 x + B2 )
= A1 x 3 + B1 x 2 + ( A1 + A2 ) x + ( B1 + B2 ) .
Equacionando os coeficientes, temos
A1 = 5
B = -3
1
.

A1 + A2 = 7
B1 + B2 = -3
Resolvendo o sistema, obtemos
A1 = 5,

B1 = -3,

A2 = 2 e B2 = 0 .

Portanto,
5 x3 - 3x 2 + 7 x - 3
5x - 3
2x
x 4 + 2 x + 1 dx = x 2 + 1 dx + ( x 2 + 1)2 dx
x
dx
x
= 5 2
dx - 3 2
+ 2 2
dx
x +1
x +1
( x + 1) 2
5
1
= ln( x 2 + 1) - 3arctg x - 2
+c.
2
x +1
2 x3 - 5 x + 7
Exemplo 2.26. Calcular
dx .
( x 2 + 4) 2
Soluo. O integrando uma funo racional prpria. O polinmio
do denominador envolve um fator quadrtico irredutvel com multiplicidade 2. Assim, o integrando pode ser escrito na forma
2 x 3 - 5 x + 7 A1 x + B1 A2 x + B2
= 2
+
.
( x 2 + 4) 2
x + 4 ( x 2 + 4) 2
Multiplicando por ( x 2 + 4) 2 a equao acima, temos
2 x 3 - 5 x + 7 = ( A1 x + B1 )( x 2 + 4) + ( A2 x + B2 )
= A1 x 3 + B1 x 2 + (4 A1 + A2 ) x + (4 B1 + B2 ).

130
Igualando os coeficientes das mesmas potncias de x , obtemos
A1 = 2

B1 = 0

4 A1 + A2 = -5
4 B1 + B2 = 7
ou seja,
A1 = 2, B1 = 0,

A2 = -13 e B2 = 7 .

Logo,
2 x3 - 5 x + 7
2x
-13 x + 7
( x 2 + 4)2 dx = x 2 + 4 dx + ( x 2 + 4)2 dx
=

2x
x
1
dx - 13 2
dx + 7 2
dx.
2
x +4
( x + 4)
( x + 4) 2
2

Para calcular as duas primeiras integrais, faremos a mudana de varivel


u = x 2 + 4 e obtemos du = 2 x dx .
Assim,

2x
du
dx =

+4
u

= ln( x 2 + 4) + c1 ,
e

(x

x
1 1
dx = 2 du
2
+ 4)
2 u
1
=+ c2 .
2
2( x + 4)

Para obter a integral


trigonomtrica.

(x

dx
vamos recorrer a uma substituio
+ 4) 2

Fazendo
x = 2 tg , para -

< < , obtemos dx = 2sec 2 d .


2
2

Segue que,
dx
2sec 2
=
( x 2 + 4)2 (4 tg 2 + 4)2 d
=

2 sec 2
d
16 sec 4

131

1
cos 2 d

1 1 + cos 2
d
8
2

1
1

+
sen 2 + c3

16
2

1
[ + sen cos ] + c3 .
16

Para retornar varivel anterior x , vamos observar a Figura 2.5. Temos


x
2
x
sen =
, cos =
e = arc tg .
2
2
2
x +4
x +4

x2 + 4
x

2
Figura 2.5

Logo,

(x

dx
1
x
2x
= arc tg + 2
+ c3 .
2
+ 4) 16
2 x + 4

Portanto,
2 x3 - 5 x + 7
13
7
x
2x
2
( x 2 + 4)2 dx = ln( x + 4) + 2( x 2 + 4) + 16 arc tg 2 + x 2 + 4 + c.

2.3.1 Exerccios
1) Escreva as formas de decomposio em fraes parciais das
funes abaixo:
a)

2x +1
;
x - 2 x2 - 5x + 6
3

2x2 - x + 4
;
b)
x3 + 4 x

c)

x +1
;
x( x + 2 x + 3) 2
2

2x2
d) 3
.
x - x2 + x + 3

132
2) Calcule as integrais.
x +1
x3 + x 2 - 6 x dx ;
2 x3 + x
dx ;
b)
x -1
1
c)
dx ;
2
x( x + x + 1)
a)

x3 + x 2 + x + 2
x 4 + 3x 2 + 2 dx ;
2x2 + 3
dx .
e) 2
( x + 1) 2

d)

Sugesto. Veja o Exemplo 2.26.

2.4 Integrao de funes racionais


de seno e cosseno (substituio
universal)
Se o integrando uma funo racional de sen x e cos x , podemos
transform-lo em funo racional de u , fazendo a substituio
x
u = tg , x (- , ) .
2
x
Vamos expressar sen x em termos de tg , e assim, em termos de
2
u . Usando as relaes
x
x
x
x
sen x = 2sen cos
e sen 2 + cos 2 = 1
2
2
2
2
podemos escrever:
x
x
2sen cos
sen x
2
2 .
sen x =
=
x
x
1
sen 2 + cos 2
2
2
x
Agora, vamos dividir numerador e denominador por cos 2 . Por2
tanto:
x
2 tg
2 = 2u .
sen x =
2
x
+ 1 u +1
tg 2
2
1- u2
De forma semelhante mostra-se que cos x = 2
.
u +1
x
Note que ao fazermos u = tg com x (- , ) , temos x = 2 arctg u e
2
2
dx = 2
du .
u +1

133

Exemplo 2.27. Calcular

dx

1 + sen x .

Soluo. Fazendo u = tg
sen x =
Assim,

2u
u +1

x
temos
2
dx =

2
du .
u +1
2

2
dx
1
1 + sen x = u +2u du
1+ 2
u +1
2
u2 +1
= 2
2
du
u + 1 u + 2u + 1
2

2
du
u + 2u + 1
2

= 2
=
=

du
(u + 1) 2

-2
+c
(u + 1)

(Voltando varivel original)

-2
+c.
x
tg + 1
2

Exemplo 2.28. Calcular

dx

7 - 2 cos x .

x
temos
2
1- u2
2
cos x = 2
e dx = 2
du .
u +1
u +1
Portanto:
2
2
dx
1
7 - 2 cos x = u 1+- u 2 du
7 - 2 2
u +1

Soluo. Fazendo u = tg

2
u2 +1
= 2

du
u + 1 9u 2 + 5
=

2
du
9u + 5
2

134

2
1
du

9 u2 + 5
9
2 3
3u
=
arc tg
+c
9 5
5
=

2 5
3 5
arctg
u+c
15
5

3 5 x
2 5
arctg
tg + c .
15
2
5

Exemplo 2.29. Calcular

(Voltando varivel original)

dx

1 + sen x - cos x .
x
temos
2
1- u2
cos x = 2
u +1

Soluo. Fazendo u = tg
sen x =
Assim,

2u
,
u +1
2

dx =

2
dx
1
1 + sen x - cos x = 2uu + 1 - u 2 du
1+ 2
u +1 u2 +1
2

2
du
2u + 2u

du
u (u + 1)

du
du
-
u
u +1

= ln | u | - ln | u + 1| +c
= ln tg

= ln

x
x
- ln 1 + tg + c
2
2

tg

x
2

x
1 + tg
2

+c.

2
du .
u +1
2

135

2.4.1 Exerccios
1) Calcular as integrais.
a)

1 + sen x

1 + cos x dx ;

2) Mostre que

dx

sen x

b)

= ln tg

x
2

dx

4sen x - 3cos x .
+ c.

Exerccios de fixao
1) Calcular as integrais abaixo:
1 - sen x
dx ;
cos x

a)

sen

x cos3 x dx ;

h)

b)

cotg x dx ;

i)

sec x dx ;

c)

cossec x dx ;

j)

tg

d)

5
4
cos x sen x dx ;

k)

sec 2 x
cotg x dx ;

e)

(1 - sen 2 x) dx ;

l)

sen 5 x sen 2 x dx ;

f)

3
sen x cos xdx ;

m)

g)

cos x tg x dx ;

x sec x dx ;

1 - tg 2 x
sec2 x dx ;
x
n)
tg 3 x 2 - 1 dx .
2
x -1

2) Calcule as integrais:
a)

cotg 2 x dx

b)

cos3 x dx

3) Calcular a rea sob o grfico de y = sen 3 x , de 0 at .


4) Calcular a rea entre as curvas y = sen 2 x e y = cos 2 x

3
at
.
de
4
4
5) Encontre a rea da regio limitada pelas curvas dadas

y = sen x, y = sen 3 x, x = 0 e x = .
2

136

0, se m n
6) Mostre que - sen mx sen nx dx =
, onde m e n so
, se m = n

inteiros positivos.

7) Calcular as integrais.
a)
b)
c)
d)

dx

e)

f)

g)

dx ;

h)

4 + x2
dx

3 - x2
dx

16 - 9 x 2
ex

dx

e2 x + 1

b2 x2 - a 2

, onde a, b > 0 ;

a2 - x2
dx , onde a > 0 ;
x2

x2
x2 - 4

dx ;

x
x2 + 4 x - 5

dx ,

Sugesto. Escreva x 2 + 4 x - 5 = ( x + 2) 2 - 9 .
x2 y 2
8) Encontre a rea limitada pela elipse 2 + 2 = 1 .
a
b
9) Use substituio trigonomtrica para mostrar que:
a)

b)

dx
2

x +a
2

= ln x + x 2 + a 2 + c , onde a > 0 ;

dx
1
x+a
=
ln
+ c , onde a > 0 .
2
-x
2a x - a

10) Calcule as integrais.


x3
dx ;
x2 + 1

a)

2x -1
( x - 1) ( x - 2)dx ;

e)

b)

3x - 7
x3 + x 2 + 4 x + 4 dx ;

f)

x2 + 1
x 2 - x dx ;

c)

(x

g)

3x

h)

2x

dx
;
- x) ( x 2 - x + 1) 2

4x4
dx ;
d) 4
x - x3 - 6 x 2 + 4 x + 8

dx
;
+ 7x + 2
dx
.
+ x2 + 2x + 1

137

4x2 - 2x + 7
11) Decomponha a funo f ( x) =
( x - 2)3 (2 x + 3) (2 x 2 + 5 x + 7) 2
numa soma de fraes parciais. No necessrio determinar
os valores numricos dos coeficientes.
12) Calcule as integrais.
dx
a)
;
x-3 x

b)

dx
.
x+4 x

Sugesto. Substituir x = u 6 .
13) Calcular a rea da regio limitada pelas curvas
1
1
y=
, y=
, x = 2 e x = 3.
( x - 1) ( x - 4)
(1 - x) ( x - 4)
x
para transformar o integrando em
2
uma funo racional de u e calcule a integral.

14) Use a substituio u = tg

dx

a)

2 + cos x ;

b)

3sen x - 4 cos x dx ;

cos x

c)

1 + cos x dx ;

d)

1 + cos x dx .

b)

ln ( x

sen x

15) Calcule as integrais:


a)

ln ( x +

1 + x 2 ) dx ;

Sugesto. Use integrao por partes.

- x + 2) dx .

138

Resumo
Neste captulo estudamos mtodos para calcular integrais cujos integrandos envolvem:
1) Funes trigonomtricas
Para calcular uma integral que envolve funo trigonomtrica,
devemos observar se possvel simplificar o integrando e usar
identidades trigonomtricas;
2) Funes com expresses da forma

a2 - x2 ,

x2 + a2 e

x 2 - a 2 com a > 0 .
Se o integrando envolve expresso do tipo a 2 - x 2 , ento devemos fazer a mudana de varivel x = a sen ;
Se o integrando envolve expresso do tipo x 2 + a 2 , ento devemos fazer a mudana de varivel x = a tg ;
Se o integrando envolve expresso do tipo x 2 - a 2 , ento devemos fazer a mudana de varivel x = a sec .
3) Funes racionais - Mtodo de Fraes Parciais
p( x)
Seja
uma funo racional prpria. Suponhamos q ( x) deq( x)
composto em fatores lineares e/ou quadrticos irredutveis. Se
q ( x) possuir:
Fatores lineares distintos, ento a cada fator ax + b associamos
a frao
A
.
ax + b
Fatores lineares repetidos, ento a cada fator ax + b de multiplicidade de k corresponde soma de fraes
Ak
A1
A2
+
+ +
.
2
ax + b (ax + b)
(ax + b) k
Fatores quadrticos irredutveis distintos, ento a cada fator
ax 2 + bx + c corresponde frao
Ax + B
.
2
ax + bx + c

139

Fatores quadrticos irredutveis repetidos, ento a cada fator


(ax 2 + bx + c) de multiplicidade k , associamos soma de fraes
Ak x + Bk
A1 x + B1
A2 x + B2
+
++
.
2
2
2
(ax + bx + c) (ax + bx + c)
(ax 2 + bx + c) k
4) Funes racionais de seno e cosseno
x
Fazer a substituio u = tg com x (- , ) . Assim, x = 2 arctg u
2
2
e dx = 2
du .
u +1
2u
1- u2
.
Lembrar que sen x = 2
e cos x = 2
u +1
u +1

Tabelas
Sejam u e v funes derivveis de x e n constante.

Tabela de derivadas
1) y = u n

y ' = n u n -1u ' .

2) y = u v

y ' = u 'v + v 'u .

u

v

y'=

4) y = a u

y ' = a u (ln a ) u ', (a > 0, a 1) .

5) y = eu

y ' = eu u ' .

3) y =

6) y = log a u
7) y = ln u

8) y = u v

9) y = sen u

u 'v - v 'u
.
v2

u'
log a e .
u
u'
y' = .
u
y' =

y ' = v u v -1 u '+ u v (ln u ) v ' (u > 0) .


y ' = u 'cos u .

140

10) y = cos u

y ' = -u 'sen u .

11) y = tg u

y ' = u 'sec 2 u .

12) y = cotg u

y ' = -u 'cossec 2 u .

13) y = sec u

y ' = u 'sec u tg u .

14) y = cossec u

y ' = -u 'cossec u cotg u .

15) y = arcsen u

y'=

16) y = arccos u

y' =

17) y = arctg u

y' =

u'
.
1+ u2

18) y = arc cotg u

y' =

-u '
.
1+ u2

19) y = arc sec u ,

y' =

20) y = arc cossec u ,

y' =

u'
1- u2
-u '
1- u2

u u2 -1
-u '
u u2 -1

1) du = u + c .
u n +1
+ c, n -1 .
n +1

du
= ln u + c .
u
au
+ c, a > 0, a 1 .
4) a u du =
ln a
3)

5) eu du = eu + c .
6) sen u du = - cos u + c .

u'

Tabela de integrais

2) u n du =

, u > 1.
, u > 1.

141

7) cos u du = sen u + c .
8) tg u du = ln sec u + c .
9) cotg u du = ln sen u + c .
10) sec u du = ln sec u + tg u + c .
11) cossec u du = ln cossec u - cotg u + c .
12) sec u tg u du = sec u + c .
13) cossec u cotgu du = - cossec u + c .
14) sec 2 u du = tg u + c .
15) cossec 2 u du = -cotg u + c .
16)

17)

du
1
u
= arctg + c .
2
+a
a
a

du
1
u-a
= ln
+ c. .
2
u -a
2a u + a

18)

19)

20)

21)

du
2

u +a

du
2

u -a

du
2

a -u

= ln u + u 2 + a 2 + c .
= ln u + u 2 - a 2 + c .
= arcsen

du
u2 - a2

u
+ c, u 2 < a 2 .
a

1
u
arc sec + c .
a
a

Frmulas de recorrncia
sen n -1u cos u n - 1
+
sen n - 2u du
1) sen u du =
n
n
n

142

sen u cos n -1 u n - 1
2) cos u du =
+
cos n - 2 u du .

n
n
n -1
tg u
3) tg nu du =
- tg n - 2u du .
(n - 1)
n

4) cotg nu du = 5) sec n u du =

cotg n -1u
- cotg n - 2u du
(n - 1)

sec n - 2 u tg u n - 2
+
sec n - 2 u du

(n - 1)
n -1

6) cossec n u du = 7)

cossec n - 2 u cotg u n - 2
+
cossec n - 2 u du

(n - 1)
n -1

du
u (u 2 + a 2 )1- n
2n - 3
du
,
=
+ 2
2
2 n
2
2

(u + a )
2a (n - 1)
2a (n - 1) (u + a 2 ) n -1

onde a * e n , n > 1 .

Identidades trigonomtricas
1) sen 2 x + cos 2 x = 1 .
2) 1 + tg 2 x = sec 2 x .
3) 1 + cotg 2 x = cossec 2 x .
4) sen 2 x =

1 - cos 2 x
.
2

5) cos 2 x =

1 + cos 2 x
.
2

6) sen 2 x = 2 sen x cos x .


7) 2 sen x cos y = sen( x - y ) + sen( x + y ) .
8) 2 sen x sen y = cos( x - y ) - cos( x + y ) .

9) 2 cos x cos y = cos( x - y ) + cos( x + y ) .

143

Respostas dos exerccios


2.1.1 Exerccios
2 cos3 x cos5 x
+c;
3
5
3
sen 4 x sen 8 x
b) x +
+
+c;
8
8
64

1) a) - cos x +

c)

1 3 x
tg e - tg e x + e x + c ;
3

d)

1
[tg( x + 2) sec( x + 2) + ln | sec( x + 2) + tg( x + 2) |] + c
2

e)

1 - cos5 2 x cos 7 2 x
+

+c;
2
5
7

tg 6 x tg 4 x
+
+c;
f)
6
4
g)

1
1

cos x - cos 9 x + c ;
2
9

h)

1
-1
+
+c.
3
3sen x sen x

2.2.1 Exerccios
1)

9 - x2
x
- arcsen + c ;
a) x
3
1
2
b) x x 2 + 4 - ln( x 2 + 4 + x) + c ;
6
3
c)

1
x x 2 - 4 + 2 ln( x 2 + x 2 - 4) + c ;
2

3 - 9 - 4x2
d) 3ln

2) a) ln(1 + 2) ;

+ 9 - 4x2 + c .

b)

;
6

144

2.3.1 Exerccios
1) a) -

1
7
1
+
;
2( x - 1) 10( x - 3) 5( x + 2)

b)

1 x -1
+
;
x x2 + 4

c)

1
1- x
x+2
+
;
2
2
2
9 x 3( x + 2 x + 3) 9( x + 2 x + 3)

d)

1
5x - 3
+
.
2
3( x + 1) 3( x - 2 x + 3)

1
3
2
2) a) - ln | x | + ln | x - 2 | - ln | x + 3 | +c ;
6
10
15
b)

2 3
x + x 2 + 3 x + 3ln | x - 1| +c ;
3

c)

1
x2
1
2x +1
+c;
ln 2
arctg
2 x + x +1
3
3

1
d) arctg x + ln( x 2 + 2) + c ;
2
1
1 x
+c.
e) 2 arctg x + arctg x +
2
2 x2 + 1

2.4.1 Exerccios
x
x
1) a) tg + ln tg 2 + 1 + c
2
2

x 1
tg 1
b) ln 2 3 + c .
5 tg x + 3
2

Exerccios de fixao
1) a)

sen 3 x sen 5 x
+c;
3
5

b) ln | sen x | +c ;
c) ln | cossec x - cotg x | +c ;
1
2
1
sen 5 x - sen 7 x + sen 9 x + c ;
5
7
9
3x
1
e)
+ cos 2 x - sen 4 x + c ;
2
8
d)

145

2
2

f) cos3 x - cos x cos x + c ;


3
7

g)

1
cos 2 x - ln | cos x | +c ;
2

h) ln(1 + sen x) + c ;
1
i) tg x + tg 3 x + c ;
3
1
j) sec3 x - sec x + c ;
3
1
k) tg 2 x + c ;
2
l)

1 1
1

sen 3 x - sen 7 x + c ;

2 3
7

1
sen 2 x + c ;
2
1
n) tg 2 x 2 - 1 + ln cos x 2 - 1 + c .
2

m)

2) a)
3)

3-

;
3

4
u.a.
3

b) 0.
4) 1u.a.

5)

1
u.a.
3

- 4 + x2
+ c ;
4x
x
+c;
b) arcsen
3
1
3x
c) arc sec + c ;
3
4

7) a)

d) ln ( e 2 x + 1 + e x ) + c ;
e)

1
ln bx + b 2 x 2 - a 2 + c ;
b

f)

- a2 - x2
x
- arcsen + c ;
x
a

146

g)

1
x x 2 - 4 + 2 ln ( x + x 2 - 4) + c ;
2
x 2 + 4 x - 5 - 2 ln

h)

x 2 + 4 x - 5 + ( x + 2) + c .

8) A = ab
10) a) ln

( x - 2)3
+ c ;
x -1

x2 + 4 1
x
+ arctg + c ;
b) ln
2
2
( x + 1) 2
c) ln

x - 1 10
2x -1
2x -1
+c;
arctg

2
x
3 3
3 3( x - x + 1)

4
68
16
d) 4 x + ln | x + 1| -4 ln | x + 2 | + ln | x - 2 | +c ;
9
9
3( x - 2)
e)

(1 - ln 2)
;
2

f) x + ln

( x - 1) 2
+c.
x

12) a) 2 x + 3 3 x + 6 6 x + 6 ln
13)

x -1 + c .

4
ln 2 u.a.
3

14) a)
b)

3 1
2 3
arctg
tg x + c ;
3
2
3
1
x
x

ln 2 tg - 1 - ln tg + 2 + c ;

5
2
2

x
c) x - tg + c ;
2
x

d) ln 1 + tg 2 + c .
2

15) a) x ln ( x + 1 + x 2 ) - 1 + x 2 - c ;
1
2x -1
b) x ln x 2 - x + 2 - 2 x - ln x 2 - x + 2 + 7 arctg
+c.
2
7

Captulo 3
Aplicaes de Integral

Captulo 3
Aplicaes de Integral
Neste captulo voc vai:
1. Resolver, utilizando integrais, equaes diferenciais
ordinrias de primeira ordem e separveis;
2. Calcular o comprimento de arco de curvas planas;
3. Calcular o volume de slidos de revoluo;
4. Calcular a rea de superfcies de revoluo;
5. Calcular as coordenadas do centro de massa de figuras planas;
6. Calcular a rea de regies planas delimitadas por
curvas em coordenadas polares.

3.1 Equaes diferenciais de


primeira ordem com variveis
separveis
O teorema fundamental do clculo nos leva concluso de que o
clculo integral , em certo sentido, uma operao inversa do clculo diferencial. Isso significa que, quando calculamos a integral
de uma funo f , estamos calculando, de fato, sua antiderivada, ou seja, estamos encontrando uma funo F , cuja derivada
a funo f . Podemos ainda elaborar questes mais complexas,
como, por exemplo: encontrar uma funo y = y (t ) que satisfaa,
junto com sua primeira derivada, uma igualdade do tipo
F (t , y, y) = 0 .

(3.1)

Esse tipo de igualdade apresentada na expresso (3.1) denominamos equao diferencial ordinria (EDO).
Definio 3.1. Uma equao diferencial ordinria (EDO) uma
igualdade envolvendo uma funo y , de uma varivel independente t , e suas derivadas em relao a essa varivel independente

150

em um determinado intervalo I . Em outras palavras, uma


igualdade do tipo
F (t , y, y, y,..., y ( n ) ) = 0, t I .
A ordem de uma equao diferencial definida como a derivada de maior
ordem da funo que ocorre na EDO.
No caso da equao (3.1), a EDO dita ser de primeira ordem, pois
somente envolve a primeira derivada da funo, e no derivadas de
ordem superior.
Cabe-nos, aqui, uma pequena observao que as EDOs no so as
nicas equaes diferenciais que podem existir. Tambm podemos
ter equaes diferenciais parciais (EDPs) que envolvem funes u ,
dependentes de duas ou mais variveis independentes, por exemplo
u = u( x, y , z, t ) , e as derivadas parciais desta funo u . As EDPs esto, por enquanto, fora de nosso escopo, por isso, vamos nos ocupar
apenas com equaes diferenciais ordinrias.
Como exemplos de equaes diferenciais, podemos citar:
1)

, y = y (t ) , para t . Neste caso, temos


uma EDO de terceira ordem.

2) y (4) + ty-t 2 y+20 = 0 , y = y (t ) , para t . Neste caso temos


uma EDO de quarta ordem.
3)

, y = y (t ) , para t . Neste caso temos uma EDO de


segunda ordem.

4) y= (1 - y ) y , y = y (t ) , para t , Neste caso temos uma EDO


de primeira ordem.
Como vamos abordar neste captulo to somente as EDOs de primeira ordem, vamos primeiramente mostrar alguns problemas motivadores envolvendo esse tipo de EDO, explicitando, assim, algumas de suas aplicaes.
Em primeiro lugar, considere uma cultura de bactrias em laboratrio. Sabemos que a reproduo das bactrias assexuada, cada indivduo se divide em dois ou mais indivduos idnticos, esse processo

151
denomina-se cissiparidade. Assim, quanto maior a populao de
bactrias, maior ser sua velocidade de reproduo, visto que existem mais indivduos reproduzindo, ou seja, a taxa de crescimento
populacional proporcional ao nmero de indivduos. Vamos supor
que cada indivduo dessa populao possui a mesma capacidade reprodutiva. Assim, se denotarmos por N (t ) o nmero de indivduos
em um certo instante de tempo t , ento a velocidade de crescimento
dessa populao ser dada pela derivada desta funo em relao
ao tempo, N (t ) . Assim, como supusemos que o crescimento da populao era proporcional ao nmero de indivduos e que cada indivduo possua a mesma capacidade reprodutiva, podemos escrever
a lei de crescimento dessa populao da seguinte forma
N (t ) = .N (t ) ,

onde esta constante codifica a homogeneidade da capacidade reprodutiva dos indivduos dessa populao. Para descrevermos a
funo que indica o nmero de indivduos dessa populao em funo do tempo, precisamos encontrar uma funo real cuja primeira
derivada seja proporcional prpria funo. O leitor j deve desconfiar que tal funo ser uma exponencial. Nesta EDO, a varivel
independente a varivel t , e a varivel dependente a varivel N .
Um problema semelhante ao primeiro o problema do decaimento radioativo. A resoluo exata desse problema possui inmeras
aplicaes, como, por exemplo, o clculo da idade de uma rocha
ou ento a datao de um fssil. O fato que, na natureza, existem
certos elementos qumicos cujos ncleos atmicos so instveis e
ao longo do tempo eles emitem partculas eletricamente carregadas (ncleos de Hlio, na radiao alfa, ou eltrons, na radiao
beta) e com isso mudam o seu nmero atmico e se tornam outros
elementos qumicos mais estveis, denominados descendentes. Ao
se examinar uma amostra em uma rocha, pode-se verificar as porcentagens relativas do elemento qumico radioativo e seus descendentes. A taxa de decaimento de uma amostra tambm depende
da quantidade do elemento presente na amostra. Tendo em vista
que cada tomo individualmente tem a mesma probabilidade de
emitir radiao e, portanto, decair em seus elementos descendentes, ento se a massa em um determinado momento for igual a
M (t ) , a taxa de decaimento neste instante de tempo ser dada por
M (t ) = - .M (t ) ,

152

onde a constante discrimina as caractersticas prprias do elemento radioativo a ser analisado, e o sinal negativo indica que a massa
desse elemento uma funo decrescente com o tempo. Novamente, essa EDO tem como varivel independente a varivel t . Na prtica, a constante de decaimento determinada experimentalmente
atravs da determinao do tempo de meia-vida do elemento. O que
se faz colocar uma amostra de massa M 0 do material e medir
com preciso o tempo T que demora para essa amostra decair at
o ponto em que se tenha a metade da massa original do elemenM
to radioativo, ou seja, M (T ) = 0 . Ento se substitui estes dados na
2
soluo da EDO acima com a condio inicial M (0) = M 0 , ou seja,
M (t ) = M 0 .exp(- t ) .
Substituindo na equao os dados obtidos da medida do tempo de
meia-vida, temos
M0
= M 0 .exp(- t ) ,
2

obtendo, assim
1
- t = 1n = -1n2 ,
2

e, portanto:
=

1n2
,
T

onde a funo ln x significa o logaritmo na base e de um nmero,


ou seja, o seu logaritmo natural .
Neste captulo, vamos tratar apenas de um caso especfico de EDO
de primeira ordem, as equaes separveis. As duas EDOs apresentadas nos exemplos so deste tipo.
Definio 3.2. Diz-se que uma EDO de primeira ordem separvel
se puder ser escrita na seguinte forma
f ( y ) y= g ( x ) .

(3.2).

A soluo dessa equao,


como veremos, dada
pela funo exponencial,
que ser denotada por
exp( x ) na maioria das
vezes por questo de
clareza na notao. Onde
for possvel, utilizaremos
tambm a notao e x para
a mesma funo. A questo
meramente esttica,
pois ao denotarmos a
funo exponencial de
uma expresso muito
grande, ocorrer que os
caracteres ficaro muito
minsculos se adotarmos a
segunda notao como um
expoente, de fato. Por isso a
necessidade de utilizarmos
a primeira notao.

153

Nosso objetivo apresentar um mtodo de resoluo de EDOs de


primeira ordem com variveis separveis como exemplo de aplicao do clculo integral. claro que o estudo das EDOs no se limita
s de primeira ordem e separveis, existe toda uma rea da matemtica que se dedica ao estudo. Primeiramente precisamos dizer o
que significa resolver uma EDO, ou seja, o que significa uma soluo
para uma EDO. De agora em diante, tudo o que fizermos ser relativo apenas a EDOs de primeira ordem.
Definio 3.3. Uma soluo, ou curva integral, da EDO F ( x, y, y) = 0 ,
uma funo = ( x) satisfazendo igualdade
F ( x, ( x), ( x)) = 0 ,

para todo x no domnio da funo .


Resolver uma EDO envolve o clculo de uma integral indefinida,
ento uma constante arbitrria automaticamente introduzida em
sua soluo. Isso significa que ao resolvermos uma EDO, no obtemos apenas uma soluo individual, mas uma famlia infinita de
solues qual denominaremos soluo geral da EDO. Essa arbitrariedade somente fixada ao estabelecermos condies iniciais,
isto , ao dizermos que a soluo especfica uma funo y = y ( x ) ,
satisfazendo condio y ( x0 ) = y0 , para algum x0 no domnio das
funes y . Quando resolvemos uma EDO especificando uma condio inicial, dizemos que resolvemos um problema de valor inicial.
O mtodo de resoluo de EDOs de primeira ordem separveis
simples. Consideremos a forma j separada da EDO
f ( y ) y= g ( x ) .
Integrando ambos os membros da igualdade acima em relao
varivel x , temos
x

x0

f ( y ( )) y( )d = g ( )d .
x0

154

Utilizando o mtodo de substituio de variveis na integral no primeiro membro da igualdade, definindo u = y ( ) , podemos ainda
obter
y( x)

y0

f (u ) = g ( )d .
x0

Seja F a funo real, tal que F (u ) = f (u ) e G a funo real tal que


G(t ) = g (t ) , ento, pelo teorema fundamental do clculo, podemos
ainda escrever
F ( y ( x )) - F ( y0 ) = G ( x ) - G ( x0 ) ,
ou, ainda,
F ( y ( x )) = G ( x ) + K ,

(*)

onde K = F ( y0 ) - G ( x0 ) uma constante arbitrria que depende totalmente de condies iniciais. Algumas vezes, a soluo geral da
EDO tem de ser deixada na forma implcita dada pela frmula (*),
isto , no possvel isolar a varivel y de forma a escrevermos
y = h( x ) .
Exerccio resolvido 3.1. Resolva a EDO y= y (1 - y ) .
Soluo: Primeiramente, temos que observar que as funes constantes y = 0 e y = 1 so solues da EDO, o mtodo de resoluo
abaixo ser utilizado para determinar as outras solues desta. Colocando na forma separada, temos
y
= 1.
y (1 - y )
Integrando-se ambos os membros da igualdade acima, podemos escrever
x
x
y(t )
dt
=
y(t )(1 - y(t )) x dt .
x0
0
Podemos, ainda, efetuar a mudana de variveis na integral no primeiro membro, obtendo
y
du
y u(1 - u) = x - x0 .
0

155

A integral acima utiliza o mtodo de fraes parciais para ser resolvida, isto , precisamos encontrar constantes A e B , tais que
1
A
B
= +
.
u(1 - u ) u 1 - u
Multiplicando-se ambos os membros por u (1 - u ) obtemos
A(1 - u ) + Bu = 1 ,
que equivalente ao sistema
A =1
B- A=0
cuja soluo A = B = 1 . Assim, temos que
1
1
1
= +
.
u(1 - u ) u 1 - u
Voltando integral, temos
y( x)

ou seja,

y0

du
+
u

y( x)

y0

du
= x - x0 ,
1- u

ln y ( x ) - ln y0 - ln(1 - y ( x )) + ln(1 - y0 ) = x - x0 .
Aps algumas manipulaes algbricas, obtemos

ou, ainda,

y
y( x)
ln
= x - x0 + ln 0 = x + K ,

1 - y( x)
1 - y0
y( x)
= e x + K = Ce x .
1 - y( x)

A soluo est na forma implcita, mas ainda podemos isolar a funo


y ( x ) , obtendo, finalmente,
Ce x
y( x) =
.
1 + Ce x
A constante C pode ser fixada atravs de condies iniciais. Por outro lado, se as condies iniciais fossem dadas, elas poderiam ter sido
colocadas desde o incio nos limites de integrao, no lugar de x0 e
y0 . Note que essa soluo vlida apenas no subconjunto \ {a} ,
onde a o nmero real tal que 1 + Ce a = 0 .

156
x2
Exerccio resolvido 3.2. Resolva a EDO y=
.
1 + y2
Soluo: Novamente, temos que colocar a EDO na forma separada,
obtendo
(1 + y 2 ) y= x 2 .
Temos que integrar os dois membros em relao varivel x , o que
nos resulta em
x

x0

x0

2
2
(1 + y(t ) ) y(t )dt = t dt ,

ou ainda, depois de uma mudana de variveis na integral no primeiro


membro,
y( x)
x 3 x03
2
(1
+
u
)
du
=
,

3
3
y0
que nos d a seguinte igualdade:
y( x) +

y ( x )3
y 3 x3 x 3
- y0 + 0 = - 0 .
3
3
3
3

Agrupando todas as constantes, finalmente temos


y( x) +

y ( x )3 x 3
=
+K,
3
3

que a soluo na forma implcita. Deixamos para o leitor a tarefa de


verificar se possvel escrever a soluo na forma explcita em algum
subconjunto de e, caso seja possvel, escrev-la.
Exerccio resolvido 3.3. Resolver a EDO y= - y , com a condio
inicial y (0) = 2 .
Soluo: Devido condio inicial do problema, a soluo desejada
no a identicamente nula, assim a EDO pode ser escrita como
y
= -1 ,
y
que, integrando na varivel x , resulta em
x

y(t )
dt = - dt .
y (t )
0

A mudana de varivel de integrao na integral do primeiro membro


nos fornece o seguinte resultado:

157
y( x)

du
y( x)
= ln y ( x ) - ln 2 = ln
= -x .
u
2

Uma pequena manipulao algbrica nos fornece


y ( x ) = 2e - x .
Note que na soluo no existem constantes arbitrrias, pois a condio inicial foi dada. Portanto, estamos falando de uma nica soluo
que satisfaz a aquela condio inicial dada. Note tambm que esta
soluo est definida em I = .

3.1.1 Exerccios
1) Resolva as seguintes EDOs separveis:
a) yy= x 2 ;
x2
b) y=
;
y (1 + x 2 )
c) y+ y 2senx = 0 ;
d) y= 1 + x + y 2 + xy 2 ;
e) y= (cos2 x ) (cos2 2 y ) ;
1

f) xy= (1 - y 2 ) 2 ;
g) y=

x - e- x
;
y + ey

h) y=

ax + b
, onde a, b, c, d so constantes;
cx + d

i) y=

ay + b
, onde a, b, c, d so constantes.
cy + d

2) Resolva as EDOs separveis a seguir, levando em conta as respectivas condies iniciais.



a) sen2 x + (cos 3 y ) y= 0 , com y = ;
2 3
-x
b) x + e yy= 0 , com y (0) = 1 ;
2x
, com y (0) = -2 ;
y + x2 y
2x
d) y=
, com y (2) = 0 .
1+ 2y
c) y=

158

3) Verifique que a EDO


y=

y - 4x
x- y

no separvel, mas que se a varivel y for substituda por uma


y
nova varivel u = , ento ela se torna separvel nas variveis x e
x
u . Ache a soluo da EDO original lembrando que y funo de x
assim como u .

3.2 Comprimento de Arco de Curvas


Planas
Consideremos uma curva plana , dada pelo grfico da funo diferencivel com derivada contnua
y = y( x) ,
com x [a, b] . Vamos utilizar o clculo integral para calcularmos o comprimento de arco da curva y . Para iso, vamos primeiramente construir uma partio P , do intervalo [a, b] , da forma
a = t0 < t1 < t2 < < t N = b de tal forma que aproximemos a curva
por uma poligonal, cujos vrtices so (t0 , y (t0 )) , (t1 , y (t1 )) , (t2 , y (t2 )) ,
..., (t N , y (t N )) , conforme nos mostra a figura 3.1 abaixo.
y

a = t0 t1 t2 t3 . . .

tn 1 b = tn

Figura 3.1. Aproximao de uma curva por uma poligonal definida pela partio P ,
do intervalo [a , b] .

Para a partio P , o comprimento da poligonal que aproxima a curva dado por

159
N

L( , P ) = (ti - ti -1 ) 2 + ( y ( i ) - y (ti -1 )) 2 .
i =1

Pelo teorema do valor mdio, ainda podemos escrever a expresso


acima como
N

L( , P ) = (ti - ti -1 ) 2 + ( y( i )(ti - ti -1 )) 2 ,
i =1

onde i ]ti -1 , ti [ , isto possvel pois y = y ( x) possui derivada cont2


nua. Assim, colocando em evidncia o termo ( ti - ti -1 ) na raiz quadrada, podemos, ainda escrever
N

L( , P ) = 1 + ( y( i )) 2 (ti - ti -1 ) .
i =1

A idia principal que, quanto mais refinada for a partio, mais


prximo o comprimento da poligonal deve estar daquilo que se espera que seja o comprimento de arco da curva. Assim, o comprimento de arco da curva ser obtido aplicando-se o limite no qual
o nmero de vrtices das poligonais tende a infinito, e o comprimento dos segmentos de cada poligonal tende a 0 . Isso equivale
a dizer que obtemos o comprimento de arco da curva tomando o
limite no qual o comprimento do maior intervalo da partio tende
a zero, isto P 0 , assim
N

L( ) = lim L( , P ) = lim 1 + ( y( i )) 2 (ti - ti -1 ) ,


| p| 0

| p| 0

i =1

mas este limite , por definio, exatamente igual integral sobre o


intervalo, ou seja,
b

L( ) = 1 + ( y( x )) 2 dx .
a

A existncia desta integral est garantida pelo fato de y '( x) ser uma
funo contnua. Essa frmula pode ser considerada como a definio do comprimento de arco de uma curva.
Definio 3.4. O comprimento de arco de uma curva diferencivel
, definida pelo grfico da funo diferencivel y = y ( x) , cuja derivada uma funo contnua, com x [a, b] , dado pela expresso
b

L( ) = 1 + ( y( x)) 2 dx .
a

(3.3)

160

Vamos tratar alguns exemplos conhecidos, e outros no to conhecidos do leitor, de comprimentos de arco de curvas.
Exerccio resolvido 3.4. Encontre o comprimento de arco da parbola y = x 2 , no intervalo [0, X ] , para algum X > 0 .
Soluo: Utilizando a frmula (3.33), temos
X

L = 1 + (2 x ) 2 dx = 1 + 4 x 2 dx .
tg
, teremos os limites de integra2
sec 2

=
arctg2X

=
0
o entre
e
, temos tambm x () =
e
2
1 + 4 x 2 = 1 + tg 2 = sec . Assim, a integral L acima pode ser escrita como
arctg 2 X
1
sec3 d .
L=

2 0

Efetuando a substituio x =

A integral de sec3 foi resolvida no exemplo 2.8, portanto, temos


b

1
b
3
sec
sec
tg
|
sec d .

a
a

2
a

Para o clculo da integral de sec , relembremos o truque padro:

com
b

sec + tg
sec 2 + sectg
d =
d
sec + tg
sec + tg
a

de

substituio

sec d =
a

sec d = sec
sec b + tg b

du
= ln u
u
sec a + tg a

variveis
sec b + tg b
sec a + tg a

u = sec + tg ,

obtemos

= ln sec b + tg b - ln sec a + tg a .

Em nosso caso, os limites de integrao so a = 0 e b = arctg2 X .


Com esses resultados em mos, o leitor pode facilmente concluir que
o comprimento de arco, L , ser dado por
L=

1
2 X 1 + 4 X 2 + ln 2 X + 1 + 4 X 2
4

)) .

161

Exerccio resolvido 3.5. Calcule o comprimento da circunferncia


x2 + y2 = r2 .
Soluo: Note que no podemos escrever toda a circunferncia como
o grfico de uma nica funo. Mas as semicircunferncias superior
e inferior podem, respectivamente, ser escritas como os grficos das
funes:
y1 = r 2 - x 2 , e y2 = - r 2 - x 2 ,
para x [ - r, r ] , e cujas derivadas so, respectivamente,
y1= -

x
2

r -x

, e y 2 =

x
2

r - x2

Como o comprimento da circunferncia igual soma dos comprimentos das semicircunferncias, temos que
L=

1 + ( y1) 2 dx +

-r

1 + ( y2) 2 dx = 2

-r

-r

r
2

r - x2

dx .

Essa integral pode ser resolvida com a substituio de variveis

, os limites de integrao so = e = , e
2
2
x() = rcos . Tambm temos que r 2 - x 2 = r 2 - r 2sen 2 = rcos .
Assim, a integral resulta em
,

que o resultado conhecido do comprimento da circunferncia.


Exerccio resolvido 3.6. Calcule o comprimento de arco da curva
y = e x , entre x = 0 e x = 1 .
Soluo: Utilizando a frmula do comprimento de arco, temos a
integral
1

L = 1 + e 2 x dx ,
0

162
que pode ser resolvida facilmente utilizando-se a substituio de varivel u 2 = 1 + e 2 x . Nessa substituio, os nossos limites de integrao
u
. Assim,
so u = 2 e u = 1 + e 2 , tambm temos que x(u ) = 2
u -1
a integral pode ser escrita como
L=

1+ e2

u2
du =
u2 - 1

1+ e2

du +

1+ e2

du
=
2
u -1

1+ e2

1
du +
2

1+ e2

1
1

du ,
u -1 u +1

onde, na ltima igualdade, foi utilizado o mtodo das fraes parciais.


O clculo das integrais finalmente resulta em
1 ( 1 + e 2 - 1)( 2 + 1)
L = 1 + e 2 - 2 + ln
,
2 ( 1 + e 2 + 1)( 2 - 1)
que ainda pode ser escrito na forma abaixo, conforme o leitor poder
verificar facilmente com uma simples manipulao algbrica:
L = 1 + e 2 - 2 + ln( 1 + e 2 - 1) - 1 - ln( 2 - 1) .

3.2.1 Exerccios
1) Calcule os comprimentos de arco das seguintes curvas nos intervalos indicados:
x3 1
a) y =
, para 1 x 2 .
+
6 2x
x 2 ln x
b) y =
, para 2 x 4 .
2
4
c) y = ln(cos x ) , para 0 x
d) y = cosh x =

e x + e- x
, para 0 x 1 .
2

e) y = x , para 0 x 4 (Dica: Tente fazer x = x ( y ) ).

3.3 Slidos e Superfcies de Revoluo


Nesta seo, vamos aprender como utilizar o clculo integral para
obtermos o volume de um slido de revoluo e a rea de uma superfcie de revoluo. Primeiramente, precisamos delimitar bem nosso objeto de estudo. Considere o grfico de uma funo, y = f ( x ) ,
contnua e no negativa, sendo o plano bidimensional x, y , um dos

163

planos no sistema cartesiano x, y e z , e a coordenada x definida


em um intervalo [a, b] . Considere tambm a regio do plano x, y
delimitada pelo eixo x , pelas retas x = a e x = b , e pelo grfico da
funo, conforme indicado na figura 3.2 a seguir.
y

y = f ( x)

z
Figura 3.2: O grfico da funo y = f ( x) no espao tridimensional e a regio do plano
sob este grfico e acima do eixo x .

x, y

Para construirmos o slido de revoluo, temos que efetuar a rotao da regio plana ao redor do eixo x . O slido de revoluo est
mostrado na figura 3.3.
y

z
Figura 3.3: Slido de revoluo construdo a partir do grfico da funo y = f ( x ) .

164
Vamos denotar o slido de revoluo assim gerado pela letra ,
e a superfcie de revoluo, pela letra . Esta superfcie gerada
apenas pela rotao da curva y = f ( x) , e que portanto no inclui os
dois crculos nas extremidades. Nesta seo, vamos mostrar como
se resolve o problema de calcular o volume de um slido de revoluo e a rea de uma superfcie de revoluo . Para o clculo
do volume, existem dois mtodos alternativos: no primeiro mtodo,
a integral obtida como o limite da soma dos volumes de cilindros
obtidos por aproximao atravs de cortes transversais de determinados por parties do intervalo [a, b] do domnio da funo f
que est sendo considerado. No segundo mtodo, considera-se a integral obtida como o limite das somas dos volumes de cascas cilndricas, obtidas atravs da rotao de retngulos em torno do eixo y .

3.3.1 Mtodo dos Discos


Vamos ilustrar o primeiro mtodo para o clculo de volumes. Seja
f : [a, b] uma funo contnua no negativa. Ao rotacionarmos a
regio sob o grfico y = f ( x ) , ao redor do eixo x , obtemos o slido ,
conforme ilustrado na figura 3.3. Considere, agora, uma partio P
do intervalo [a, b] , ou seja, nmeros reais t0 = a < t1 < t2 < < t N = b .
O volume de pode ser aproximado pela soma dos volumes dos
cilindros de raio f (ti ) , para 1 i N e de altura ti - ti -1 , determinados por esta partio, conforme a figura 3.4 abaixo.
y

ti 1

ti

z
Figura 3.4: Aproximao do volume do slido de revoluo pela soma dos volumes dos cilindros determinados pela partio P : t0 = a < t1 < t2 < < t N = b .

165
O volume de cada cilindro dado por
Vi = f (ti ) 2 (ti - ti -1 ) ,

assim, a soma de todos os volumes dos cilindros determinados pela


partio P dada por
N

i =1

i =1

VP = Vi = f (ti ) 2 (ti - ti -1 ) .

Finalmente, o volume do slido dado pelo limite quando


| P | 0 sobre todas as parties do intervalo [a, b] destas somas
parciais, ou seja,
N

V () = lim VP = lim f (ti ) (ti - ti -1 ) = f ( x) 2 dx .


P 0

P 0

i =1

Novamente, essa frmula pode ser tomada como a definio do volume de um slido de revoluo.
Definio 3.5: O Volume de um slido de revoluo gerado pelo
grfico da funo f : [a, b] , ao redor do eixo x , dado pela
integral
b

V = ( f ( x ) ) dx .
2

(3.4)

Antes de iniciarmos os exemplos resolvidos devemos ainda fazer


trs observaes:

y = g ( x)

A primeira que podemos tambm calcular o volume de um slido


de revoluo ao redor do eixo y , para o qual teremos que integrar em
relao varivel y , mas para isso devemos ter o
grfico de uma funo no negativa x = g ( y ) e a
y = f ( x)
regio delimitada pelo grfico da funo, pelo eixo
y e pelas retas y = a e y = b no plano x, y ; caso
contrrio, teremos que dividir a curva em partes.

Figura 3.5: Regio delimitada pelos grficos das funes f


e g no intervalo [a , b] .

A segunda observao diz respeito a slidos


de revoluo gerados por regies que so delimitadas no plano por dois grficos de funes,
y = f ( x ) e y = g ( x ) , de modo que a regio esteja situada acima do eixo x como nos mostra a
figura 3.5 ao lado:

166

Nesse caso, devemos calcular a diferena entre os volumes gerados


separadamente pelos grficos, tomando, obviamente, o cuidado de
observar qual das duas funes assume o maior valor. Isso pode
ser feito de duas maneiras: a primeira forma consiste em separar o
intervalo em partes (no caso da figura 3.5 acima h duas partes distintas, uma onde os valores f ( x) so maiores que os valores g ( x)
e outra em que ocorre o contrrio. A segunda maneira tomarmos
o valor absoluto da diferena entre as duas funes, ou melhor, dos
quadrados delas, na frmula do volume. Assim, a expresso para o
clculo do volume do slido de revoluo delimitado pelos grficos
das funes f e g se escreve como
b

V = ( f ( x) ) - ( g ( x) ) dx .
2

A terceira e ltima observao que tambm possvel calcular


o volume do slido de revoluo gerado pelo grfico da funo
y = f ( x ) girando-se ao redor de qualquer eixo horizontal y = y0 .
Basta transladarmos este eixo horizontal at o eixo x , que tem equao y = 0 ; para isso, teremos que transladar de igual modo o grfico
da funo f , subtraindo o valor y0 em todos os pontos, com isso,
teremos que o volume gerado pela funo f girada ao redor do
eixo horizontal y = y0 ser igual ao volume gerado pelo grfico da
funo f , dado
por f ( x ) = f ( x ) - y0 , girado ao redor do eixo x .
Exerccio resolvido 3.7: Calcular o volume da esfera slida de raio
r centrado na origem.
Soluo: A esfera slida a regio do espao dada pela inequao
x2 + y 2 + z 2 r 2 ,
e delimitada pela superfcie da esfera cuja equao
x2 + y 2 + z 2 = r 2
e pode ser obtida a partir do grfico da funo
y = f ( x) = r 2 - x2 ,
definida no intervalo [ - r, r ] , rotacionando a regio entre o grfico
de f e o eixo x ao redor do eixo x , como nos mostram as figuras
3.6 e 3.7 abaixo.

167
y

Figura 3.6: Regio do plano

x, y

y = f ( x) = r 2 - x2 .

sob o grfico da funo

z
2

Figura 3.7: Esfera slida, cuja superfcie a esfera x + y + z = r , gerada pela rotao do
grfico acima ao redor do eixo x .

Utilizando a frmula (3.4) para volumes de slidos de revoluo para


a funo f ( x) = r 2 - x 2 , temos que o volume da esfera de revoluo ser dado por
Novamente, o cone a
superfcie que delimita
o slido, mas tambm
comum dizer volume
do cone, para denotar o
volume do slido delimitado
pelo cone.

x3
2r 3 4 r 3
V = (r - x )dx = r 2 x - = 2r 3 .
=
3 -r
3
3

-r
2

Exerccio resolvido 3.8. Calcular o volume de um slido cnico de


raio da base igual a r e altura igual a h .

168
Soluo: Este cone pode ser obtido rotacionando ao redor do eixo x
o grfico da funo
r
y = f ( x) = - x + r ,
h
definida no intervalo [0, h ] . Para obtermos o slido, necessrio rotacionar a regio do plano sob o grfico dessa funo, conforme nos
mostram as figuras 3.8 e 3.9 abaixo.
y

y = f ( x)

h
Figura 3.8: Regio do plano

r
x, y , sob o grfico da funo y = f ( x ) = - x + r
h
entre x = 0 e x = h .

z
Figura 3.9: Slido de revoluo delimitado pelo cone de base circular de raio r e de
altura h .

Ento teremos que o volume do slido de revoluo ser


h

h
r 2 x2
r2x 2
rx

+ r dx
V = - + r dx = 2 - 2
h
h
h

0
0

169
h

r 2 x3 r 2 x 2
r 2 h
2
.
2 + r x =
h
3
3h
0
Novamente, o paraboloide
a superfcie que delimita o
slido, estritamente falando,
impreciso dizer volume
do paraboloide, porm
utiliza-se esta denominao
corriqueiramente.

Exerccio resolvido 3.9. Calcular o volume do paraboloide gerado pela rotao da regio do plano x, y sob o grfico da funo
f ( x ) = x ao redor do eixo x , a partir do ponto x = 0 at o ponto
x=h.
Soluo: As figuras 3.10 e 3.11 nos mostram a formao deste slido
de revoluo.
y

z
Figura 3.10: Regio do plano x, y sob o grfico da funo y = f ( x ) =

x.

z
Figura 3.11: Paraboloide de revoluo gerado pela regio acima entre os pontos x = 0 e x = h .

170
Novamente, aplicando a frmula (3.4), para a funo f ( x ) = x ,
teremos
h
h 2
V paraboloide = xdx =
.
2
0

3.3.1 Exerccios
1) Calcule o volume do elipsoide de revoluo gerado pela elipse
x2 y2
+
= 1 , rodada ao redor do eixo x . (Obs: note que voc s
a 2 b2
precisa rotacionar, de fato, a parte superior da elipse, que o
grfico de uma funo.)
2) Calcule o volume do toro de revoluo gerado pela rotao do
crculo, delimitado pela circunferncia ( x - a ) 2 + y 2 = b 2 , com
a > b > 0 , ao redor do eixo y . (Obs: note que aqui voc ter que

considerar duas funes: x = f1 ( y ) e x = f 2 ( y ) , que correspondem s semicircunferncias interna e externa. A integrao


tambm tem que ser feita na varivel y .)
3) Calcule o volume do slido de revoluo ao redor do eixo x
gerado pela rotao da regio compreendida entre a parbola
y = x 2 e pela reta y = x , para 0 x 1 .
4) Calcule o volume do slido de revoluo gerado pela mesma
figura do exerccio (3) acima girada ao redor do eixo horizontal
y = 2.
5) Calcule o volume do slido de revoluo conhecida como
Trombeta de Gabriel, que consiste na rotao ao redor do
1
eixo x da regio do plano sob o grfico da funo f ( x ) =
x
para x 1 . (Aqui, o leitor ter que efetuar uma integral imprpria de 1 a . Consulte o captulo 2 para mais detalhes sobre
integrais imprprias.)
6) Calcule o volume do slido obtido pela rotao ao redor do
eixo x da regio do plano compreendida entre o grfico da
funo y = sen x e o eixo x para x [0, ] .
7) Calcule o volume do slido de revoluo obtido pela rotao ao
redor do eixo x da regio sob o grfico de y = e - x para x 0 .

171

3.3.2 Mtodo das Cascas Cilndricas


O segundo mtodo de clculo de volumes de slidos de revoluo
atravs de cascas cilndricas. Esse mtodo ser til para calcularmos
o volume do slido de revoluo gerado pelo grfico de uma funo
y = f ( x ) girado agora em relao ao eixo y . Considere o grfico da
funo no negativa e contnua f : [a, b] com a 0 conforme
mostrado na figura 3.12 abaixo.
y

y = f ( x)

z
Figura 3.12: Grfico da funo y = f ( x) para

x [a , b] .

Ao rotacionarmos a regio sob este grfico e que est acima do eixo


x ao redor do eixo y , obtemos o slido de revoluo mostrado na
figura 3.13 abaixo.
y

y = f ( x)

x
z

Figura 3.13: Slido de revoluo gerado pelo grfico de y = f ( x ) , girado ao redor do eixo y .

172
A ideia dividirmos esse slido de revoluo em cascas cilndricas cujo eixo o eixo de rotao do slido em questo, o eixo y .
Para obtermos essas cascas cilndricas, definimos uma partio
P : a = t0 < t1 < < t N = b no intervalo [a, b] de definio da funo
f . Uma casca ciclndrica obtida ao rotacionarmos o retngulo
cuja base est sobre o intervalo [ti -1 , ti ] e que tem altura igual a f (ti ) .
A espessura desta casca cilndrica, portanto, dada pela diferena
ti - ti -1 e sua altura dada por f (ti ) , conforme nos mostra a figura
3.14 abaixo.
y

x
z
Figura 3.14: Cascas cilndricas para o slido de revoluo gerado pela rotao da regio no
plano x, y sob o grfico de y = f ( x ) , girado ao redor do eixo y .

Assim, o volume do slido de revoluo pode ser calculado pelo


limite das somas dos volumes das cascas cilndricas quando as parties ficam arbitrariamente finas. Para a partio P , o volume de
cada casca cilndrica a diferena entre o volume do cilindro de raio
ti e altura f (ti ) e o volume do cilindro de raio ti -1 e altura f (ti ) ,
logo
Vi = ( ti2 - ti2-1 ) f (ti ) ,
E a soma de todos os volumes das cascas cilndricas associadas a
essa partio pode ser escrita como
N
N
t +t
VP = (ti2 - ti2-1 ) f (ti ) = 2 i i -1 (ti - ti -1 ) f (ti ) ,
2
i =1
i =1

Aplicando o limite | P | 0 , no qual as parties ficam cada vez mais


finas, podemos ver que a mdia aritmtica

173

ti + ti -1

2
pode ser aproximada pelo valor ti . Assim, obtemos a integral
b

V () = 2 xf ( x)dx = 2 lim ti f (ti )(ti - ti -1 ) .


P 0

i =1

Essa integral pode ser tomada como a definio do volume do slido


de revoluo obtido.
Definio 3.6. O volume de um slido obtido pela rotao ao
redor do eixo y da regio entre o grfico da funo contnua no negativa y = f ( x ) definida no intervalo x [a, b] , com a 0 , e o eixo
x dado por
b

V () = 2 xf ( x)dx .

(3.5)

Exerccio resolvido 3.10. Encontre o volume do slido obtido ao rotacionarmos ao redor do eixo y a regio compreendida entre o grfico da funo y = x ( x - 1) 2 e o eixo x .

Como nos outros exemplos,


o toro a superfcie
que delimita o slido. A
expresso volume do toro
inexata, porm utilizada
normalmente.
Note que a funo f 2
negativa, ento o volume
do slido de revoluo
obtido pela rotao da
regio compreendida entre
seu grfico e o eixo x
igual ao volume gerado
pela funo que a cada
x [ R - r , R + r ] associa o
valor - y , ou seja, o volume
determinado a partir da
funo f1 .

Soluo. Antes de qualquer coisa, observemos que quando nos referimos regio estamos falando de uma regio limitada. A regio
sob o grfico da funo f ( x ) = x ( x - 1) 2 est entre os pontos que o
grfico cruza ou toca o eixo x , isto , suas razes, que so exatamente x = 0 e x = 1 , sendo assim, nosso intervalo de integrao ser o
intervalo [0,1] . Portanto, utilizando a frmula (3.5), temos
1

x5
x 4 x3

V = 2 x ( x - 1) dx = 2 ( x - 2 x + x )dx = 2 - 2 + = .
4 3 0 15
5
0
0
2

Exerccio resolvido 3.11. Calcular o volume do toro, obtido ao rotacionarmos ao redor do eixo y o crculo cuja circunferncia
( x - R ) 2 + y 2 = r 2 , com 0 < r < R .
Soluo. Essa Circunferncia pode ser vista como a unio dos grficos das
funes y = f1 ( x) = r 2 - ( x - R) 2 e y = f 2 ( x) = - r 2 - ( x - R) 2 ,
assim, podemos utilizar a simetria da figura e calcular o volume do
toro como sendo o dobro do volume do slido de revoluo gerado
pelo grfico mostrado na figura 3.15 a seguir

174

Rr

R+r

Figura 3.15: Regio sob o grfico da funo

y = r 2 - ( x - R)2 .

O intervalo de integrao tambm fica explcito na figura acima, logo


podemos calcular o volume do toro:

I = 4

R+r

R r 2 - ( x - R ) 2 dx .

R-r

Seja a integral
I = 4

R+r

R r 2 - ( x - R) 2 dx .

R-r

Observe que, somando e subtraindo a integral I na expresso do


volume V , obtemos
V = 4

R+r

( x - R)

R-r

r - ( x - R) dx + 4 R

R+r

r 2 - ( x - R) 2 dx .

R-r

Na primeira integral, efetuamos a substituio u = x - R , e na segunda integral efetuamos a substituio x - R = rcos . Assim, o leitor capaz de verificar facilmente que obtemos
r

-r

V = 4 u r 2 - u 2 du + 4 Rr 2 sen 2 d .
A primeira integral na expresso acima identicamente nula, pois se
trata de uma integral de funo mpar em um intervalo simtrico no
que diz respeito origem. A segunda integral pode ser resolvida fa1
cilmente, lembrando-se que sen 2 = (1 - cos2) , assim a integral
2

resulta em . Portanto, o volume do toro igual a


2
V = 2 2 Rr 2 .

175

Note que o volume do toro igual ao produto do comprimento da


circunferncia de raio R pela rea do crculo de raio r .
Exerccio resolvido 3.12. Calcule o volume do slido gerado pela
rotao ao redor do eixo y da regio delimitada pelas parbolas
y = x2 e y = 1 - x2 .
Soluo. A regio em questo est descrita na figura 3.16 ao lado
y

x0

x0

Figura 3.16: Regio delimitada pelas parbolas y = x e y = 1 - x .

Primeiramente, necessrio que se determine o ponto x0 de interseco entre as duas parbolas. Isso simples, pois x02 = 1 - x02 , o que
1
2
nos d x02 = ou ainda x0 =
. Note que o volume do slido de
2
2
revoluo ser dado somente pela regio considerada pelo intervalo
[0, x0 ] , assim temos que a expresso do volume se escreve como
2
2

2
2

2
2

V = 2 x((1 - x 2 ) - x 2 )dx = 2 x(1 - 2 x 2 )dx = 2 ( x - 2 x3 )dx ,


de onde resulta
2
2

x
x

V = 2 - 2 = .
4 0
4
2
2

176

3.3.2 Exerccios
1) Calcule o volume da esfera de centro na origem e raio r , utilizando o mtodo das cascas cilndricas.
2) Calcule, utilizando o mtodo das cascas cilndricas, o volume
de um cone vertical reto de base circular com raio r e com
altura h (sugesto: coloque os eixos coordenados de maneira
apropriada e compare o seu resultado com o exerccio resolvido 3.8).
3) Considere dois slidos esfricos de raios R1 e R2 , respectivamente (suponha, sem perda de generalidade, que R2 > R1 . Dessas duas esferas, retire dois cilindros slidos de raios r1 e r2 ,
respectivamente, de forma que os anis resultantes tenham a
mesma altura h , conforme ilustrado na 3.17 abaixo ( fcil ver
que neste caso r2 > r1 ). Qual dos dois anis tem maior volume?

Figura 3.17: Figura para o exerccio 3.

4) Calcule o volume do slido de revoluo gerado pela rotao


ao redor do eixo y da regio sob a curva y = sen( x 2 ) e acima
do eixo x , para x [0, ] .
5) Calcule o volume do slido de revoluo gerado pela rotao
ao redor do eixo y da regio entre as curvas y = x e y = 2 x .
6) Utilize o mtodo das cascas cilndricas para obter o volume do
slido de revoluo obtido pela rotao ao redor do eixo y da
regio entre o grfico da funo y = e - x e o eixo x , para x 0 .
7) Utilize o mtodo das cascas cilndricas para obter o volume
do slido de revoluo obtido pela rotao ao redor do eixo y
da regio entre o grfico da funo y = sen x e o eixo x , para
y = r sen .

177

3.3.3 reas de Superfcies de Revoluo


Para finalizarmos esta seo, vamos deduzir o clculo da rea de
uma superfcie de revoluo. Como o leitor j deve ter notado pelo
processo de obteno de frmulas para o clculo de volumes de slidos de revoluo, sempre utilizamos aproximaes com o auxlio
de parties de um intervalo, posteriormente tomamos o limite e
obtemos uma integral ao longo do segmento. Adotaremos o mesmo
procedimento no clculo da rea de uma superfcie de revoluo.
Considere a rotao ao redor do eixo x do grfico de uma funo diferencivel y = f ( x ) , definida para x [a, b] , satisfazendo f ( x ) 0 ,
e cuja derivada uma funo contnua neste intervalo. Tomando-se
uma partio P : a = x0 < x1 < < xN = b , podemos aproximar a rea
da superfcie pela soma das reas laterais dos troncos de cones determinados pela rotao dos segmentos unindo os pontos ( xi -1 , f ( xi -1 ) )
e ( xi , f ( xi ) ) , conforme ilustrado na figura 3.18.
y

x
z
Figura 3.18: Uma superfcie de revoluo aproximada por troncos de cones determinados pela
partio P do segmento [a , b] .

De incio, temos uma tarefa de geometria elementar, a saber, calcular a rea lateral de um tronco de cone de uma maneira apropriada.
Primeiramente, vamos calcular a rea lateral de um cone circular
reto de raio da base igual a r e comprimento da geratriz igual a l .
Se planificarmos o cone, cortando-o ao longo de uma geratriz, obtemos um setor circular de raio l e ngulo central = AB conforme
ilustrado na figura 3.19.

178
b
O


2 r

a
Figura 3.19: Planificao da superfcie lateral de um cone circular reto.

igual a
Como sabemos que o comprimento do arco AB
, mas
2 r
tambm igual a l , temos ento que =
. A rea do setor cirl
cular, que a rea lateral do cone, resulta em
1
1 2 r
A = l 2 = l 2
= rl .
2
2 l

Agora, considere um tronco de cone de raio menor r1 , raio maior r2


e geratriz l . Esse tronco pode ser visto como a diferena entre um
cone de raio r2 e geratriz l + l1 e um cone menor de raio r1 e geratriz
l1 , conforme ilustrado na figura 3.20.

1
r1

1 +

r2

Figura 3.20: Tronco de um cone de raio menor

r1 , raio maior r2

e geratriz

l.

A rea da superfcie lateral do tronco de cone se escreve como


A = r2 (l + l1 ) - r1l1 = r2l + (r2 - r1 )l1 .

179
Podemos eliminar l1 pela semelhana de tringulos na figura 3.20.
O leitor pode verificar facilmente que
l1 l + l1
=
( r2 - r1 )l1 = rl
1 .
r1
r2
Substituindo na expresso da rea, temos finalmente que a rea de
um tronco de cone dada por
A = (r1 + r2 )l = 2 rl .
onde r =

r1 + r2
o raio mdio.
2

Utilizando a expresso obtida, podemos escrever a soma das


reas laterais dos troncos de cones determinados pela partio
P do intervalo [a, b] e pelo grfico da funo y = f ( x ) . A geratriz l dada pelo comprimento do segmento unindo os pontos pi -1 = ( xi -1 , f ( xi -1 ) ) e pi = ( xi , f ( xi ) ) , e o raio mdio dado por
1
1
ri = ( yi -1 + yi ) = ( f ( xi -1 ) + f ( xi )) . Assim, a rea total relativa
2
2
partio P ser dada por

onde o ponto i , cuja existncia est garantida pelo teorema do valor


mdio, tal que xi -1 < i < xi , e a aproximao da ltima expresso
pode ser feita, pois a funo f contnua. Quando a partio bem
fina, os subintervalos ficam com comprimentos muito pequenos e os
valores da funo nas extremidades so muito prximos, logo o erro
pequeno ao considerarmos f constante em cada um dos subintervalos. Finalmente, tomando o limite | P | 0 , que o mesmo que o
limite N , obtemos a seguinte frmula para a rea
b

A = 2 f ( x) 1 + ( f ( x)) 2 dx .
a

Novamente, essa frmula pode ser tomada como a definio da rea


da superfcie de revoluo.

180
Definio 3.7. A rea da superfcie de revoluo gerada pela rotao ao redor do eixo x do grfico da funo diferencivel y = f ( x )
definida no intervalo [a, b] , cuja derivada uma funo contnua e
com f ( x ) 0 , igual a
b

A = 2 f ( x) 1 + ( f ( x)) 2 dx .

(3.6)

Exerccio resolvido 3.13. Determine a rea da esfera de centro na


origem e raio r .
Soluo. A esfera x 2 + y 2 + z 2 = r 2 gerada pela rotao ao redor
do eixo x do grfico da funo f ( x ) = r 2 - x 2 , para x [ - r, r ] . A
derivada de f pode ser facilmente calculada, resultando em
-x
f ( x ) =
.
r2 - x2
Assim, temos que
1 + ( f ( x )) 2 = 1 +

x2
r2
=
,
r2 - x2 r2 - x2

o que, finalmente, utilizando a frmula (3.6), nos d a expresso para


a rea da esfera:
r

A = 2 r 2 - x2
-r

r
r 2 - x2

dx = 2 r dx = 4 r 2 .
-r

Exerccio resolvido 3.14. Determine a rea da superfcie do toro obtido


pela rotao ao redor do eixo y da circunferncia ( x - R ) 2 + y 2 = r 2 ,
com 0 < r < R .
Soluo. Para resolvermos esse problema, temos que adaptar a expresso da rea de superfcies de revoluo para funes do tipo x = f ( y ) ,
e efetuarmos a integrao na varivel y . Na verdade, teremos dois
grficos, a saber, f1 ( y ) = R + r 2 - y 2 e f 2 ( y ) = R - r 2 - y 2 , com
y [ - r, r ] . As derivadas so, respectivamente,
f1( y ) =

-y
2

r -y

e f 2 ( y ) =

y
2

r - y2

Assim, a rea do toro ser a soma de duas integrais, resultando em

181
r

A = 2 R + r - y
-r
r

r
2

r -y

dy + 2 R - r 2 - y 2
-r

r
2

r - y2

dy = t


2 2

dy dy
r cos
dd
rcos
2
= 4=4Rr
4 42Rr
Rr
Rr d d =

Rr
- r r 2 -2y 2 =2 44RrRr
r 2 -2r 2 sin2 2 2= 44Rr

r y
r
r
sen

r
2

2 2

Na penltima igualdade, efetuamos a mudana de variveis y = r sin .


Note que a rea do toro o produto do comprimento da circunferncia de raio R pelo comprimento da circunferncia de raio r .
Exerccio resolvido 3.15. Calcule a rea da superfcie de revoluo obtida rotacionando-se ao redor do eixo x o grfico da funo
y = f ( x ) = x , para x [1, 4] .
Soluo. A derivada da funo f facilmente obtida, resultando em
1
4x + 1
f ( x ) =
. Assim, tambm obtemos 1 + ( f ( x )) 2 =
,e
4x
2 x
finalmente a rea da superfcie
.

3.3.3 Exerccios
1) Na frmula (3.6), consideramos somente o caso f ( x ) 0 , mostre que, quando f no , necessariamente positiva, a frmula
correta para a rea de superfcies de revoluo fica
b

A = 2 f ( x) 1 + ( f ( x)) 2 dx .
a

2) Encontre a rea da superfcie de revoluo gerada pela rotao


ao redor do eixo x do grfico da funo f ( x ) = e - x para x 0 .
3) Verifique que a trombeta de Gabriel, que gerada pela rotao
1
ao redor do eixo x do grfico da funo f ( x ) = para x 1 ,
x
muito embora o slido de revoluo associado tenha volume
finito (como o leitor j calculou em um exerccio anterior), possui rea infinita.
4) Encontre a rea da superfcie obtida pela rotao ao redor do
eixo y da curva y = 3 x para y [1, 2] .

182
5) Encontre a rea do elipsoide obtido pela rotao ao redor do
eixo x da elipse 2 x 2 + y 2 = 1 .

3.4 Centro de massa de regies planas


Em Fsica, o estudo de sistemas complexos exige algumas simplificaes de forma a tornar os problemas tratveis do ponto de vista matemtico. Para o estudo de corpos extensos ou para o estudo de um sistema de muitas partculas, utiliza-se o recurso do clculo do centro
de massa, ou centroide do sistema. O centroide um ponto que tem
a propriedade simplificadora na qual toda a massa do sistema pode
ser vista como se estivesse concentrada naquele ponto. Assim, toda a
fsica do sistema inicial pode ser estudada em duas partes, em geral
mais simples: a primeira o comportamento do centro de massa em
relao a algum referencial pr-fixado, e a segunda o comportamento das partculas do sistema em relao ao centro de massa.
Para o clculo do centro de massa de um sistema com n partculas
no plano, com massas m1 , m2 , , mn e coordenadas, respectivamente, ( x1 , y1 ),( x2 , y2 ), ,( xn , yn ) , devemos fazer a mdia ponderada das
coordenadas em relao s massas. Assim, as coordenadas do centro de massa sero ( x, y ) , com
x=

m1 x1 + + mn xn
m y + + mn yn
e y= 1 1
.
m1 + + mn
m1 + + mn

Nosso objetivo nesta seo estabelecer frmulas para o clculo das


coordenadas do centro de massa de certas regies planas delimitadas por curvas contnuas. O centro de massa dessas regies tambm ser o centro de gravidade, ou de equilbrio, dessas figuras, ou
seja, se pendurarmos a figura plana exatamente por este ponto, a
figura permanecer em equilbrio. Uma outra forma de ver o centro de massa ainda dizer que toda reta que passe pelo centro de
massa divide a figura em duas figuras de mesma massa. Tendo em
vista as ferramentas matemticas disponveis ao leitor at o momento, vamos delimitar um pouco mais o problema: vamos considerar
apenas regies delimitadas por curvas que sejam grficos de funes reais contnuas de uma varivel y = f ( x ) para x em algum
intervalo [a, b] .
Outra restrio no tratamento do problema ser no sentido de como
vamos considerar as massas dos sistemas. O fato que, ao passarmos de um sistema discreto de partculas, no qual as massas esto

183
localizadas em pontos, para um sistema contnuo, como o caso das
regies planas, precisamos dizer como a massa do sistema est distribuda. No caso de regies planas isso feito atravs da densidade
superficial. Vamos tentar entender este conceito. O leitor deve estar
acostumado com o conceito de densidade como a razo entre a
massa e o volume. Ento, a densidade superficial deve ser a razo entre a massa total e a rea da regio plana considerada. Essa
ser a simplificao que utilizaremos nesta seo. Para que fssemos mais precisos, teramos que considerar a densidade superficial
como uma funo
que pudesse variar ponto a ponto; isso
descreveria, por exemplo, um material que no estivesse uniformemente distribudo sobre a regio, mas o clculo para regies com
densidade superficial varivel s poder ser feito com o auxlio de
integrais duplas, que sero abordadas no Clculo 3. Assim, todo o
nosso tratamento ser para regies de densidade superficial constante = ( x, y ) = constante .
Considere, portanto, uma regio compreendida entre os grficos
das funes y = f ( x ) e y = g ( x ) , para x [a, b] , com densidade superficial constante . Por simplicidade, podemos considerar ambas
as funes positivas, como nos indica a figura 3.21.
y
y = f ( x)

y = g ( x)

Figura 3.21: Regio plana da qual sero calculadas as coordenadas do centro de massa, ou
centroide.

Um primeiro clculo importante o clculo da massa total da regio, que igual ao produto da densidade pela rea da regio. Assim, temos
b

M = A = f ( x) - g ( x) dx .
a

184
A razo de termos o valor absoluto na integral acima simplesmente para lidar com casos nos quais os grficos das duas funes se
cruzem no intervalo de integrao. A massa total sempre aparecer
no denominador das duas coordenadas do centro de massa da regio. Na verdade, como veremos, o denominador ser dado apenas
pela rea da regio. Isso se deve ao fato de a densidade ser constante, o que nos permite cancel-la na expresso do centro de massa.
Seja, agora, uma partio P : a = t0 < t1 < < t N = b , do intervalo
[a, b] , dividamos a regio da figura 3.21 em retngulos e olhemos
em detalhes o i-simo retngulo, cuja base o segmento [ti -1 , ti ] e a
altura o segmento de comprimento f (ti ) - g (ti ) , conforme a figura 3.22.
y

ti1 ti

Figura 3.22: Partio para o clculo do centro de massa.

Da geometria elementar, sabemos que o centroide de um retngulo


se encontra no ponto de interseco das duas diagonais, que coincide com o ponto cujas coordenadas so as coordenadas dos pontos
mdios dos lados. Assim, as coordenadas do centro de massa do
i-simo retngulo so
t +t
f (ti ) + g (ti )
xi = i i -1 e yi =
,
2
2

e a massa desse mesmo retngulo dada por

mi = f (ti ) - g (ti ) (ti - ti -1 ) = Ai ,


onde ai a rea do i-simo retngulo da partio.

185
As coordenadas do centride da regio so dadas pelas mdias aritmticas ponderadas das coordenadas dos centrides de cada retngulo, considerando-se que toda a massa dos mesmos esteja concentrada sobre seus centros de massa. Temos, ento, que associadas a
essa partio as coordenadas aproximadas do centro de massa sero

1 N ti + ti -1

f (ti ) - g (ti ) (ti - ti -1 ) =

mi i=1 2

xp =

N ti + ti -1


f (ti ) - g (ti ) (ti - ti -1 )

Ai i =1 2
1

1 N f (ti ) + g (ti )

f (ti ) - g (ti ) (ti - ti -1 ) =
2

mi i =1

yp =

1
mi
i

N ( f (ti )) 2 - ( g (ti )) 2

(ti - ti -1 ) =

2
i =1

2
2
( f (ti )) - ( g (ti )) (ti - ti -1 )
2 Ai i =1

A razo de o denominador ficar somente dependente das reas Ai ,


nas expresses do centro de massa, que
, assim a densidade se cancela na expresso. Finalmente, ao tomarmos o limite em
que as parties ficam arbitrariamente finas, temos que as coordenadas do centro de massa ficam
b
b
1
1
x = x f ( x ) - g ( x ) dx e y =
( f ( x )) 2 - ( g ( x )) 2 dx .

Aa
2A a
Essas frmulas podem ser tomadas como a definio das coordenadas do centroide da regio plana.
Definio 3.8: As coordenadas do centro de massa da regio plana
delimitada pelas curvas y = f ( x ) e y = g ( x ) , para f e g funes
positivas e x [a, b] , so iguais a
x=

1
1
x f ( x ) - g ( x ) dx e y =
( f ( x )) 2 - ( g ( x )) 2 dx . (3.7)

Aa
2A a

onde A a rea da regio.

186

Exerccio resolvido 3.16. Calcular as coordenadas do centro de massa


da regio delimitada pela semicircunferncia x 2 + y 2 = r 2 , com y 0 .
Soluo: Precisamos primeiramente calcular a rea da regio Neste
caso, o resultado pode ser obtido com matemtica elementar, ou enr 2
to o leitor pode utilizar o clculo integral, resultando em A =
.
2
As funes em questo so f ( x ) = r 2 - x 2 e g ( x ) = 0 , com
x [ - r, r ] . Assim, temos as coordenadas do centro de massa como
x=

2
r 2

r 2 - x 2 dx = 0 ,

-r

pois a integral de uma funo mpar em um intervalo simtrico, e


1
y= 2
r

1 2
x3
4r
.
=
(
r
x
)
dx
r
x
=
2
- r
r
3 - r 3
2

Exerccio resolvido 3.17. Calcular as coordenadas do centro de massa da regio sob a parbola y = 1 - x 2 com y 0 .
Soluo: Aqui as funes so f ( x ) = 1 - x 2 e g ( x ) = 0 , para
x [ -1,1] . Calculemos, em primeiro lugar, a rea da regio:
1

x3
4
A = (1 - x ) dx = x - = .
3 -1 3

-1
2

Agora, calculando as coordenadas do centro de massa, temos


1

3
x = x (1 - x 2 ) dx = 0 ,
4 -1
pela mesma razo do exerccio resolvido anterior, e
1

1
1
2
3
3
3
2 x3 x5
2
+ = .
y = (1 - x 2 ) dx = (1 - 2 x 2 + x 4 ) dx = x 8 -1
8 -1
8
3
5 -1 5

Exerccio resolvido 3.18. Encontre as coordenadas do centro de massa da regio compreendida entre a reta y = x e a parbola y = x 2 .
Soluo: Aqui as funes so f ( x ) = x e g ( x ) = x 2 , para. Note que
neste intervalo f ( x ) g ( x ) 0 . Primeiramente, o clculo da rea da
regio resulta em

187

x2 x3
1
A = ( x - x )dx = - = .
3 0 6
2
0
2

Finalmente, as coordenadas do centro de massa se calculam como


1

x3 x4
1
x = 6 x ( x - x ) dx = 6 ( x - x )dx = 6 - = ,
3 4 0 2
0
0
2

x3 x5
2
y = 3 ( x - x ) dx = 3 - = .
3 5 0 5
0
2

3.4.1 Exerccios
1) Encontre as coordenadas do centro de massa da regio no
primeiro quadrante, compreendida entre a circunferncia
x 2 + y 2 = r 2 , e os eixos coordenados.
2) Calcule as coordenadas do centro de massa da regio compre
endida entre as curvas
, y = cos x com x [0, ] .
4
3) (Teorema de Pappus) Mostre que o volume de um slido de
revoluo obtido pela rotao ao redor do eixo y de uma regio plana (sem perda de generalidade, assuma que a figura
se encontra totalmente no primeiro quadrante) pode ser obtida
pelo produto da rea da regio pelo comprimento da circunferncia, cujo raio a coordenada x do centro de massa, isto
, V = 2 xA . (Sugesto: utilize o mtodo das cascas cilndricas
para o clculo do volume e identifique no resultado a frmula
da coordenada x do centro de massa.)
4) Use o Teorema de Pappus para calcular o volume de uma esfera de raio r (sem perda de generalidade, assuma que est
centrada na origem).
5) Verifique, utilizando as frmulas para as coordenadas do centro de massa, que o centro de massa de um tringulo issceles
est situado sobre a altura relativa base a uma distncia da
base de um tero da altura total. (Sugesto: sem perda de generalidade, pode-se colocar a base do tringulo sobre o eixo x
e o terceiro vrtice do tringulo sobre o eixo y .)

188

3.5 Curvas e reas em coordenadas


polares
Vamos estudar o clculo de reas compreendidas por curvas descritas em coordenadas polares. Como o leitor j deve ter estudado em
geometria analtica, o sistema de coordenadas polares uma outra
maneira de descrever os pontos do plano como um par ordenado de
nmeros reais. A primeira coordenada de um ponto neste sistema
a distncia deste ponto a um ponto fixo no plano, o qual denominamos polo. A segunda coordenada do ponto, a medida do ngulo formado
por duas semi-retas: a semi-reta que une o polo ao ponto e o eixo polar, que
uma semi-reta fixa partindo do polo. A figura 3.23 abaixo nos ilustra os
elementos da representao em coordenadas polares.
P = ( r, 0)
r

0
Figura 3.23: Representao de um ponto do plano em coordenadas polares. Na figura, a letra
o representa o polo e a semirreta horizontal representa o eixo polar. As coordenadas polares
.
so dadas pelo par ordenado

As coordenadas polares de um ponto P no plano, portanto, so


. A origem no possui uma representao unvoca em termos
de coordenadas polares, uma vez que qualquer ngulo poderia representar a origem. Dissemos que a coordenada r representa uma
distncia, portanto deveria ser um nmero real positivo, mas em
muitas situaes ser til e necessrio introduzirmos valores negativos para a coordenada r , que definiremos como (- r , ) = (r , + ) ,
com r > 0 , conforme nos ilustra a figura 3.24 abaixo:

189

( r , )
r

( r , ) = ( r , + )
Figura 3.24: Coordenadas polares com raio negativo.

Podemos comparar as coordenadas polares de um ponto no plano


com suas coordenadas cartesianas. Por conveno, costuma-se fazer
o polo coincidir com a origem do sistema de coordenadas cartesianas e o eixo polar coincidir com o eixo x positivo. As expresses
mais importantes que vo nos permitir trabalhar com coordenadas
polares so as frmulas de mudanas de coordenadas, de polares
para cartesianas e de cartesianas para polares. A figura 3.25 nos
ilustra como obtermos essas transformaes.
y
x

( r, 0)
y

0
Figura 3.25: Transformaes de coordenadas.

190

Na figura, vemos facilmente que

. Assim, se

quisermos expressar as coordenadas cartesianas em termos das coordenadas polares, teremos


e y = r sen .

(3.8)

Similarmente, se quisermos expressar as coordenadas polares em


termos das cartesianas, teremos
r = x 2 + y 2 e = arc tg

y
.
x

(3.9)

O leitor convidado a verificar que a expresso de em termos do


arco tangente vlida nos quatro quadrantes, inclusive sendo consistente com os limites laterais quando x tende a zero pela esquerda
ou pela direita. Assim, para se determinar o ngulo , necessrio
levar em conta o quadrante onde est o ponto ( x, y ) .
A utilidade das coordenadas polares est na representao de lugares geomtricos cujas equaes em coordenadas cartesianas podem
ser bem complicadas. Um exemplo trivial so as circunferncias
com centro na origem e de raio R , cuja equao cartesiana conhecemos muito bem, que x 2 + y 2 = R 2 , inserindo as frmulas (3.8)
obtemos a igualdade r = R . Pode haver casos em que a equao cartesiana seja efetivamente mais simples, como o caso das retas, com
equaes ax + by + c = 0 , cuja substituio por coordenadas polares
transforma em uma igualdade do tipo ar cos + br sen + c = 0 , notoriamente mais complicada. Porm, no caso em que tivermos c = 0 ,
a
a reta pode ser escrita como y = - x , se b 0 , ou, x = 0 se b = 0 ,
b
a
no primeiro caso, a equao polar da reta fica = arc tg - , e no
b
segundo caso, quando x = 0 , temos a equao

Vamos apresentar mais alguns exemplos de curvas em coordenadas


polares:
Exemplo 3.1. Considere a curva em coordenadas polares dada por
r = 2sen . Multiplicando ambos os lados da igualdade por r , temos

191
r 2 = 2rcos , que pode ser rapidamente convertido para coordenadas
cartesianas como x 2 + y 2 = 2 x , ou ainda ( x - 1) 2 + y 2 = 1 . Ou seja,
trata-se de uma circunferncia de centro (1,0) e raio 1.

Antes de passarmos ao prximo exemplo, vamos fazer alguns comentrios sobre o mtodo de esboarmos curvas em coordenadas
polares. O leitor vai perceber que a maioria das curvas interessantes
poder ser escrita na forma r = f () . Ento, a primeira coisa a fazer
esboar o grfico da funo f em um plano auxiliar, cujas coordenadas cartesianas so (, r ) (isso mesmo: a coordenada dada pela
posio em relao ao eixo horizontal e a coordenada r em relao
ao eixo vertical). Neste exemplo, a funo seria f () = 2cos , que
o grfico da funo co-seno a menos de uma mudana de escala
na direo vertical. Com este grfico em mos, podemos esboar a
curva real, no plano ( x, y ) , utilizando as frmulas (3.8). Vejamos: o
grfico da funo f () = 2cos no plano (, r ) est representado na
figura 3.26.
r

claro que voc poder


tambm utilizar uma tabela,
com valores especficos de
e r , para esboar alguns
pontos da curva.

-2
Figura 3.26: Grfico da funo f () = 2cos no plano com coordenadas cartesianas (, r ) .

A curva pode ser esboada com o auxlio deste grfico, lembrandose que, se um ponto possui a coordenada r < 0 , significa que devemos efetuar a adio de na parte angular das coordenadas do

192

ponto e trabalharmos com a parte radial igual a -r > 0 . Temos, portanto, a curva r = 2 cos no plano cartesiano ( x, y ) , ilustrada na figura 3.27. Nesta figura, marcamos alguns pontos, cujas coordenadas


polares so, respectivamente, A = 3, , B = 2, , C = 1, ,
6
4

3
2
3
5

0 = 0, , D = -1,
, E = - 2,
, F = - 3,
.A represen3
4
6
2

tao desta curva no plano polar um crculo de raio 1 centrado no


ponto de coordenadas cartesianas (1, 0) , conforme a figura 3.27 a
seguir.
r

(1,0)

(2,0)

Figura 3.27: Circunferncia de equao

r = 2sen .

Exemplo 3.2. Considere a curva dada em coordenadas polares


dada por r = 1 + cos . Note que a equao em coordenadas cartesianas pode realmente ficar bem complexa neste caso. De fato,
multiplicando a igualdade por r , temos r 2 = r + rcos , ou seja,
2
x 2 + y 2 = x 2 + y 2 + x , ou ainda ( x ( x - 1) + y 2 ) = x 2 + y 2 . Para fazermos o esboo desta curva, novamente precisamos recorrer ao grfico da funo f () = 1 + cos no plano cartesiano (, r ) , conforme
nos ilustra a figura 3.28.

193
r
2

Figura 3.28: Grfico da funo f () = 1 + cos no plano com coordenadas cartesianas (, r ) .

Este nome deve-se ao fato


de o formato da curva
lembrar a forma de um
corao.

Com o auxlio deste grfico, podemos desenhar a curva, que denominada Cardioide, como nos mostra a figura 3.29.
y
1

Em 1694, o matemtico
suo Jacob Bernoulli
(1654-1705) publicou na
revista Acta Eruditorum
um artigo introduzindo
uma nova curva que levou
originalmente o nome de
Lemniscus. Na verdade, a
Lemniscata tambm pode
ser vista como um elemento
de uma famlia infinita de
curvas chamadas Ovais
de Cassini, introduzidas
pelo matemtico italiano
Giovanni Domenico Cassini
(1625-1712) no ano de
1680.

Figura 3.29: Cardioide

r = 1 + cos .

Exemplo 3.3. A Lemniscata de Bernoulli o lugar geomtrico dos


pontos cujo produto das distncias a dois pontos fixos uma constante. Para fixarmos um exemplo, considere os pontos ( a,0) e ( -a,0)
(em coordenadas cartesianas) e todos os pontos do plano cujo produto das distncias a estes dois pontos dados seja igual a a 2 . Assim,
teremos

(( x - a)

+ y 2 )( ( x + a ) 2 + y 2 ) = a 4 ,

194
que aps algumas manipulaes algbricas resulta na equao cartesiana
2
( x 2 + y 2 ) = 2a 2 ( x 2 - y 2 ) .
A equao acima pode ser colocada em coordenadas polares resultando em
r 2 = 2a 2 cos2 .

Desafiamos o leitor a obter a expresso em coordenadas polares a


partir da equao da curva em coordenadas cartesianas. O esboo
da Lemniscata est representado na figura 3.30 abaixo:
y

-1

Figura 3.30: Lemniscata de Bernoulli.

3.5.1 Exerccios
1) Esboce a curva r = 2sen , conhecida como roscea de quatro
ptalas. Encontre sua equao em coordenadas cartesianas.
2) Encontre as equaes cartesianas das seguintes curvas:
a) r = 2sen .

1
.
1 - cos
5
c) r =
.
3 - 4sen
d) r 2 .
b) r =

195
3) Determine as equaes polares das seguintes curvas:
a) 2 xy = 1 .
Nicomedes foi um
matemtico grego do sculo
III a.C. A Conchoide uma
curva especial construda
para auxiliar na soluo de
dois problemas clssicos de
construes geomtricas,
a saber, a trisseco de
ngulos e a duplicao do
cubo.

b) x 2 = 4 y .
c) x 2 - y 2 = 1 .
d) y = x + 1 .
4) Esboce a curva r = 4 + 2sec , conhecida como Conchoide de
Nicomedes. (Sugesto: verifique que a reta vertical x = 2
uma assntota da curva, isto , verifique que lim x = 2 .)
r

Diocles foi um matemtico


grego que viveu
aproximadamente no
perodo de 240 a.C a 180
a.C. Diocles tambm tratou
do problema clssico
de duplicao do cubo
e props uma soluo
atravs da curva especial
denominada Cissoide, que
tem esse nome em aluso
ao nome grego da folha da
planta hera.

5) Esboce a curva r = sen tg , conhecida como Cissoide de Diocles. (Sugesto: verifique que a reta x = 1 assntota vertical
dessa curva. Verifique tambm que a curva fica restrita regio do plano definida por 0 x < 1 .)
Finalmente, como aplicao de integral, vamos calcular a rea da
regio compreendida por uma curva em coordenadas polares. Considere uma curva dada pela equao r = f () . O objetivo calcular
a rea da regio radial delimitada pela curva entre as retas
e
= b , isto , as semirretas a partir da origem com inclinaes dadas
pelos ngulos
e = b . Suponha, sem perda de generalidade,
que b > a conforme ilustrado na figura 3.31 abaixo:
y

=b
r = f ( )

=a
x
Figura 3.31: rea em coordenadas polares.

196

Novamente, consideremos uma partio P : a = 0 < 1 < < N = b


do intervalo [a, b] . Entre os ngulos i -1 e i , considere o ngulo *i , tal que a rea do setor delimitado pela curva compreendido entre i -1 e i tenha a mesma rea que o setor circular de raio
f (*i ) e ngulo central. Assim, a rea entre i -1 e i ser dada por
1
Ai = ( f (*i )) 2 (i - i -1 ) .Assim, uma aproximao para a rea total
2
associada dada por essa partio ser
N
1
AP = ( f (*i )) 2 (i - i -1 ) .
i =1 2

Tomando o limite, quando as parties se tornam arbitrariamente


finas, obtemos a integral
A=

1
( f ()) 2 d .
2 a

(3.10)

Exerccio resolvido 3.19. Encontre a rea da regio delimitada por


um lao da roscea de quatro ptalas r = cos2 .
Soluo: Aqui, temos f () = cos2 . Agora, note que a equao

nos fornece que r - = r = 0 , enquanto r () > 0 para


4
4

- < < , logo temos um lao. A rea da regio delimitada por


4
4
esse lao ser, ento,
A=

1
2

cos
-

2 d =

1
4

(1 + cos 4)d = 4 1 + 4 sen 4

4
-

.
8

Exerccio resolvido 3.20. Determine a rea da regio interior ao crculo delimitado pela circunferncia r = 3sen e exterior ao cardioide
.
Soluo: Primeiramente, precisamos encontrar os pontos de interseco das duas curvas, isso feito igualando-se as duas expresses:
5

1
3sen = 1 + sen , ou seja, sen = , o que nos d = e =
.
6
2
2

197

Assim, a rea da regio ser


A=

1
2

5
6

((3sen)

- (1 + sen) 2 )d .

Podemos ainda explorar a simetria das curvas em relao reta ver


tical x = 0 , ou seja, = , resultando em
2

A = (8sen 2 - 1 - 2sen)d = (3 - 4cos2 - 2sen)d = .

3.5.2 Exerccios
1) Encontre a rea da regio delimitada por um lao da curva
r = 2cos4 .
2) Encontre a rea da regio dentro da curva r = 1 - cos e fora da
3
curva r = .
2
3) Encontre a rea da Lemniscata

4) Encontre a rea da regio interna, simultaneamente, s curvas

r sen ee r cos .
5) Determine a rea delimitada pelo eixo polar e pela espiral de
Arquimedes,
, para 0 2 .

Respostas dos exerccios


3.1.1 Exerccios
1)
y 2 x3
=
+K.
2
3
y2
b)
= x - arctan x + K .
2
1
c) = - cos x + K .
y
x2
d) arctan y = x + + K .
2
a)

198

1
e) tan 2 y = x + sen2 x + K .
2
f)
g)

.
y2
x2
+ ey =
+ e- x + K .
2
2

h) y =

a
1
ad
x + b ln cx + d + K .
c
c
c

i)

c
1
cb
y + d - ln ay + b = x + K .
a
a
a

a)

1
1
sin 3 y = (cos 2 x + 1) = cos2 x .
3
2

2)

y2
1
= ( x + 1)e - x - .
2
2
2
y
c)
= ln(1 + x 2 ) - 1 .
2
b)

d) y + y 2 = x 2 - 4 .
3) Divida o segundo membro no numerador e denominador por
2
x e note que y= xu+u , a equao resultante fica xu= u - 4 .
1- u

3.2.1 Exerccios
1)
a) L =

17
.
12

1
b) L = 2 + ln 2 .
4
c) L = ln(1 + 2) .
e - e -1
d) L =
.
2
1
e) L =
2 + ln(1 + 2) .
4

199

3.3.1 Exerccios
4 ab 2
.
1) V =
3
2) V = 2 2 ab 2 .
8
3) V =
.
15
8
.
15
5) V = .

4) V =

2
.
2

7) A = .
4
6) V =

3.3.2 Exerccios
3) Os dois slidos tm o mesmo volume.
4) V = 2 .

.
240
6) V = 2 .

5) V =

7) V = 2 2 .

3.3.3 Exerccios

.
4

4) A =
(145 45 - 10 10) .
27

2) A =

5
5 +1
5) A = 2
+ ln
.
4
2

200

3.4.1 Exerccios
4r 4r
1) , .
3 3

2)
2 + 1 ,
.

4( 2 - 1)
2 -1 4

3.5.1 Exerccios
y

1)

-1

2) a) x1 + ( y - 1) 2 = 1 .b) y 2 = 1 + 2 x .c) x 2 + y 2 = arctan


3) a)

b) rcos 2 = 4sen .
c) r 2 cos2 = 1 .
d)

.
y

4)

y
.
x

201
y

5)

3.5.2 Exerccios
1) A =

.
4

2) A =

9 3
- .
8
4

3) A = 2 2 .
1 1
4) A = - .
2 4 2

5) A =

4 3
.
3

Captulo 4
Sries Numricas

Captulo 4
Sries Numricas
Neste e no prximo captulo estudaremos sries, denominao das somas de infinitos termos. Comearemos
com sries numricas que formam a base para expressar muitas funes, como polinmios infinitos, denominados de sries de potncias, que desempenham um
papel fundamental no entendimento do Clculo.

4.1 Introduo
O primeiro significado de infinito que encontramos no dicionrio
no finito e, entre outras definies, encontramos ter um tamanho ou valor absoluto que maior que qualquer nmero natural,
grandeza cujo mdulo arbitrariamente grande. Podemos pensar que o homem encontrou o infinito sob a forma de distncias
grandes demais para serem medidas e nmeros grandes demais
para serem contados, e a ideia de somar infinitos nmeros reais
bem antiga. Pelo menos quatro dos paradoxos de Zeno de Eleia
(490-425 a.C.) sobre o movimento envolvem a soma de infinitos
termos positivos a um nmero finito. Arquimedes (287-212 a.C.),
para suas demonstraes rigorosas das frmulas de certas reas e
volumes, encontrou vrias somas que contm infinitos termos.
Ele tambm utilizou o mtodo de exausto (argumento sequencial) de Zeno e tentou explicar como somas infinitas poderiam
ter resultados finitos, inventando argumentos muito engenhosos
que incorporam alguns detalhes tcnicos do que hoje chamamos de limite. Durante sculos as sries intrigaram matemticos
e muitos contriburam para seu desenvolvimento. No possuindo
o conceito de limite, propriamente dito, para alcanar resultados
esses matemticos inventavam tcnicas, desenvolviam esquemas
algbricos complicados ou apelavam para intuio geomtrica ou
filosfica, em algum ponto crtico. Nicole dOresme (1325-1382)
realizou estudos usando aproximao sequencial e inventou um

206
argumento para mostrar a divergncia da srie harmnica. Simon
Stevin (1548-1620) somou sries e analisou sequncias, mas parou
antes de definir ou explicar limites e convergncia.
Na metade final do sculo XVII teve incio a investigao de
sequncias e sries de funes com Isaac Newton (1642-1727) e
Gottfried Wilhelm Leibniz (1647-1716), que desenvolveram
representaes de sries para algumas funes. Leibniz somou e
analisou vrias sequncias geomtricas, tentou explicar o conceito
de limite e descobriu muitos resultados, hoje estudados em clculo.
O clculo obteve vrios sucessos no sculo XVIII e se desenvolveu
rapidamente, mas pouca ateno foi dada aos seus fundamentos mais
tericos, muito menos s ideias de limite e convergncia de sequncias
e sries. No seu trabalho principal, Jean Le Rond dAlembert (17171783) considerou a derivada como o limite do quociente de diferenas
e, tambm, desenvolveu o teste da razo para determinar a convergncia de muitas sries. Atravs do trabalho de dAlembert a natureza
da pesquisa sobre sries estava mudando de clculos prticos para
uma fundamentao mais terica. No final do sculo XVIII, Joseph
Louis Lagrange (1736-1813) fez um esforo heroico para tornar o clculo prioritariamente algbrico, eliminando limites inteiramente. Carl
Friedrich Gauss (1777-1855) produziu o primeiro tratamento estritamente rigoroso da convergncia de sequncias e sries, embora no
tenha usado a terminologia de limites. Augustin-Louis Cauchy (17891857) foi o primeiro a definir por completo as ideias de convergncia
absoluta de sries infinitas. Ele comeou o seu curso de clculo para
estudantes de engenharia na cole Polythechnique, em Paris, do nada.
Ele escreveu as suas prprias notas de aula, essencialmente seus prprios livros, comeando com uma definio moderna de limite e usando o princpio de limite como base para introdues precisas continuidade e convergncia, a derivada, a integral, e o resto do clculo.

Como surgem as sries


O primeiro exemplo de como aparecem sries numricas pode ser
dado pelo seguinte paradoxo de Zeno: para um corpo percorrer
uma certa distncia, ele tem primeiramente de percorrer a primeira
metade, antes dessa metade a metade dessa metade, e assim por
diante, isso pode ser explicado pela frmula

207
1 1 1 1 11
1 1 1
+ . +
+ ... = + 2 + 3 + ... = 1 .
2 2 2 2 22
2 2 2
1
= 0,3333 , um
3
decimal infinito que pode ser escrito como a srie numrica
3
3
3
3
1
0,333 = + 2 + 3 + ... + n + ... = .
10 10 10
10
3
Na matemtica elementar aprendemos que

De modo geral podemos escrever


a
a
a
a
0, a1a2 a3 an = 1 + 22 + 33 + + nn , ai {0, 1, 2, 3, ..., 9} .
10 10 10
10
Antes da definio de srie importante ficar claro que no podemos interpretar expresses da forma a1 + a2 + + an + literalmente como uma soma de infinitos nmeros reais, ou seja, no se
trata simplesmente um exemplo envolvendo a adio. Lembremos
que a adio uma operao binria, o que significa que sempre
operamos dois nmeros de cada vez. O resultado da operao um
nmero a que chamamos de soma. Quando operamos um nmero
finito de nmeros, podemos agrupar os nmeros e, ento, adicionlos dois de cada vez. As propriedades associativa e comutativa garantem que obtenhamos o mesmo resultado, no importando como
agrupemos os nmeros e a soma um nmero real. Entretanto, se
a soma de infinitos nmeros reais algumas vezes resulta em um
nmero, como no primeiro exemplo acima, j outras vezes teremos
expresses como
1 + 2 + 3 + 4 + =
(Lembre-se que infinito no um nmero!), e ainda pode ocorrer
que seja impossvel definir um resultado:
1 1 + 1 1 + 1 1 +

( zero? um? Ou no zero nem um?)


Ento, primeiro preciso definir o que significa a soma. Tambm
preciso provar que valem as operaes usuais quando envolvem
infinitos termos. Lembramos que, sempre que utilizarmos propriedades relativas s operaes com nmeros reais ou s relaes de
ordem, com as quais estamos habituados, devemos estar bem atentos ao fato que elas so vlidas apenas em processos envolvendo um
nmero finito de parcelas.

208

4.2 Definies
Comecemos analisando as seguintes somas de infinitos termos.
1 1 1 1
+ + + + ... = 1
2 4 8 16
preciso dar um sentido para o primeiro membro da igualdade. Como no podemos somar todas as parcelas do primeiro membro da igualdade de uma vez, comecemos somando
os dois primeiros nmeros e, a cada passo, adicionaremos o
prximo termo ao resultado obtido. Denotemos por Si a soma
1 1 1
das primeiras i parcelas da sequncia: , , ,... , sendo i :
2 4 8
1 1 3
S2 = + = ;
2 4 4

A) Para dar um significado igualdade

S3 =

1 1 1 1 1 1 3 1 7
+ + = + + = + = ;
2 4 8 2 4 8 4 8 8

S4 =

1 1 1 1 1 1 1 1 7 1 15
+ + + = + + + = + = .
2 4 8 16 2 4 8 16 8 16 16

Observe que os nmeros envolvidos nas somas possuem uma


caracterstica: o numerador sempre 1 e no denominador
temos 2n . Assim,
1 1 1
1
S n = + 2 + 3 + ... + n .
2 2 2
2
As somas, encontradas acima, tambm tm uma caracterstica:
2n - 1
1
S n = n ou S n = 1 - n .
2
2
Assim,
Sn =

1 1 1
1
1
+ 2 + 3 + ... + n = 1 - n .
2 2 2
2
2

Antes de prosseguirmos, vamos provar que a frmula encontrada para Sn vale para qualquer nmero natural n, usando o
Princpio de Induo Matemtica.
(Fundamentos de Matemtica I). Para a prova, apesar de no
ser uma soma, propriamente dita, denotamos o primeiro ter1
mo da soma por S1 ; S1 = .
2

Induo matemtica
um mtodo de prova
matemtico, usado para
demonstrar a verdade
de infinitas, porm
enumerveis proposies.
A forma mais simples e
mais comum de induo
matemtica para provar que
uma propriedade vale para
todos os nmeros naturais
n e consiste de dois passos:
1) A base: mostrar que o
enunciado vale para n = 1.
2) O passo indutivo:
mostrar que, se o enunciado
vale para n = k, ento, o
mesmo enunciado valer
para n = k + 1.

209
1
o valor de S n para n = 1 , o que prova
2
que a frmula vale para n = 1 e assim temos o primeiro
passo da prova.

a) Observe que

b) Supondo que o mesmo vale para n = k ,


Sk =

1 1 1
1
1
+ 2 + 3 + ... + k = 1 - k , provaremos para n = k + 1 ;
2 2 2
2
2

1 1 1
1
1
1
1 1 1
+ 2 + 3 + + k + k +1 = + 2 + 3 + + k
2 2 2
2 2
2
2 2 2
1 1

= 1 - k + k +1
2 2

S k +1 =

1
+ k +1
2

1
1
= 1 - k - k +1
2 2
1
= 1 - k +1
2
Portanto, a frmula vale para todos os nmeros naturais.
Outra maneira de chegarmos mesma expresso para
1 1 1
1
S n observar que + 2 + 3 + ... + n a soma dos n pri2 2 2
2
meiros termos de uma progresso geomtrica de razo
1
r = e lembrarmos que a soma dos n primeiros termos
2
de uma progresso geomtrica, da qual o primeiro termo
a e a razo r ,
a (r n - 1)
a + ar + ar 2 ++ ar n -1 =
.
r -1
Obtivemos, assim, uma sequncia numrica S1 , S 2 , , S n , ,
1
onde S n = 1 - n .
2
Como j estudamos sequncia e sua convergncia, sabemos
1
1
= 0 , o que implica que lim S n = lim 1 - n = 1 .
n
n
n

2
2
1 1 1 1
Portanto, a igualdade + + + + ... = 1 pode ser escri2 4 8 16

que lim

1 1 1
1
ta como lim + + + ... + n = 1 .
n 2
4 8
2

210
Lembrando do conceito de limite, a igualdade significa
que, a cada vez que adicionamos uma parcela na soma
obtida (o que se faz aumentando o valor de n), mais a
soma aproxima-se de 1. Assim, ao primeiro membro da
igualdade no significa realmente que adicionamos infinitos termos um a um (e nem todos. Por qu?). E, claro,
substituindo o valor de n na frmula de Sn, que a soma de
qualquer nmero finito de termos no 1.
Uma prova geomtrica da igualdade: considere um
quadrado unitrio e divida-o infinitamente ao meio,
conforme a Figura 4.1 a seguir. A soma das reas dos retngulos igual a um.

1
8
1

1
16

1
2
1
4
Figura 4.1

B) Outro exemplo a soma de todos os nmeros naturais, que


voc, j deve ter visto: 1 + 2 + 3 + 4 ++ n + .
A soma dos n primeiros nmeros naturais calculada pela
seguinte frmula:
n(1 + n)
1 + 2 + 3 + + n =
.
2
(Voc j viu a prova dessa frmula em Fundamentos da Matemtica I, Captulo 5, mas, se desejar, fcil provar usando novamente o Princpio de Induo Matemtica).
n(1 + n)
muito grande quando n grande e,
2
quanto maior n, maior a soma.
Note que S n =

Nos exemplos A) e B) apresentados at aqui possvel notarmos que a base terica de sries a convergncia de sequn-

211
cias e a soma de infinitos termos definida usando a noo
de limite.
Definio 4.1. Seja (an ) uma sequncia de nmeros reais. Uma expresso da forma a1 + a2 + + an + denominada srie numrica
ou simplesmente srie. O nmero an denominado o ensimo termo
ou termo geral da srie.
A partir da sequncia (an ) formamos uma nova sequncia (Sn ),
cujos elementos so as seguintes somas:
S1 = a1
S 2 = a1 + a2
S3 = a1 + a2 + a3

S n = a1 + a2 + + an

Essas somas so denominadas somas parciais ou reduzidas da srie e


so nmeros reais porque, para cada n natural, S n soma de um
nmero finito de nmeros reais.
Se existir o limite S = lim S n = lim(a1 + a2 ++ an ) , dizemos que a
n

srie convergente e o limite S definido como a soma da srie.


Nesse caso, a1 + a2 + + an + = S . Caso contrrio, ou seja, se o limite no existir ou tender a infinito, dizemos que a srie divergente.
Nota: Rigorosamente, uma srie infinita a1 + a2 ++ an + apenas um sinal para indicar que voc deve calcular a sequncia
S1 , S 2 , S3 , , S n das somas parciais e verificar se ela convergente ou divergente.
Notao: Em qualquer caso, convergente ou divergente, usamos a

notao sigma para srie:

n =1

= a1 + a2 + + an + .
n

Assim, a soma parcial S n = a1 + a2 + + an = ak .


k =1

Outras notaes:

a , a
k =1

i =1

estiver claro que n = 1, 2,3,

etc., ou simplesmente

, quando

212

s vezes conveniente considerarmos sries que comeam em a0 ,


em vez de a1 e, nesse caso, escrevemos

a
n =0

1
2
3
+
+
+ . Para escrever a srie
2.3 3.4 4.5
usando a notao sigma precisamos da expresso do termo geral.
Observe que

Exemplo 4.1. Seja a srie

1
1
=
2.3 (1 + 1)(1 + 2)
2
2
=
3.4 (2 + 1)(2 + 2)
3
3
.
=
4.5 (3 + 1)(3 + 2)
Conclumos que an =

n
, e escrevemos
(n + 1)(n + 2)

1
2
3
n
.
+
+
+ =
2.3 3.4 4.5
(
n
1)(
n
2)
+
+
n =1

Exemplo 4.2. No processo inverso, escrevemos os primeiros termos


de uma srie:
n

1 1 1
1 n
1

- = 1 - + - + + (- ) + .
2
2 4 8
2
n =0
(Note que essa srie comea com a0 ).
1
convergente e sua soma fcil de
n =1 n( n + 1)
calcular. conhecida como Srie Telescpica.

Exemplo 4.3. A srie

Antes de calcularmos as somas parciais, note que o termo geral da


1
1
1
1
srie an =
= . Assim,
pode ser escrito como
n(n + 1) n n + 1
n(n + 1)
1 1
1
S1 = a1 = - = 1 1 2
2
1
1 1 1
S2 = a1 + a2 = s1 + a2 = 1 - + - = 1 3
2 2 3
1
1 1 1
S3 = a1 + a2 + a3 = s2 + a3 = 1 - + - = 1 4
3 3 4

213
e a soma parcial S n
1
1
1 1
.
S n = S n -1 + an = 1 - + = 1n +1
n n n +1
Note que a expresso encontrada vale para todo n . (Por qu?)

1
1

1
Como o lim S n = lim 1 =
,
temos,
ento,
que
= 1.

n
n
n +1
n =1 n( n + 1)
Observao 4.1. Exemplos de sries convergentes e de que a soma
fcil de calcularmos, como a srie do Exemplo 4.3 anterior, no
so frequentes. Nos exemplos vistos at agora, temos uma expresso
para a soma parcial S n , o que nem sempre acontece.
Exemplo 4.4. A srie

(-1)
n =1

Teorema estudado em
Clculo I: Se lim xn = l ,
n

ento toda subsequncia


de ( xn ) converge para o
limite l .

n +1

= 1 - 1 + 1 - 1 + divergente.

De fato, as somas parciais S n de ordem mpar, S 2 k -1 , so iguais a


1 e as de ordem par S 2k so iguais a zero (Verifique!). Como duas
subsequncias, ( S 2 k ) e ( S 2 k -1 ) , convergem para limites diferentes, a
sequncia ( S n ) no convergente. Logo, a srie dada divergente.
Nota: Voltamos questo de notao:

a
n =1

apenas um smbolo.

Se no tomarmos cuidado, podemos facilmente chegar a concluses


erradas. Na histria da Matemtica, at mesmo grandes matemticos cometeram equvocos.
No Exemplo 4.4 visto h pouco, se tivssemos o direito de considerar a soma S dessa srie, S = 1 1 + 1 1 + , teramos
1
S = 1 - (1 - 1 + 1 - 1 +) e S = 1 S , o que implica S = , o que no
2
verdade, porque a srie divergente!
Na introduo deste Captulo lembramos voc de que as propriedades da adio so vlidas quando se trata de nmero finito de parcelas. A srie 1 1 + 1 1 + no pode ser substituda por
(1 1) + (1 1) + (1 1) +, que convergente para zero!
Exemplo Especial. Sries Geomtricas.
Provavelmente, o primeiro exemplo de uma srie convergente que
voc estudou (e, talvez, no exatamente dessa maneira) foi a srie

214

geomtrica. Ela, tambm, um dos processos infinitos sobre os


quais os matemticos estavam relativamente seguros antes do surgimento do Clculo. Na verdade, uma classe de sries, muito til,
como veremos no desenvolvimento da teoria; para as sries geomtricas sabemos tudo sobre a convergncia e divergncia e, para as
sries convergentes, a frmula para a soma conhecida.
Numa srie geomtrica, cada termo obtido multiplicando-se o seu
termo precedente pelo mesmo nmero, r 0 , conhecido como razo. Assim, uma srie geomtrica da forma

a + ar + ar 2 + + ar n -1 + = ar n -1 ,
n =1

onde a e r so nmeros reais fixos, diferentes de zero.


Ateno! Se r = 0 , a expresso esquerda da igualdade igual a a ,
porm temos um problema na notao sigma direita. Observemos
que a potncia de r n 1 , pois o primeiro termo a = ar 0 , mas 00
no um nmero (No igual a um! uma indeterminao exponencial. Veja em Clculo I ou disciplinas anteriores.). Estabelecendo
que

ar

n -1

= a quando r = 0 , usaremos a notao tambm nesse

n =1

caso, sem comprometer a matemtica!


Antes da anlise da srie observemos que no h restrio sobre o
nmero r , que pode ser positivo ou negativo.
Vamos deduzir aqui a frmula da soma dos n primeiros termos de
uma progresso geomtrica que a ensima soma parcial da srie:
S n = a + ar + ar 2 + + ar n -1 .
Multiplicando-se os dois membros da igualdade por r obtemos:
rS n = ar + ar 2 + + ar n -1 + ar n .
Subtraindo-se rS n de S n obtemos:
S n - rS n = a + ar + ar 2 + + ar n -1 - ar - ar 2 - - ar n -1 - ar n = a - ar n ,
ou (1 - r ) S n = a (1 - r n ) .

215
a (1 - r n )
.
Para r 1 (Por qu?) temos: S n =
1- r
Usando resultados para limites de sequncias (Clculo I), temos:
a (1 - r n )
a
=
.
n
1- r
1- r

i) Se 1 < r < 1 , ento lim S n = lim


n

ii) Se r 1 ou r > 1 (lembre-se que r 1 ), a sequncia ( S n )


divergente e, portanto, a srie geomtrica divergente para
tais r .
iii) Para r = 1 , a soma parcial S n = a + a + a + + a = na e, tambm, nesse caso a sequncia ( S n ) divergente. Logo, a srie
geomtrica divergente para r = 1 .

r < 1 e a soma

ar

mtrica divergente.

n -1

n =1

n =1

n =0

ar n-1 = ar n convergente se

Concluso: A srie geomtrica

= ar n =
n =0

a
. Se r 1 a srie geo1- r

Tarefa: Siga as etapas a partir da soma parcial da srie para mostrar

ar
que ar n =
, se | r | < 1 . (Lembre-se que a propriedade distri1- r
n =1
butiva s vale para um nmero finito de parcelas).

Exerccios resolvidos
1) Verifique se cada srie convergente ou divergente. No caso de
ser convergente, encontre a soma da srie.
n -1

2
2
a) 3 uma srie geomtrica de razo r = e a = 3 .
5
n =1 5
Sendo 0 < r < 1 , a srie convergente e, pela frmula da soma,

2
S=
= 5 . Assim, 3
2
n =1 5
15

n -1

=5.

216
n

1 1 1
1
1
b) - = 1 - + - + + - + , do Exemplo 4.2
2
2 4 8
2
n =0
1
anterior, uma srie geomtrica. Como a = 1 e r = - que
2
est no intervalo de convergncia, temos:

1
2
1
= .

- =
2
1 3
n =0
1- -
2
(Note que a potncia n e a srie comea em a0 ).

2 3

c) +
+
+ = uma srie divergente, pois
2 4
8
n =1 2
uma srie geomtrica de razo maior que 1.

2) Escreva o nmero 1,232323... como uma razo de dois nmeros


inteiros.
23
23
23
+
+
+
Resoluo: 1, 232323 = 1 +
100 10000 1000000
23 23 23
= 1+ 2 + 4 + 6 +
10 10 10
23 23 1
23 1
= 1+ 2 + 2 2 + 2 4 +
10 10 10 10 10
2

23
23 1 23 1
.1 + 2 2 + 2 2 + ... a srie geomObserve que
2
10
10 10 10 10
1
1
23
e a = 2 . Usando-se a frmula da
trica de razo r = 2 =
10 100
10
23
23 122
, e conclumos que 1, 232323 = 1 +
.
soma, obteos S =
=
99
99 99
3) Escreva a soma da srie

x , onde
n

1< x <1.

n =0

Soluo: Basta voc observar que a srie a srie geomtrica, com

1
n
x
=
x n -1 . Assim, x n =
r = x , a = 1 e que
, pois x < 1 .

1- x
n =0
n =1
n =0
Nota: Se efetuarmos a diviso do polinmio constante, p ( x) = 1 ,
pelo polinmio q ( x) = 1 x , obtemos 1 + x + x 2 + x 3 +, que a srie
dada. Verifique!

217

4.2.1 Exerccios
1) Qual o termo geral das sries a seguir?
a)

3 3
3
3
+ +
+
+ ;
5 25 125 625

b) x + x 2 +

x3
x4
+
+ , x ;
1.2 1.2.3

r2 r3 r4
c) r + 10 + 10 + 10 + , r .
2
3
4
2) Escreva a srie a1 + a2 + + an + , cujo termo geral :
a) an = (-1) n -1 3n -1 ;
cos(n )
.
n2

b) an =

3) Encontre uma frmula para a ensima soma parcial de cada


srie e use-a para encontrar a soma, se a srie for convergente.
7 7 7
7
a) 7 + + + + + n-1 +
2 4 8
2
b) 1 - 2 + 4 - 8 + + (-1) n -1 2n -1 +
c)

5
5
5
5
+
+
+ +
+
2.3 3.4 4.5
(n + 1)(n + 2)

4) Escreva os primeiros termos de cada srie geomtrica e calcule


sua soma, se a srie for convergente.
a)
b)

(-3) n
;

n
n =0 7

3
n =1

n -1

c)

5n
;

n
n =0 4

d)

(-1)

n =0

n .

218
5) Encontre os valores de x para os quais a srie dada convergente. Para esses valores de x escreva a soma como uma
funo de x .
a)

4( x - 3)

n =0

b)

2n

n =0

x +1
c)
;
n =0 2

d)

xn .

n =0

6) Uma bola jogada de uma altura de a metros, sobre uma superfcie plana. Cada vez que a bola atinge a sutperfcie depois
de cair de uma distncia a , ela rebate a uma distncia ra ,
onde r um valor positivo, mas menor que 1 (um). A distncia
vertical total percorrida pela bola pulando para cima e para
baixo (veja a Figura 4.2):
S = a + 2ar + 2ar 2 + 2ar 3 +.

a
ar
ar 2
ar 3

Figura 4.2:

1+ r
.
1- r
b) Calcule S se a = 4m e ar = 3m .

a) Mostre que S = a

219
7) Encontre uma srie infinita de termos diferentes de zero cuja
soma seja:
a) igual a 1;
b) igual a 3;
c) igual a zero.
8) Encontre o valor de t para o qual 1 + et + e 2t + e3t + = 5 .

4.3 Condies de convergncia e


divergncia
O problema principal da teoria das sries determinar se uma srie
convergente ou no. Nesta seo, estabeleceremos alguns critrios
de convergncia de uma srie que, em geral, so consequncias dos
resultados referentes convergncia das sequncias (Clculo I), a
comear do teorema a seguir.
Teorema 4.1. Critrio de Cauchy para sries
A demonstrao pode
ser encontrada em Lima,
1989 (Ver Bibliografia
Complementar, p. 307).

O enunciado do teorema equivalente a srie

a
n =1

conver-

gente se, e somente se, para cada > 0 , existir N tal que
| an +1 + an + 2 + + an + p |< , quaisquer que sejam n > N e p .
Nota: No critrio de Cauchy temos uma condio necessria e suficiente para convergncia de uma srie. Como comentamos na Observao 4.1, nem sempre fcil encontrar uma expresso para as
somas parciais, ento, para a prtica muito til reconhecer se uma
srie convergente ou no a partir do comportamento dos seus termos an , sem passar pelo clculo das somas parciais.
Teorema 4.2. Condio necessria de convergncia.
Se a srie

a
n =1

convergente, ento lim an = 0 .

Demonstrao: A srie

a
n =1

convergente, ento, pelo Teorema

4.1, dado > 0 , se existir N tal que | an +1 + an + 2 + + an + p | < ,

220
para quaisquer n > N e p . Em particular, para m = n + 1 e n > N
qualquer, vale | am | < . Assim, | an - 0 | < , qualquer n > N + 1 , ou
seja, lim an = 0 .
n

Demonstrao alternativa: Seja S a soma da srie convergente


Ento, lim S n = S , onde
n

a
n =1

( S n ) a sequncia das somas parciais,

S n = a1 + a2 + + an . claro que, tambm lim S n -1 = S . Observe que


n

S n - S n -1 = an e assim, lim an = lim( S n - S n -1 ) = S - S = 0 .


n

Observao 4.2. No teorema temos uma condio necessria para


convergncia de uma srie, ou seja, se uma srie convergente, ento o termo geral converge a zero. Mas, o problema principal da
teoria das sries justamente determinarmos se uma srie convergente ou no! Por outro lado, a forma negativa do teorema muito
til e a enunciamos como um teste de divergncia. Para isso, apesar
de bvio, vale a pena voc lembrar de que o limite de uma sequncia
ou de uma funo pode existir ( um nmero) ou no.
Teorema 4.3. Teste do termo geral para divergncia
Se o lim an 0 ou o lim an no existe, ento a srie
n

a
n =1

divergente.

De fato, se a srie fosse convergente, ento o limite do termo geral


seria zero.
Observao 4.3. O Teorema 4.3, a forma negativa do Teorema 4.2,
constitui uma condio suficiente para uma srie ser divergente. Por
outro lado, a recproca do Teorema 4.2 no verdadeira, ou seja, a
condio lim an = 0 no suficiente para (no garante) uma srie ser
n
convergente.
O exemplo clssico de srie, onde o termo geral converge a zero e diver
1
1 1
1
gente, a srie harmnica = 1 + + + + + .
2 3
n
n =1 n
1
Note que o lim an = lim = 0 , e vamos mostrar que a srie divern
n n
gente.
De fato, na sequncia das somas parciais ( S n ) consideremos os termos de ndices n = 2k ( S 2 , S 4 , S8 ,) .

221
1 1
1
1 1 1 1 1 1 1
1
1
+ ... + k >
S 2k = 1 + + + ... + k = 1 + + + + + + + + ... + k -1
2 3
2
2 3 4 5 6 7 8
2
2 +1
1 1 1 1 1 1 1
1
1
> 1 + + + + + + + + ... + k + ... + k
2 4 4 8 8 8 8
2
2
1 2 4
2k -1
1 1
1
Assim, S 2k > 1 + + + + ... + k = 1 + + + ... +
2 4 8
2
2 2
2
ou seja, S 2k > 1 + k

1
.
2

Note que o lim S 2k = + . Como ( S 2k ) uma subsequncia de ( S n ) ,


k

ento a sequncia de ( S n ) no convergente (mesmo teorema do Clculo I citado no Exemplo 4.4). Logo, a srie harmnica divergente.
Exemplo 4.6. Aplicao do teste do termo geral
a)

(-1)

n -1

divergente, como j vimos, porque o lim(-1) n -1 no


n

n =1

existe;
b)

n2
n2
1
lim

divergente,
pois
o
= 0;

2
2
n 2n + 5
2
n =1 2n + 5

c)

1- n
1- n
= -1 .
divergente porque lim
n n
n =1 n

4.3.1 Exerccios
1) Analise, justificando, quais sries so convergentes e quais so
divergentes.
a)

1
1
1
1
+
+ ;
10 100 1000 10000

b) 1 + 2 + 3 + + n + ;
c)

n +1 ;

f)

2n

ln n ;
n =1

cos(nx) .
n =1

1
d)
;
2
n =0

2
n =1

g)

n =1

e)

222

4.4 Operaes sobre sries


Das propriedades aritmticas do limite de sequncias resultam as
seguintes propriedades:
Teorema 4.4. Propriedades de sries convergent es
Se

an e

bn so sries convergentes e

n =1

n =1

an = A e
n =1

b
n =1

= B,

ento:

(a

i) a srie

n =1

+ bn ) convergente e

n =1

n =1

(an + bn ) = an + bn = A + B (Regra da adio ).


n =1

ii) para todo k , a srie

ka

ka
n =1

convergente e

= k an = kA (Regra da multiplicao por constante).

n =1

n =1

Note que i) e ii) (com k = -1 ) implicam a regra da diferena

(a
n =1

- bn ) .

Demonstrao:
i) Sejam An = a1 + a2 + + an e Bn = b1 + b2 + + bn as somas parciais de
de

an e
n =1

(a
n =1

b
n =1

, respectivamente. Ento, as somas parciais

+ bn ) so:
S n = (a1 + b1 ) + (a2 + b2 ) + + (an + bn )
= (a1 + a2 + + an ) + (b1 + b2 + + bn )

= An + Bn
Como o lim An = A e o lim Bn = B , ento o lim S n = A + B .
n

ii) As somas parciais de

ka
n =1

so:

S n = (ka1 ) + (ka2 ) + + (kan )


= k (a1 + a2 + + an )
= kAn
Assim, o lim S n = kA .
n

223

Exerccio resolvido
1) Encontre a soma das seguintes sries:
1 (-1) n

n+ n .
5
n =0 2

a)

Soluo: Observe que

2
n =0

a srie geomtrica de razo

1
e a = 1.
2

(-1) n
1
5
-1
=
= ,

=
n
1 6
n =0 5
n =0 5
1+
5
n

1 (-1) 17
-1
pois a srie geomtrica de razo
. Assim, n + n = .
5
5 6
n =0 2

1
1
Ento, n =
= 2 . Tambm,
1
n =0 2
12

2n +1 2.2n
1
2
b) n = n = 2 = 2
= 6 , porque
2
n =0 3
n =0 3
n =0 3
13
2
serie geomtrica de razo .
3

a
n =0 3

Ateno! As propriedades so vlidas para sries convergentes. Para

as duas sries

an e
n =1

b
n =1

divergentes no podemos concluir, em

geral, que a srie (an + bn ) convergente ou divergente. Por exemplo,

(-1)

n =1

n -1

n =1

= 1 - 1 + 1 - 1 + divergente,

tambm

[(-1)
n =1

n -1

ser

divergente,

por

(-1)

n =1

= -1 + 1 - 1 + 1 - 1 +

outro

lado

srie

+ (-1) n ] = 0 + 0 + 0 + ser convergente.

No entanto, temos as seguintes propriedades:

i) Se uma das sries, an ou bn , convergente e a outra divern =1

gente, ento as sries

(a
n =1

ii) Se a srie

n =1

+ bn ) e

(a
n =1

- bn ) so divergentes.

an divergente, ento a srie


n =1

para todo k .

ka
n =1

divergente,

224

Adio ou subtrao de termos


Podemos aumentar ou diminuir um nmero finito de termos de
uma srie sem alterar o fato de a srie ser convergente ou divergente, ou seja, a srie convergente continua sendo convergente e a divergente continua sendo divergente. No caso da convergncia isso, de
modo geral, muda a soma da srie.

Proposio 4.1. Uma srie

n = n0

convergente se, e somente se,

n =1

for convergente para n0 qualquer, porm fixo. Nesse caso,

a
n =1

= a1 + a2 + + an0 -1 + an .
n = n0

A prova fcil. Basta notarmos que, se as somas parciais de


so S n , as somas parciais de

n = n0

so Tn = S n S n0 -1 .

a
n =1

Exemplo 4.7. No Exemplo A, no incio da Seo 4.2, vimos que


n

1 1 1 1
1
1
+ + + + ... = 1 , ou n = = 1 . Por outro lado, a srie
2 4 8 16
n =1 2
n =1 2

1
1
1
geomtrica de razo r = , com a = 1 , n = 2 = 1 + n .
2
n =0 2
n =1 2

Reindexao
Podemos re-indexar qualquer srie, na notao sigma, sem alterar
sua convergncia, desde que a ordem de seus termos seja mantida.
Para aumentar o valor inicial do ndice n em h unidades, substitumos n , na frmula para o termo geral an , por n - h (aumentamos o
valor de n , logo diminumos em an ) .

an =
n =1

n =1+ h

n-h

= a1 + a2 + + an + .

(Note que a srie no , e nem pode ser, alterada!)

225

Para diminuir, substitumos n por n + h :

an =
n =1

n =1- h

n+h

= a1 + a2 + + an + .

Exemplo 4.8. Na notao da srie geomtrica, diminumos o ndice


n = 1 em h = 1 unidade:

ar n-1 =
n =1

n =1-1

ar n -1+1 = ar n .
n =0

1
1
No Exemplo 4.5 visto anteriormente, ,
n =1 2
n =0 2

1
1
=


n =1 2
n =0 2

n +1

1
1
=


n =0 2
n =5 2

Tambm,

1
1
e =
n =0 2
n =1 2

n -5


n =-2 2

n -1

n+2

4.4.1 Exerccios
1) Encontre a soma das seguintes sries:
a)

5
n =0

b)
c)

1
;
2n

1 (-1) n

n + n ;
4
n =0 3

1
1
.
n
n +1
n =1

Sugesto para c): Calcule a ensima soma parcial e use-a para calcular a soma.

4.5 Sries de termos positivos


ou nulos
Na teoria das sequncias um importante critrio de convergncia
o de que toda sequncia montona limitada convergente.

226
Apesar de ter sido estudado recentemente, no Clculo I, vamos recordar que denominamos de sequncias montonas: as sequncias
crescentes ( x1 < x2 < x3 < , ou seja, xn < xn +1 , para todo n ), no
decrescentes ( xn xn +1 , para todo n ) , decrescentes ( xn > xn +1 ,
para todo n ) , e no crescentes ( xn xn +1 , para todo n ) .
Uma sequncia ( xn ) limitada se existirem nmeros reais a, b tais
que a xn b , para todo n . Se temos xn b , para todo n , a
sequncia ser limitada superiormente e, se a xn , para todo n ,
a sequncia ser limitada inferiormente. Uma sequncia no decrescente ser sempre limitada inferiormente, por exemplo, pelo seu
primeiro termo. Analogamente, uma sequncia no crescente ser
sempre limitada superiormente.
A restrio s sries de termos positivos ou nulos se deve ao fato
de que, nesse caso, as somas parciais so sempre no decrescentes:
S n +1 = S n + an +1 , e de an +1 0 segue que
S n +1 S n , para todo n , ou seja, S1 S 2 S3 S n S n +1 .
Teorema 4.5. Seja an 0 para todo n . A srie

a
n =1

ser conver-

gente se a sequncia de suas somas parciais ( S n ) for limitada


superiormente.
Demonstrao: Os termos da srie so positivos ou nulos ento pelo
que vimos acima, a sequncia das somas parciais no-decrescente
e limitada inferiormente. Por hiptese, a sequncia limitada superiormente, ento limitada e assim conclumos que ela convergente.
Logo, a srie convergente.

Nota: Do mesmo modo que o Teorema de Cauchy, o Teorema 4.5


base para os testes prticos que permitem determinar se uma srie
convergente ou divergente.

Teste da integral
Antes de enunciarmos o teste, vejamos o seguinte exemplo:
Exemplo 4.9.
Considere a srie de termos positivos

n
n =1

1 1 1 1
+ + + + ... .
12 22 32 42

227
1
= 0 , mas isso no nos garante que a srie
n n 2
seja convergente. Para mostrar que convergente consideremos a
1 1
1
soma parcial S n = 1 + 2 + 2 + .. + 2 .
2 3
n
fcil vermos que o lim

No conhecemos uma frmula simples para S n , mas usando os


conhecimentos de clculo j adquiridos, poderemos pensar nos
1
termos da srie como valores da funo f ( x) = 2 para x = n .
x
Assim, S n = f (1) + f (2) + + f (n) .
y

y = f ( x) =
1

1
x2

Figura 4.3: Retngulos de base com comprimento 1 e altura f ( n ) sob o grfico de f ( x) =

1
.
x2

1
, n , como reas dos retngulos de
n2
1
base de comprimento 1 e altura de comprimento 2 , como a Figura
n
4.3, temos:
n

1
1
S n = 1. f (1) + 1. f (2) ++ 1. f (n) < 1 + 2 dx < 1 + 2 dx .
x
x
1
1

1
Observe que 2 dx uma integral imprpria que calculamos no
x
Captulo 2: 1
Interpretando os valores

1
1
-1
-1
dx = lim = lim + 1 = 1 .
2
1 x 2 dx = lim

b x
b
x 1 b b

Logo, S n < 1 + 1 = 2 , para todo n , o que significa que a sequncia


das somas parciais limitada superiormente por 2. Pelo Teorema
4.5, a srie convergente. (Na verdade, Leonhard Euler (1707-1783)
2
mostrou que a soma
1, 64493 ).
6

228
Proposio 4.2. Teste da integral

Seja

n =1

uma srie de termos positivos. Suponha que an = f (n) ,

onde f uma funo de x contnua, positiva e decrescente, para


todo x N (para algum N fixo ).

i) Se a integral imprpria

a
n =1

f ( x)dx for convergente, ento a srie

ser convergente.

ii) Se a integral imprpria

a
n =1

f ( x)dx for divergente, ento a srie

ser divergente.

Demonstrao: Vamos provar para N = 1 porque a prova para N


qualquer anloga.
Com as caractersticas dadas da funo temos ideia do seu grfico
y = f ( x) e de x 1 , segue que o intervalo [1, ) est no domnio
de f . Aproximemos a rea da regio abaixo do grfico e acima do
eixo x , entre as retas verticais x = 1 e x = n , pela soma das reas
de retngulos de base de comprimento 1 e alturas de comprimentos
f (n) = an ou f (n + 1) = an +1 . Veja as figuras a seguir.

n n +1

Figura 4.4: a) Retngulo de base de comprimento 1 e altura de comprimento f ( n) = an .

229

n 1

Figura 4.4: b) Retngulo de base de comprimento 1 e altura de comprimento

f (n + 1) = an +1
n

Ento,

f ( x)dx a + a
1

n +1

ou

+ + an -1 e a2 + a3 + + an f ( x)dx ,
1

f ( x)dx a1 + a2 + + an -1 + an e a1 + a2 + a3 + + an a1 + f ( x)dx ,
n +1

e, assim,

f ( x)dx a1 + a2 + + an a1 + f ( x)dx .
1

i) Se a integral imprpria

f ( x)dx

for convergente, basta a de-

sigualdade do lado direito para mostrar que as somas parciais so li

mitadas superiormente por M = a1 + f ( x)dx e, pelo Teorema 4.5


1
a srie convergente.

ii) Se a integral imprpria

f ( x)dx

for divergente, usamos o lado

esquerdo da desigualdade para concluir que a srie divergente.

Exerccio resolvido
2) Mostre que a srie

n
n =1

1
n

De fato, observe que an =

convergente.
1

= f (n) , onde f ( x) =

, que
n n
x x
ser uma funo de x , contnua, positiva e decrescente se x 1
Para usar o teste da integral, calculamos a integral imprpria

1
-

-2

x dx = lim -2 x 2 = lim
+ 2 = 2 . Logo, a
1 x x dx = blim

1 b b
1

3
2

srie convergente.

230

Ateno! A convergncia da integral imprpria apenas garante que a srie convergente. O valor encontrado no a soma
da srie (veja o Exemplo 4.9).
2) Analise a srie

n =1

1
.
n

1
= 0 , mas isso no garante que a
n
n
1
1
= f (n) , onde f ( x) =

srie seja convergente. Note que an =


n
x
uma funo de x , contnua, positiva e decrescente se x 1 . Assim,
Resoluo: Observe que lim

b
1
12
1
2
x dx = lim 2 x = lim(2 x - 2) = + .
1 x dx = blim
b

1 b
1

1
divergente.
Portanto, a srie
n
n =1

Aplicao do teste da integral: Srie p ou p -Srie


A srie do Exemplo 4.9 e as dos Exerccios resolvidos 2) e 3) so da

1
forma p (sendo p uma constante), denominadas sries p ou
n =1 n
p -sries. Essa uma classe de sries (as sries geomtricas so outra) das quais sabemos tudo sobre a convergncia e divergncia.
Proposio 4.3. A p -srie

n
n =1

, sendo p uma constante real,

convergente se p > 1 , e divergente se p 1 .


De fato, primeiro lembremo-nos de um teste fcil de aplicar: o teste
1 0, p > 0
do termo geral. Para isso, o lim an = lim p =
.
n n
n
+, p < 0
1
E, para p = 0 , temos que 0 = 1 , para todo n , logo a srie din
vergente.

1
Ento, a srie p divergente se p 0 .
n =1 n
1
Para p > 0 , o lim p = 0 , mas, somente com isso, no podemos
n n
concluir nada sobre a srie.

231

1
, que para x 1 funo conxp
tnua, positiva e decrescente, pois p > 0 . Para aplicar o teste da integral, vamos calcular a integral imprpria para p 1 :
Observe que an = f (n) , onde f ( x) =

b
1
b
, p >1
x - p +1
1
1 1

-p
= lim
- 1 = p - 1
x dx = lim
.

1
p
1 x p dx = blim

b - p + 1

1 b 1 - p b
1
+, p < 1

Para p = 1 , temos

n , a srie harmnica que sabemos que divern =1

gente. Esse fato tambm pode ser provado pelo teste da integral:

1
1
dx = lim ln x
1 x dx = lim

b x
b
1

b
1

) = lim(ln b) = + .
b

1
Portanto, conclumos que p divergente se p 1 , e convergenn =1 n
te se p > 1 .

Nota: O teste da integral permite verificar a convergncia ou no


de soma de infinitos valores f (n) , n = 1, 2,3, 4,para funes f ,
diferentes das usadas na p -srie.

Exerccios resolvidos
1) Verifique se a srie

ln n
convergente ou divergente.
n =1 n

Resoluo: Aplicando o teste do termo geral temos:


ln n
1/ n
lim
= lim
= 0 . Ento, nada a concluir.
n n
n 1
ln n
ln x
Observe que an =
, x > 0 , em particular
= f (n) , para f ( x) =
n
x
x 1 . A funo f contnua, positiva e, para verificar que decrescente, usamos o teste da derivada primeira (Clculo I).
(1 x).x - 1.ln x 1 - ln x
=
< 0 , para x > e , conclumos que
x2
x2
f decrescente em [e, +) (veja clculo I). Assim, podemos aplicar o teste da integral e, para tal, vamos calcular a integral imprpria

b
ln x
ln x
dx , pois 3 > e .
3 x dx = blim

x
3

De f ( x) =

232
ln x
(ln x) 2
dx
=
+c,
x
2
usando integrao por partes ou pelo mtodo de substituio
(u = lnx) .
Usando uma primitiva na integral definida obteremos:
No Captulo 1 de integrao encontramos

(ln b) 2 (ln3) 2
ln x
=
dx
lim

= + . Sendo a integral imprpria


3 x
b
2
2

divergente, a srie dada divergente.


en
2) Mostre que a srie
convergente.
2n
n =1 1 + e
en
en
1
De fato, observe que lim
= lim 2 n = lim n = 0 , condio
n 1 + e 2 n
n 2e
n 2e
necessria, mas no suficiente, para concluir que a srie dada
convergente.
ex
Seja f ( x) =
, contnua, positiva em e f (n) = an . A deri1 + e2 x

vada de f ,
e x (1 + e 2 x ) - e x 2e 2 x e x (1 - e 2 x )
f ( x) =
=
< 0,
(1 + e 2 x ) 2
(1 + e 2 x ) 2
para x 1 (porque e x > 0 ,para todo x e 2 < e < 3 ), ento conclumos que f decrescente em [1, +) . Com as condies do
teste da integral satisfeitas, vamos calcular a integral imprpria.
ex
dx = arctg(e x ) + c (ver no Captulo 1 Tabela de integrais),
2x
1+ e

ex

dx = lim(arctg(eb ) - arctg(e)) = - arctg(e) .


segue que
2x
b

1+ e
2
1
Mostramos, assim, que a srie dada convergente.

De

Estimativa da soma de uma srie convergente


Em geral difcil encontrar a soma exata de uma srie convergente
(o problema encontrar uma frmula para a soma parcial S n ). Uma
maneira eficaz de calcular somas de certas sries atravs do desenvolvimento de funes conhecidas em sries, o qual ser estudado
no prximo captulo. O teste da integral nos d pelo menos uma
estimativa da soma.

233

Seja an uma srie convergente, cuja convergncia pode ser verificada


n =1

pelo teste da integral. Se S = an , denotaremos por Rn o erro comen =1

tido se usarmos a soma parcial S n como uma aproximao para a


soma total S , ou seja, Rn = S S n .
Na prova do teste da integral encontramos as seguintes desigualdades:
n +1

f ( x)dx a1 + a2 + + an -1 + an e a1 + a2 + a3 + + an a1 + f ( x)dx .
1

an+1 a
n+2

an+1 an+2
0

n+1

Figura 4.5: a)

Figura 4.5: b)

Das Figuras 4.5 a) e b) conclumos que

f ( x)dx an +1 + an + 2 + = Rn

n +1

n +1

e Rn = an +1 + an + 2 + f ( x)dx . Logo,

f ( x)dx Rn f ( x)dx uma

estimativa do resto para o teste da integral.

Somando S n em cada lado das desigualdades e usando Rn + S n = S


obtemos: S n +

n +1

f ( x)dx S S n + f ( x)dx .
n

Exerccio resolvido
6) Seja a p -srie

n
n =1

a) Faa uma aproximao da soma S da srie usando a soma


parcial S10 ;

234
b) Faa uma estimativa do erro envolvido na aproximao feita
em a).
c) Faa uma estimativa da soma S da srie usando S10 ;
d) Quantos termos sero necessrios para garantir que a soma
tenha preciso de 0,0005?
Resoluo:
a) Com o auxlio de uma calculadora encontramos
1 1 1
1
S10 = 3 + 3 + 3 + ... + 3 1,197532
1 2 3
10
( aproximadamente igual a).

1
Ento, 3 1,197532 .
n =1 n
b) Para fazer a estimativa do erro Rn , vamos calcular a
n qualquer:

x
n

dx para

b
x -2 b
1
1
1
1
-3
= lim 2 - 2 = 2 .
x dx = lim
n x3 dx = lim
b
b -2
b

2n 2b 2n
n
n

Pela estimativa do resto para o teste da integral temos que, para


1
1
=
= 0, 005 . Assim, o erro cometido menor
n = 10 , R10
2
2.10
200
ou igual a 0,005.

c) Usando os resultados encontrados nos itens anteriores, conclumos


que:
S = R10 + S10 1,197532 + 0, 005 = 1, 202532 .
Tambm, podemos encontrar um limitante inferior usando

1
1
n+1 x3 dx = 2(n + 1)2 .

Ento,

11

dx =

1
1
1
1
=
, e de S10 +
S S10 +
2
2(11)
242
242
200

obtemos que1, 201664 S 1, 202532 .


1
d) Temos de encontrar n tal que Rn 0, 0005 =
.
2000
1
1

Usando o item b), temos que


, o que implica n 2 103
2
2n
2000
ou n 1000 31, 6 .

235

Como n nmero natural, tomamos n = 32 . Assim, a partir de 32 termos, temos uma aproximao de S com preciso menor que 0,0005.
Observao 4.4. A escolha de n = 10 para facilitar o clculo da
soma parcial. claro que a estimativa da soma S poder ser melhorada se n for maior. Mesmo a aproximao de S por S10 pode
ser melhorada com melhor aproximao se S10 . Por outro lado, podemos ter uma estimativa de S10 , sem utilizar calculadora usando
n

S n = a1 + a2 + + an a1 + f ( x)dx , para n = 10 , obtemos


1

10

S10 1 +
1

1
1 3
1
-1
dx =1 +
+ = = 1, 495 .
3
2
x
2.10 2 2 200

Observao 4.5. As desigualdades


n +1

f ( x)dx a1 + a2 + + an a1 + f ( x)dx
1

valem para todo n ( n finito). Ento, mesmo para sries divergentes, possvel fazer uma estimativa das somas parciais S n .

Exerccio resolvido
7) Seja a srie harmnica

n.
n =1

a) Faa uma estimativa para a soma parcial S10 .


b) Quantos termos sero necessrios para formar uma soma
parcial maior que 20?
Resoluo: Na prova da Proposio 4.3 vimos que an = f (n) , onde
1
f ( x) = , a qual, para x 1 , funo contnua, positiva e decresx
cente.
n +1

Ento,

De

1
1
1 1
1
dx S n a1 + dx , sendo S n = 1 + + + ... + , n 1 .
2 3
x
x
n
1

xdx = ln x + c segue que ln(n + 1) S

1 + lnn .

236

a) ln11 S10 1 + ln10 ou 2,397895 S10 2,302585 .


b) Como queremos S n > 20 , ento 20 < S n 1 + lnn ou 20 < 1 + lnn .
Assim, lnn > 19 , e como a funo exponencial crescente, temos
n > e19 .
Usando uma calculadora encontramos e19 = 178.482.301 = N , ou
seja, preciso pelo menos esse nmero de termos para formar
uma soma parcial da srie harmnica que seja maior que 20.
Nota: O valor encontrado mostra a lentido com que as somas parciais da srie harmnica crescem. Lembre-se de que a srie divergente e, sendo os termos positivos, as somas parciais so crescentes
e tendem ao infinito.

*Resumo do que estudamos at agora


Listamos abaixo as sries que estudamos at agora e sabemos sobre
convergncia ou divergncia.
Tabela 4.1:
Sries convergentes

Soma

Sries divergentes

a
n =1

Srie geomtrica

ar

n -1

, com | r | < 1 ;

n =1

1
;

n =1 n( n + 1)

Srie telescpica

p -Srie

1
, com p > 1 .

p
n =1 n

a
S=
;
1- r

qualquer que lim an no existe ou 0 ;


n

Srie geomtrica

ar
n =1

S =1

n -1

Srie harmnica

p -Srie

n
n =1

n
n =1

, com | r | 1 ;

, com p 1 .

237
Proposio 4.4. Teste de comparao
Sejam

a e b
n

n =1

n =1

duas sries de termos no negativos. Suponha-

mos que existe N tal que an bn , para todo n > N .


a) Se

bn convergente, ento
n =1

b) Se

an divergente, ento
n =1

n =1

b
n =1

convergente.

divergente.

Demonstrao: Sejam An = a1 + a2 + a3 ++ an e
Bn = b1 + b2 + b3 + + bn as ensimas somas parciais de

b
n =1

n =1

, respectivamente.

De an bn , para todo n > N ,


An a1 + a2 + a3 + + aN + bN +1 + bN + 2 + + bn se n > N .

a) Se a srie

n =1

convergente, ento suas somas parciais so

limitadas. Logo, existe um K > 0 tal que


bN +1 + bN + 2 + + bn b1 + b2 + b3 + + bn K , para todo n .
Assim, An a1 + a2 + a3 + + aN + K = L , para todo n > N . Tambm,para n N vale An L ,porque A1 A2 AN AN +1 ,
j que os termos da srie so no negativos. Assim, mostramos que
a sequncia das somas parciais ( An ) limitada superiormente e,

pelo Teorema 4.5, segue que a srie

a
n =1

b) Suponhamos que

a
n =1

convergente.

seja divergente. Para mostrar que

b
n =1

divergente, vamos mostrar que suas somas parciais no so limitadas superiormente, usando o mtodo de prova por contradio.
Suponhamos que as somas parciais Bn so limitadas superiormente.
Como an bn , para todo n > N as somas parciais An tambm sero
limitadas superiormente. Sendo uma srie de termos positivos ou
nulos,

a
n =1

convergente. Mas, isso contradiz a hiptese! Logo, as

somas parciais Bn no podem ser limitadas superiormente, e a srie

b
n =1

divergente.

238
Observao 4.6 Podemos aplicar o teste da comparao para sries
com termos no negativos a partir de uma certa ordem n0 , porque
um nmero finito de termos no altera a convergncia ou divergncia de uma srie (volte Seo 4.7 - Operaes sobre sries). Por
outro lado, preciso incluir todos os termos da srie a partir dessa
ordem.

Na prtica, apenas uma srie an dada para analisar. Para aplicar


n =1

o teste da comparao, a outra srie deve ser escolhida, adequadamente, entre as conhecidas (da Tabela 4.1 ou de uma tabela mais
completa).

Exerccio resolvido
Aplicao do teste da comparao
8) Mostre que a srie

... convergente.
n ! = 1! + 2! + 3! + + n ! +
n =1

Resoluo: Lembrando que n ! = n(n - 1)(n - 2) 2 1 , observe que


1! = 1 , assim:

2! = 2.1 = 2
3! = 3.2! > 2.2 = 22
4! = 4.3! > 2.22 = 23

n ! > 2n -1


Vamos mostrar que a desigualdade vale para todo n > 2 .
De fato, supondo vlido, para n , que n ! > 2n -1 , vamos mostrar para n + 1 .
(n + 1)! = (n + 1)n ! > 2.2n -1 = 2n (n > 2 implica que n + 1 > 2) .
Incluindo n = 1 e n = 2 , n ! 2n -1 ou 2n -1 n ! , para todo n .

1
1
1
Logo,
n -1 , para todo n . Como n -1 uma srie geomtrin! 2
n =1 2

1
ca convergente, pelo teste da comparao a srie convergente.
n =1 n !
Nota: O fato de a srie

n!
n =1

ser convergente significa que

sua soma s um nmero real. Adicionando uma unidade,

1
1 1 1
1
1 + = 1 + + + + + + , temos a srie e (e o nmero real
1! 2! 3!
n!
n =1 n !

239
conhecido como nmero de Euler). Note que e > 2 , e como a srie
geomtrica tem soma 2, teremos que 2 < e < 3 .
Sabemos - e possvel provar - que e um nmero irracional. O valor de e, correto at a quinta casa decimal, 2,71828 e ser calculado
no Captulo 5 (Srie de Taylor). Pelo que veremos, voc poder calcular o valor com quantas casas decimais desejar (ou saber o que
est por trs da calculadora)! (e 2, 7182818284590452) .
Observao 4.7. O teste da comparao se aplica a sries com termos
no negativos

a e b
n =1

n =1

, tal que an bn , para todo n > N , N .

bn convergente, ento

a) Se

n =1

n =1

convergente (A srie dos

termos maiores convergente implica que a srie dos termos


menores convergente.);
a)

an convergente no implica
n =1

b) Se

n =1

divergente, ento

b
n =1

n =1

convergente;

divergente (A srie dos ter-

mos menores divergente implica que a srie dos termos maiores divergente.)
b)

bn divergente no implica
n =1

a
n =1

divergente.

1 1

para todo n 1 . A srie harmnica


n2 n
1
vergente e a p-srie 2 convergente ( p = 2) .
n =1 n

Por exemplo,

di-

n =1

No prximo teste de comparao, a condio an bn substituda


pelo limite do quociente de an por bn .
Proposio 4.5. Teste de comparao no limite
Sejam

an e
n =1

an
.
n b
n

o lim

b
n =1

duas sries de termos positivos. Calculemos

240
an
= l e 0 < l < , ento as sries dadas (as duas)
bn
so ao mesmo tempo convergentes ou divergentes.

a) Se o lim

a1) Se o limite l = 0 , ento a srie

n =1

srie an convergente.

convergente implica a

n =1

an
= + , ento a srie
n b
n

b) Se o lim

a
n =1

b
n =1

divergente implica a srie

divergente.

Demonstrao:
a) Usando a definio de limite, dado > 0 , existe N tal
an
a
que, para todo n > N temos
- l < ou < n -l < ,
bn
bn
an
ou ainda, l <
< l + . Para < l , em particular, temos l > 0 ,
bn
e, como bn > 0 , resulta que 0 < (l )bn < an < (l + )bn , para todo
n > N . Se

b
n =1

convergente, a srie

(l + )b

convergente

n =1

(veja o Teorema 4.4), ento an convergente. Se


gente, a srie

(l - )b

n =1

n =1

divergente, ento

n =1

b
n =1

diver-

divergente.

an
a
< ou - < n < , para todo n > N . De
bn
bn
bn > 0 segue que - bn < an < bn , para todo n > N . Como

a1) Se l = 0 temos

o teste de comparao se aplica a sries com temos no negativos, podemos apenas concluir que se

n =1

ento a srie

a
n =1

convergente,

convergente.

b) Pela definio de limite infinito, dado M > 0 , existe N tal que


an
> M , para todo n > N . Isso implica an > Mbn e, pelo teorema da
bn

comparao, se bn divergente, ento an divergente.


n =1

n =1

241

Exerccio resolvido
Aplicao do teste da comparao no limite.
9) Verifique se a srie convergente ou divergente usando o teste
da comparao:

1
a) n
.
n =1 2 + 1

1
A srie dada convergente porque n convergente e
n =1 2
n
n
a
1 2
2
1
lim n = lim n
= lim n
= lim
= 1.
n b
n 2 + 1 1
n 2 + 1
n
1
n
1+ n
2
1
1
n
n
Note que 2 + 1 > 2 , para todo n, assim,
< n
n
2 +1 2
e, tambm, podemos usar o teste de comparao anterior para concluir que a srie dada convergente.
b)

n +1 .
n =1

Essa srie parecida com a anterior: n + 1 > n implica


1
1
< , para todo n .
n +1 n

1
Mas, como divergente, no possvel aplicar o prin =1 n
meiro teste de comparao para concluir que a srie dada
divergente.
a
n
1 n
= lim
= 1 , pelo Teorema de
Como lim n = lim
n b
n n + 1 1
n n + 1
n

1
comparao no limite podemos concluir que
din =1 n + 1
vergente.
c)

ln n

(note que n 2 ).

n=2

1
, n = 2,3, 4, . Para descobrir bn observe que
ln n
1
lne = 1 e lnn > 1 para n > 3 . Assim, para bn = , calculemos
n
an
n
1 n
1
lim = lim
= lim
= lim
= lim n = + . Como
n b
n ln n 1
n ln n
n (1/ n )
n
n

1
divergente, ento a srie dada divergente.

n =1 n

Seja an =

242
Os prximos critrios de convergncia de uma srie no so
de comparao com uma outra srie conhecida. As regras
envolvem apenas os termos da srie, mas a base dos resultados ainda uma srie conhecida: a srie geomtrica.
Sabemos que

ar

convergente se | r | < 1 , e divergente se

n =1

| r | 1 ,ou seja, a convergncia ou no depende da razo r . Obsera


ve que r = n +1 , constante, para todo n , onde an = ar n o termo
an
a
geral da srie. Mas, nem sempre a razo n +1 constante. Por
an

a
n!
1 n!
1
1
exemplo, para
temos n +1 =
.
=
=
(n + 1)! 1 (n + 1)! n + 1
an
n =1 n !
Uma extenso do resultado vlido para a srie geomtrica
o Teste da Razo.
Proposio 4.6. Teste da Razo (Critrio de LAlembert)
Seja

a
n =1

uma srie de termos positivos (an > 0 , para todo n ) .

an +1
= l < 1 , ento a srie convergente.
an
a
a
2) Se lim n +1 = l > 1 ou lim n +1 = , ento a srie divergente.
n a
n a
n
n
an +1
Observao 4.9. Se lim
= 1 nada se pode concluir,ou
n a
n
seja, a srie pode ser convergente ou divergente. Por

1
exemplo: Seja a p -srie p . Para qualquer p temos
n =1 n
1) Se lim

an +1
np
np
1
n
lim
= lim
= lim
= lim
= 1.
n a
n ( n + 1) p 1
n ( n + 1) p
n n + 1

Mostramos

na Proposio 4.3 que a p -srie convergente para p > 1 e divergente para p 1 .


Demonstrao da proposio:
a
a
Se lim n +1 = l , ento dado > 0 , existe N tal que n +1 - l <
n a
an
n
a
ou - < n +1 - l < para todo n N .
an

243

1) Supondo l < 1 seja r um nmero tal que l < r < 1 . (Pergunta:


Existe um tal nmero r ? D um exemplo!) Para = r - l > 0 , vale
an +1
a
- l < r - 1 , em particular, n +1 - l < r - l , para todo n
a
an
an +1 n
a partir de algum N . Ento
< r , quando n N e assim,
an
aN +1 < raN
aN + 2 < raN +1 < rraN = r 2 aN
aN +3 < raN + 2 < rr 2 aN = r 3 aN

aN + m < raN + m -1 < rr m -1aN = r m aN

Consideremos a srie

b
n =1

, onde bn = an para n = 1, 2, , N e

bN +1 = raN , bN + 2 = r 2 aN , , bN + m = r m aN , ,

b
n =1

ou

seja,

= a1 + a2 ++ aN + raN + r 2 aN ++ r m aN + .

A srie geomtrica aN r m convergente porque r < 1 , ento a srie

m =1

bn convergente.
n =1

Como an bn , para todo n , pelo Teste da Comparao (Proposio


4.6.3)

a
n =1

convergente.

2) Se l > 1, l 1 > 0 e de modo anlogo primeira parte, conclumos


a
que n +1 > 1 a partir de algum ndice N .
an
a
A mesma desigualdade n +1 > 1 , para todo n N , N , vale se
an
an +1
lim
=.
n a
n
an +1
> 1 , para todo n N , segue que aN < aN +1 < aN + 2 < .
an
Logo, lim an no pode ser zero e pelo Teste do ensimo termo (Teo-

De

rema 4.3), a srie divergente.

244
Exemplo 4.10. Aplicao do Teste da Razo

1
Usamos, acima, a srie
para mostrar que o quociente
n =1 n !
an +1
1
=
no constante.
an
n +1
1
Como lim
= 0 < 1 , a srie convergente, o que est de acordo
n n + 1
com que mostramos usando o Teste da Comparao.

Exerccio resolvido
10) Analise as seguintes sries.
a)

n !n !

(2n)! ;
n =1

b)

x n +1
, x>0.

n
n =1 3

Resoluo:
a) Seja an =

n !n !
e
(2n)!

an +1 (n + 1)!(n + 1)! (2n)!


(n + 1)n !(n + 1)n !(2n)!
(n + 1)(n + 1)
=
=
=
.
(2(n + 1))! n !n ! (2n + 2)(2n + 1)(2n)!n !n ! (2n + 2)(2n + 1)
an
an +1
(n + 1)(n + 1)
1 + 1/ n
1
= lim
= lim
= < 1.
n a
n 2( n + 1)(2n + 1)
n 2(2 + 1/ n)
4
n

Ento, lim

Logo, pelo Teste da Razo a srie dada convergente.


an +1 x ( n +1) +1 3n
x n +1 x3n x
b) Para x > 0 ,
= n +1 . n +1 = n n +1 = , para todo n . Enan
x
3
3 3x
3
a
x
to, lim n +1 = .
n a
3
n
x
Pelo Teste da Razo, a srie convergente se < 1 ou x < 3 e
3
divergente se x > 3 .
Se x = 3 ,

x n +1
= 3 = 3 + 3 + 3 + , divergente.

n
n =1 3
n =1

Pelos exemplos acima possvel notar que o Teste da Razo eficaz quando nos termos de uma srie aparecem fatoriais ou ex-

245
presses elevados ensima potncia. O mesmo acontece com o
prximo critrio e ambos podem fornecer a soluo do problema
de convergncia de uma srie, quando a aplicao dos teoremas
gerais difcil.
Proposio 4.7. Teste da Raiz (Critrio de Cauchy)
Seja

a
n =1

uma srie com an 0 , para n N , para algum N .

1) Se lim n an = l < 1 , ento a srie convergente.


n

2) Se lim n an = l > 1 ou lim n an = , ento a srie divergente.


n

Observao 4.9. Se lim n an = 1 no se pode concluir que a srie


n
convergente ou divergente.
Omitimos a prova do Teste da Raiz porque anloga prova do
Teste da Razo.
Nota: Existe ainda o critrio n r an , que no colocaremos aqui, que

pode fornecer um resultado para a srie

a
n =1

no caso que os Testes

da Razo e da Raiz falham, isto , quando o limite igual a um. Mas


claro que isso no resolve totalmente o problema de convergncia
de sries com termos positivos.
Tarefa: Pesquise e encontre exemplo de srie que ainda no se sabe
se convergente ou divergente!

Exerccio resolvido
11) Decida se as sries so convergentes ou divergentes, aplicando
o Teste da Raiz.
3n + 2
a)

n =1 4n + 1

b)

n3

n
n =1 3

3n
c) 3
n =1 n

246
Resoluo:

3n + 2
a) Seja an =
(quociente elevado potncia n ).
4n + 1
3n + 2 3
= < 1 e assim pelo Teste da Raiz, a srie
Ento lim n an = lim
n
n 4n + 1
4
dada convergente.
3

n3
n3
nn
b) No caso an = n , temos o limite lim n n = lim .
n
n 3
3
3
Para calcular o limite calculemos primeiro,
1
3
3

3
n
n
ln lim n = lim ln(n ) = lim lnn = lim 3 n = 0 ,
n n
n
1
n n
3

o que implica que lim n n = e0 = 1 .


n

Assim, lim n an =
n

vergente.

1
< 1 , e pelo Teste da Raiz a srie
3

n3
con
n
n =1 3

3n
3n
3
n
c) Agora an = 3 e lim 3 = lim 3 = 3 > 1 . Ento pelo Teste da Raiz

n
n
n
n
nn
n

3
a srie 3 divergente.
n =1 n

4.5.1 Exerccios
1) Use o Teste da Integral para mostrar que as seguintes sries
so convergentes:
a)
b)

arctgn
2
+1
n =1

n(ln n)
n=2

2) Obtenha uma estimativa do resto Rn na forma | Rn | , para


cada uma das seguintes sries convergentes:
1
a) 1,1
n =1 n

b)

(n !)
n =1

3) Use um dos testes de comparao para determinar quais das


sries so convergentes e quais so divergentes.

247

a)

n=2

1
n -1

c)

n=2

sen 2 (2n - 1)

n2
n =1

b)

ln n

d)

ln n
3
n =1 n

4) Use o Teste da Razo ou da Raiz para determinar quais sries


so convergentes.
n
a)

n =1 3n + 1

n2e- n

b)

n =1

c)

nn
d) n
n =1 2

e)

(ln n)
n=2

n!

5
n =1

5) Mostre que nem o Teste da Razo nem o Teste da Raiz forne


1
cem informaes sobre a convergncia da p-srie p .
n =1 n

4.6 Sries alternadas e sries


absolutamente convergentes
Na seo anterior estudamos alguns testes de convergncia que podem ser aplicados apenas s sries com termos no-negativos. Agora, vamos estabelecer alguns critrios de convergncia para lidar
com sries sem essa restrio.
J vimos exemplos de sries que tem termos negativos:
1)
2)

(-1)
n =1

= -1 + 1 - 1 + , divergente.
n -1

1 1 1
-1

= 1 - + - + , srie geomtrica de razo


2 4 8
n =1 2
1
2
r = - < 0 , convergente, com soma S = .
2
3

Essas sries pertencem a uma classe importante que estudaremos a seguir.

248

4.6.1. Sries alternadas


Uma srie

a
n =1

na qual os termos an so alternadamente positivos

e negativos denominada srie alternada.


Mais exemplos:

1 1 1
(-1) n -1
3) Srie harmnica alternada: 1 - + - + =
2 3 4
n
n =1

4) 1 - 2 + 3 - 4 + = (-1) n +1 n
n =1

1 2 3 4 5
n
5) - + - + - + = (-1) n
2 3 4 5 6
n +1
n =1

Notao: Observe que an =


no Exemplo 3).

(-1) n -1
1
= (-1) n -1 = (-1) n -1 bn , (com bn > 0
n
n

an = (-1) n +1 n = (-1) n +1 bn , (com bn > 0 no Exemplo 4).


an = (-1) n

n
= (-1) n bn , (com bn > 0 no Exemplo 5).
n +1

Usamos a potncia n 1 (ou n + 1 ) se a1 > 0 e n se a1 < 0 .


Mas lembrando que um nmero finito de termos no altera a convergncia ou no de uma srie, adotamos a notao
onde bn > 0 para todo n , para uma srie alternada.

(-1)
n =1

n -1

bn ,

Teorema 4.6. Teste da Srie Alternada (Critrio de Leibniz)


A srie alternada

(-1)
n =1

n -1

bn = b1 - b2 + b3 - b4 + , bn > 0 , conver-

gente se satisfaz as duas seguintes condies:


i) bn bn +1 , para todo n , ou seja, (bn ) uma sequncia decrescente e
ii) lim bn = 0 .
n

Nota: O teste continua vlido se a sequncia (bn ) decrescente a


partir de um certo termo de ndice N (grande).

249

Demonstrao. As somas parciais da srie alternada so:


S1 = b1 ,

S 2 = b1 b2 ,

S5 = b1 b2 + b3 b4 + b5 ,

S3 = b1 b2 + b3 ,

S 4 = b1 b2 + b3 b4 ,

S6 = b1 b2 + b3 b4 +b5 b6 , etc...

Usando a propriedade associativa (para soma de finitos termos), as somas parciais de ordem par, n = 2k , so
S 2 k = (b1 b2 ) + (b3 b4 ) + + (b2 k -1 b2 k ) e as de ordem mpar,
n = 2k + 1 , so S 2 k +1 = S 2 k + b2 k +1 .

Analisemos a subsequncia ( S 2 k ) . Da condio i) bn bn +1 ou


bn bn +1 0 , para todo n , segue que S 2 k 0 e que S 2( k +1) = S 2 k + 2 S 2 k .
Assim, ( S 2 k ) uma sequncia de termos positivos e crescente. Por outro lado, usando novamente a propriedade associativa,
S 2 k = b1 - (b2 - b3 ) - (b4 - b5 ) -- (b2 k - 2 - b2 k -1 ) - b2 k , o que mostra
que S 2 k b1 , para todo k e assim a sequncia limitada superiormente. Logo, a sequncia ( S 2 k ) convergente e seja S = lim S 2 k .
k

Para a sequncia ( S 2 k +1 ) , da condio ii) lim bn = 0 , segue que


n
lim S 2 k +1 = lim( S 2 k + b2 k +1 ) = S + 0 = S .
k

Se no foi provado em Clculo I, no difcil provar que se lim S 2 k = S


k

e lim S 2 k +1 = S ento lim S n = S . (Prove como exerccio!). Portanto, a


n
k
srie alternada convergente.

Esboo do comportamento das somas parciais na


reta real:
S1 = b1 > 0
S 2 = b1 - b2 0 pois, b1 b2 e
de S1 )

S 2 = S1 b2 < S1 ( S 2 fica esquerda

S3 = b1 b2 + b3 = S 2 + b3 , ento S3 0 e S3 S 2 ( S3 direita de S 2 ).
Assim, por diante.

250

+b1
b2
+b3
b4

S2

S4

S3

S1

Figura 4.6: As somas parciais da srie alternada que satisfaz as condies do teste.

Como lim S 2 k = S e lim S 2 k +1 = S , S est entre as somas parciais


k

de ordem par e as de ordem mpar ( S 2k esquerda de S e S 2 k +1


direita). A cada etapa a distncia entre S n e S n +1 fica menor porque
lim bn = 0 .
n

Observao 4.10. A condio ii) necessria para a srie alternada


ser convergente, mas a condio i) no , como mostra o seguinte
exemplo:
1 1 1 1 1 1
1 1
- + - +
- + + n - n + no satisfaz a condio i).
3 2 9 4 27 8
3 2
Mostre!
1
Mas a srie convergente e sua soma S = - .
2
Mostre!
Exemplo 4.11. Aplicao do Teste da Srie Alternada
A srie harmnica alternada
De fato, para todo n , an =
De n < n + 1 temos

(-1) n -1
convergente.

n
n =1

(-1) n -1
1
1
= (-1) n -1 , com bn = > 0 .
n
n
n

1
1
>
, ou seja, bn > bn +1 , para todo n .
n n +1

251

1
= 0 . Ento, como as condies do teste esn
to satisfeitas, a srie harmnica alternada convergente.

Alm disso, lim bn = lim


n

Nota: Veremos mais tarde que a soma da srie harmnica alternada


ln2 (logaritmo natural de dois).
Mais geral: p -Sries Alternadas
(-1) n -1
1
1
1
= 1 - p + p - p + convergente
A p -srie alternada
p
2
3
4
n
n =1
se p > 0 .

A prova anloga. Note que se p = 0 , a srie

(-1)

n -1

divergente e

n =1

se p < 0 , a p -srie divergente pelo Teorema do ensimo termo.

Exerccio resolvido
(-1) n 2n
12) Determine se a srie
convergente ou no.
n =1 3n - 1
Resoluo: A srie dada alternada e escrevemos

(-1) n 2n
2n
2n

=
(-1) n -1
, onde bn =
> 0 , para todo n .

3n - 1
3n - 1
n =1 3n - 1
n =1

2n
2
2
= lim
= 0.
n 3n - 1
n
1 3
3n
Ento a condio ii) do Teste da Srie Alternada no est satisfeita.

O limite lim

Analisaremos a convergncia ou no da srie, calculando o limite do


n -simo termo da srie:
(-1) n 2n
.
lim an = lim
n
n 3n - 1
O limite no existe, assim pelo teste do termo geral, a srie divergente.

Estimativas de Sries Alternadas


Seja

(-1)
n =1

n -1

bn , bn > 0 para todo n , convergente pelo Teste da

Srie Alternada.

252

As somas parciais S 2 k +1 = S 2 k + b2 k +1 e S 2 k = S 2 k -1 b2 k podem ser escritas como S n +1 = S n bn +1 . Como S =


ento, | S S n | bn +1 .

(-1)
n =1

n -1

bn est entre S n e S n +1

Exemplo 4.12. Vamos testar a desigualdade | S S n | bn +1 em uma


srie cuja soma conhecemos:
n

1 1 1 1 1
1
1
1
-1
+
-

= 1- + - + - + 2 4 8 16 32 64 128 256
n =1 2

2
Usando uma calculadora S8 = 0, 6640625 . Sabemos que S = .
3
Ento
2
|S S8 | = - 0, 6640625 = 0, 002604166 .
3
1
Pela estimativa acima, | S - S8 | b9 =
= 0, 00390625 .
256

Exerccio Resolvido
(-1) n
13) Mostre que a srie
convergente e encontre a soma
n!
n =1
com preciso de trs casas.

Resoluo:
A srie

(-1) n -1
(-1) n
1 1
1 1

=
+
+
=
+
=
1
1
..

2! 3!
n!
n!
2! 3!
n =1
n =1

1
sendo bn = > 0 para todo n . Primeiro vamos mostrar que a
n!
srie convergente.
1
1
i) (n + 1)! = (n + 1) n ! > n ! , implica que
>
. Logo, bn > bn +1
n ! (n + 1)!
para todo n .
ii) 0 < bn =

1 1
1
1
< , pois n ! > n . Como lim = 0 ento lim = 0 .
n n
n n !
n! n

Ento pelo Teste da Srie Alternada a srie dada convergente.


n

(-1) k
(-1) n
Seja S =
e seja S n =
a ensima soma parcial. Quek!
n!
k =1
n =1
remos encontrar n tal que S S n bn +1 0, 0001 .

253
1 1
1
1
1
= = 0,5 , b3 = =
= = 0,1666 ,
2! 2
3! 3 2! 6
1 1
1
1
b4 = =
= 0, 04166 , b5 = =
= 0, 00833 ,
4! 24
5! 120
b1 = 1 , b2 =

b6 =

1
1
1
1
=
= 0, 001388 , b7 = =
= 0, 00019841
6! 720
7! 5040

Ento, n + 1 = 7 e n = 6 . Assim,
1 1 1
1
1
S6 = -1 + - + +
= -0, 6318
2 6 24 120 720
aproxima S com preciso de trs casas decimais.

4.6.2 Sries absolutamente convergentes


Continuamos considerando sries que tm termos negativos, mas
agora a troca dos sinais algbricos irregular.
Exemplo 4.12 Seja a srie
Lembremos que

Seja

se

+ 2k < <
+ 2k , k . Como n 2 > 0 , para todo n ,
2
2

cos < 0 se
an =

cos > 0

cos n cos1 cos 2 cos 3


= 2 + 2 + 2 + .
2
1
2
3
n =1 n

- + 2k < < + 2k , k e
2
2

cosn
< 0 para alguns ndices n , mas no de modo alternado.
n2

a
n =1

, uma srie qualquer (sem restrio de sinais algbri-

cos nos termos an ) e consideremos a srie cujos termos so os valores

absolutos dos termos desta srie, isto , | an | = | a1 | + | a2 | + | a3 | + .


n =1

Definio 4.2. Uma srie


srie

| a
n =1

a
n =1

absolutamente convergente se a

| convergente.

Exemplo 4.13.
a) claro que, toda srie de termos no-negativos convergente
absolutamente convergente.
b) A srie geomtrica

n -1

, com 1 < r < 1 , absolutamente

n =1

convergente (na verdade convergente).

254
n -1

1 1 1
-1

= 1 - + - + absoluta2 4 8
n =1 2
mente convergente porque a srie dos valores absolutos
n -1

1 1 1
1

= 1 + + + + convergente.
2 4 8
n =1 2
Por

exemplo,

A convergncia absoluta importante por dois motivos: primeiro


porque como |an | 0 , para todo n ,

| a

n =1

| uma srie de termos

positivos para o qual temos testes eficazes para verificar sua


convergncia. O segundo motivo dado pelo prximo teste.
Proposio 4.8. Teste da Convergncia Absoluta
Se uma srie absolutamente convergente ento ela convergente,
ou seja, se

| an | convergente, ento
n =1

Demonstrao: Se a srie

| a
n =1

a
n =1

convergente.

| convergente ento pelo Cri-

trio de Cauchy (Teorema 4.1), dado > 0 , existe N tal que


| an +1 | + | an + 2 | + + | an + p | < , quaisquer que seja n > N , e p .
De | an | 0 , para todo n ,
| an +1 | + | an + 2 | + + | an + p | =| an +1 | + | an + 2 | + + | an + p | an +1 + an + 2 + + an + p
e temos |an +1 + an +2 + + an + p | < , quaisquer que seja n > N ,
e p . Logo, pelo mesmo critrio a srie

a
n =1

convergente.

Observao 4.11. A recproca do teste falsa: nem toda srie convergente absolutamente convergente.
Exemplo 4.14. clssico A srie harmnica alternada convergente, com mostramos no Exemplo 4.11 (acima), mas ela no ab
1

solutamente convergente porque a srie harmnica


no
n =1 n
convergente.
Nota: Se uma srie

a
n =1

convergente, mas no absolutamente

convergente, dizemos que ela condicionalmente convergente.

255

Exerccio Resolvido
14) Analise a srie

cos n cos1 cos 2 cos 3


= 2 + 2 + 2 +
1
2
3
n =1 n2

cos n
a srie dos valores absolutos dos termos.
n2
n =1
cos n cos n
1
=
2 . Como an > 0 e
De |cos n| 1 segue que an =
2
2
n
n
n
1
bn = 2 > 0 , para todo n , podemos usar o Teorema da Comparao
n

1
cos n
e do fato de 2 ser convergente, segue que 2 convern
n =1 n
n =1
gente. Assim, a srie dada absolutamente convergente e pelo Teste
da Convergncia Absoluta ela convergente.

Resoluo. Seja

Exemplo 4.15. p -Srie Alternada


Abaixo do Exemplo 4.10, observamos que a p -srie alternada
(-1) n -1

np
n =1
vergente

n
n =1

convergente
se p 0 .

convergente se

Como

se
a

p>0

an bn

p > 1 e divergente se p 1 ,

dip -srie
ento

(-1) n -1
absolutamente convergente se p > 1 e condicionalmente

np
n =1
convergente se 0 < p 1 .

Por exemplo;

1
1
1
(-1) n -1
1+
+ = 1/2 condicionalmente convergente.
n
2
3
4
n =1
1-

1
1
1
(-1) n -1
+
+
=

absolutamente convergente.

23/2 33/2 43/2


n3/2
n =1

Nota: Pelo Teste da Convergncia Absoluta, para provar que uma


srie convergente, basta demonstrar que absolutamente convergente. Para isso, podemos usar os critrios de convergncia para sries com termos positivos porque todo critrio de convergncia para
sries com termos positivos um critrio de convergncia absoluta.
Por exemplo,

256
Proposio 4.6. Teste da Razo (Critrio de LAlembert)
Seja a srie

a
n =1

, com an 0 , para todo.

an +1
= l < 1 , ento a srie absolutamente convergente e
n a
n
portanto convergente.
a
a
2) Se lim n +1 = l > 1 ou lim n +1 = + , ento a srie divergente.
n a
n a
n
n

1) Se lim

3) Se lim

an +1
= 1 , nada se pode concluir.
an

Proposio 4.7. Teste da Raiz (Critrio de Cauchy)


Seja

a
n =1

uma srie qualquer.

1) Se lim n an = l < 1 , ento a srie absolutamente convergente e


n
portanto convergente.
2) Se lim n an = l > 1 ou lim n an = + , ento a srie divergente.
n

3) Se lim n an = 1 , o teste falha.


n

Exemplo 4.16. Podemos generalizar o Exerccio resolvido 4.12 b) do


seguinte modo: seja a srie

n c

r n

, onde c e r so nmeros reais


r

n =1

quaisquer porm fixos. Calculando o limite lim n n r c n = lim c n n = | c |


n

r
n

( lim n = 1 , calculado no referido exerccio para r = 3 ). Ento pelo


n

Teste da Raiz a srie

n c

r n

(absolutamente) convergente quando

n =1

|c| < 1 . claro que , se|c| 1 , a srie divergente porque o termo geral no a tende a zero.
Nota: Aplicamos o Teste da Raiz, mas obteramos o mesmo resultado usando o Teste da Razo. Em geral mais fcil calcular o limite
da razo do que o da raiz porque, quando efetuamos o quociente
quase sempre fazemos simplificaes, mas o Teste da Raiz mais
eficaz do que o da Razo. A ltima afirmao comprovada pelo
a
fato que se lim n +1 existe ento lim n an existe e os dois limites so
n
n a
n
iguais. Tambm h exemplos que comprovam a afirmao, mas deixamos para o leitor pesquisar.

257

4.6.1 Exerccios
1) Use o Teste da Srie Alternada para verificar se as sries so
convergentes ou no.

(-1) n
(-1) n
a)
d)
n +1
n =1 n + 2
n =0
b)
c)

n
(-1) n +1

10
n =1

(-1) e
n

e)

( -1)

n +1

n=2

1
ln n

-n

n =0

2) Verifique se as sries so absolutamente convergentes:


a)

(-1)n+1
n =1

b)

(-1)n+1
n =1

c)

(-1)

n +1

n=2

n3
n5 + 1

d)

2+n
3+ n

e)

1
n ln n

f)

(-1) n -1

2
n =1 n + 2n + 1

(-1)n+1
n =1

102 n -1
(2n - 1)!

cos(n )
n n
n =1

3) Quais das sries convergentes no Exerccio 16) no so absolutamente convergentes, ou seja, so condicionalmente convergentes? Justifique sua resposta.
4) Analise, justificando, quais das sries so convergentes e quais
so divergentes. (Lembre-se de que pode existir mais de uma
justificativa.)
a)
d)
g)
j)

-n

b)

n =1

n2

3
n =1 n + 1

arctgn

2
n =1 1 + n

nsen n
n =1

n
1
e) 2
n = 2 n ln n
n =1

h)

n!

n
n =1 n

c)
f)

n=2
2

n2 - 1

n!
n =1

i)

(ln n) n

nn
n=2

258

Respostas dos exerccios


4.2.1 Exerccios
1) a) an =

3
xn
rn
a
=
a
=
;
b)
;
c)
n
n
n10
5n
(n + 1)!
7(1 - ( 12 ) )
n

3) a) S n =

1 - 12

, S = 14

b) S n =

1 + (-1) n -1 2n
, divergente
3

c) S n =

5
5
5
, S=
2 n+2
2

4) a) S =

7
21

; b) S = ; c) divergente; d) S =
10
2
1+

5) a) x no intervalo ]2, 4[ , S =
b) x tal que 1 < x < 1 , S =

4
;
4- x

1
;
1 - x2

2
;
1- x
1
1
1
1
d) x tal que - < x < < x < , S =
.
5
5
5
1 - 5x

c) x em ] 3, 1[ , S =

6) 28m
4
8) t = ln
5

4.3.1 Exerccios
1) a) convergente; b) divergente; c) divergente; d) convergente ;
e) convergente; f) divergente ; g) divergente .

259

4.4.1 Exerccios
1) a) S =

1
23
; b) S =
; c) S =1 .
2
10

4.5.1 Exerccios
2) a) Rn

10
1
; b) Rn
0,1
n
n

b) convergente (comparao:

n =1

c) divergente (comparao com

n =1

3) a) convergente (comparao com

);

3
2

);

);

n =1

d) convergente (comparao com

n =1

4) a) convergente , l =

).

1
1
; b) convergente , l = ;
3
e

c) e d) divergentes , l = + .

4.6.1 Exerccios
1) a) convergente; b) divergente; c) convergente; d) convergente;
e) convergente.
2) a) absolutamente convergente (compare com

n =1

b) divergente ( lim an = 1 ) ;
n

);

c) divergente (Teste da Integral) ;

d) absolutamente convergente (compare com

n
n =1

);

e) absolutamente convergente (Teste da Razo: l = 0 ) ;


1

f) absolutamente convergente an = 3/2 ; p - srie convergente


n

3) a) ; d) e e)

4) a) convergente (srie geomtrica, r =


b) divergente (p-srie , p =
c) divergente desde que
d) divergente ( an >

1
);
2
1

n2 - 1

1
) ;

1
;
2n 2/3

1
) ;
2n

e) convergente (compare an com

1
) ; f) convergente ( l = 0 ) ;
n2

g) convergente (Teste da Integral) ;


i) convergente ( l = 0 ) ;
termo).

h) convergente , =

1
;
e

j) divergente (Teste do ensimo

Captulo 5
Sries de Potncias

Captulo 5
Sries de Potncia
No resumo da histria das sries, mencionamos que a investigao das sequncias e sries de funes teve incio na
segunda metade do sculo XVII, com Newton e Leibniz, que
desenvolveram representaes de sries de algumas funes. Em 1669, Newton, com menos de 30 anos, descobriu
que a funo (1 + x)c , com c real arbitrrio, pode ser escrita como uma srie de potncias de x .
As sries de Taylor e de Maclaurin no foram inventadas
por, respectivamente, Brook Taylor (1685-1731) e Colin
Maclaurin (1698-1746). James Gregory (1638-1675) j tinha trabalhado com a srie de Taylor e publicado a srie
de Maclaurin para muitas funes trigonomtricas antes
deles terem nascido. Taylor desenvolveu, sem conhecer o
trabalho de Gregory, um mtodo baseado em clculo para
gerar representaes de funes em sries. O livro que Maclaurin escreveu em 1742 popularizou as representaes
de funes em sries e, embora ele nunca tenha afirmado
que as tinha descoberto, a srie de Taylor centrada em zero
ficou conhecida como a srie de Maclaurin.

5.1 Introduo
No Exerccio 3 de aplicao da srie geomtrica vimos que
1 + x + x 2 + x3 + =

1
, se 1 < x < 1 .
1- x

(1)

Considerando x varivel, a expresso esquerda da igualdade (1)


define a funo
1
x
, cujo domnio \{1} .
1- x
A correspondncia x 1 + x + x 2 + x3 + define uma funo real no
intervalo ] 1,1[ , pois para cada x nesse intervalo a srie conver-

264
gente, ou seja, 1 + x + x 2 + x3 + = lim(1 + x + ... + x n ) existe e um nn

mero real e sabemos que se o limite existe ele nico.


A igualdade acima (1), ou a igualdade das duas funes, vlida
apenas no intervalo ] 1,1[ e nesse domnio a srie de potncias de
1
x representa a funo f ( x) =
.
1- x
Reescrevendo a igualdade (1) acima, trocando de lado as expresses,
1
como uma srie de potnpodemos escrever a funo f ( x) =
1- x
cias de x :
1
= 1 + x + x 2 + x3 + , se 1 < x < 1 .
1- x
Note que evitamos a notao sigma

para lembrar mais uma

n =0

vez a observao feita quando estudamos as sries geomtricas. A


srie geomtrica a + ax + ax 2 + ax 3 + convergente, para todo x
tal que | x | < 1 , em particular, para x = 0 converge para o valor a .
Quando usamos a notao sigma

ax

, para x = 0 e n = 0 temos 00,

n =0

que no um nmero! Para evitar isso, podemos colocar x 0 e

analisar quando x = 0 ou escrever a + ax n . Mas como


n =1

ax

n =0

apenas notao e a srie a + ax + ax 2 + ax 3 + , estabelecemos que

ax
n =0

igual a a , quando x = 0 (e no resultado da substituio

de x por 0 ), e usamos a notao

ax

, sendo x tal que 1 < x < 1 .

n =0

Primeiro estudaremos as sries do tipo

e depois aprendere-

n =0

mos a representar certas funes como sries de potncias de x .

5.2 Srie de potncias e convergncia


1
, se
1- x
1 < x < 1 . Por definio 1 + x + x 2 + x3 + = lim(1 + x + ... + x n ) .

Antes da definio, voltemos ao Exemplo 1 + x + x 2 + x3 + =


n

Note que as somas parciais S n = 1 + x + x 2 + + x n so polinmios.


Mas, do mesmo modo que a srie numrica no uma soma infinita,

265
a srie de potncias de x no um polinmio infinito. Lembrese que pela definio de polinmio as potncias das variveis so
nmeros inteiros no negativos, portanto finito. A forma geral de
um polinmio a uma s varivel dada por c0 + c1 x + c2 x 2 + + cn x n ,
em que os ci so nmeros reais constantes. Alm disso, o domnio
de qualquer funo polinomial todo e no exemplo acima o domnio do polinmio infinito 1 < x < 1 .
Definio 5.1. Uma expresso da forma

c ( x - a)
n

n =0

= c0 + c1 ( x a) + c2 ( x a) 2 + + cn ( x a) n +

denominada srie de potncias em ( x a ) ou srie de potncias


centrada em a (ou ao redor de a). Os nmeros reais c0 , c1 , c2 , , cn ,
so os coeficientes da srie.
O caso particular a = 0 ,

c x
n =0

= c0 + c1 x + c2 x 2 + + cn x n + de-

nominada simplesmente srie de potncias.


Observao 5.1. O caso geral se reduz ao caso particular a = 0 pela
mudana de varivel
y = xa:

cn ( x - a)n = cn y n .
n =0

n =0

(E resultados obtidos para cn x n so facilmente adaptadas para as


sries centradas em a 0 ). n =0
Exemplo 5.1
a) O exemplo da introduo

x
n =0

= 1 + x + x 2 + x3 + uma srie

de potncias (centrada em a = 0 ), com coeficientes


c0 = c1 = c2 = = cn = = 1 .
b) A srie de potncias
n

1
1
1
1
2
n
n

- ( x - 3) =1 + - ( x 3) + ( x 3) + + - ( x 3) +
2
4
2
2
n =0
1
est centrada em a = 3 e os coeficientes so c0 = 1 , c1 = - ,
n
2
1
1
c2 = , , cn = - ,
4
2

266
c) A srie
n

x
x x 2 x3
= 1 + + + +

1! 2! 3!
n =0 n !

uma srie de potncias (centrada em a = 0 ) e os coeficien1


1
tes so c0 = 1 ( 0! = 1 , por definio), c1 = 1 , c2 = , c3 = , ,
2!
3!
1
cn = ,
n!
A primeira questo saber para quais valores de x uma srie de
potncias convergente. Note que para x = a , ( x - a ) n = 0 , para todo
n 1 e ento qualquer srie de potncias da forma

c ( x - a)
n =0

convergente quando x = a . Assim, toda srie de potncias sempre


convergente pelo menos quando x o centro.
Como para cada valor x = k temos uma srie numrica, utilizamos
os resultados do captulo anterior para estudar a convergncia das
sries de potncias.
Exemplo 5.2. O exemplo da introduo
e divergente se |x| 1 .

De modo geral, a srie de potncias

convergente se x < 1

n =0

c( x - a)

, com coeficientes

n =0

cn = c, n, convergente se | x a | < 1 (a 1 < x < a + 1) e divergente


se | xx - aa| 1 ( x a + 1 ou x a 1) .
Observao 5.2. A srie geomtrica base do Exemplo 5.2. Associamos a uma srie de potncias dado uma srie geomtrica se cn = c ,
constante, para todo n . Lembremos que a srie geomtrica uma
srie especial de que temos informao completa em relao convergncia e divergncia.
n

x
x x 2 x3
=
1
+
+ + + .

1! 2! 3!
n =0 n !

Exemplo 5.3. Seja a srie de potncias

xn
, para cada
Para o centro x = 0 a srie convergente. Seja an =
n!
x 0 (positivo ou negativo), e calculemos
x
an +1
x n +1 n !
x
=
=
=
.
n
an
n +1 n +1
(n + 1)! x

267

x
= 0 independentemente do valor de x , pelo Teste
n n + 1
de Razo a srie numrica convergente, qualquer que seja x .
Logo, a srie de potncias dada convergente para todos os nmeros reais.
Como lim

Exemplo 5.4. Consideremos agora a srie de potncias

n! x
n =0

= 1 + x + 2! x 2 + 3! x3 +

Para cada x , escrevemos an = n ! x n , e assim,


an +1
(n + 1)! x n +1
=
= (n + 1) x , x 0 .
an
n! xn
an +1
= + , se x 0 , resultado que pelo Teste da Razo imn a
n
plica que a srie divergente, qualquer que seja x 0 . Ento a srie
de potncias convergente apenas quando x = 0 .
Logo, lim

Nota. Nos dois ltimos exemplos aplicamos o Teste da Razo, mas


devemos estar atentos porque os dois testes, da Razo e da Raiz,
falham quando o limite igual a um!
Exemplo 5.5. Encontrar todos valores de x para os quais a srie de
n

( x - 2)
potncias
convergente.
n
n =1
Resoluo. Sabemos que a srie de potncias convergen( x - 2) n
e
te quando x = 2 . Para x 2 , o termo geral an =
n
an +1
n
n
( x - 2) n +1
x-2 .
=
=
n
an
n + 1 ( x - 2)
n +1
n
x - 2 = x - 2 pelo Teste da Razo, a srie absolun n + 1
tamente convergente, portanto convergente se x satisfaz | x 2 | < 1 ,
e divergente se | x 2 | > 1 . O teste falha quando | x 2 | = 1 , ou
seja, quando x = 1 ou x = 3 . Lembre-se que queremos encontrar todos os valores de x para os quais a srie de potncias convergente.

Como, lim

268
Para x = 1 , temos
n

(1 - 2)
(-1)
=
,

n
n =1
n =1 n

que a srie harmnica alternada, portanto convergente. E se x = 3 ,


n

(3 - 2)
1
=
,

n
n =1
n =1 n

a srie harmnica que divergente.


De | x 2 | < 1 temos 1 < x 2 < 1 , ou 1 < x < 3 . Assim, a srie de
potncias convergente quando x tal que 1 x < 3 .
Nos exemplos acima, encontramos trs diferentes conjuntos de valores de x para os quais as sries de potncias so convergentes: um
intervalo finito (Exemplos 5.2 e 5.5), todo o conjunto dos nmeros
reais (Exemplo 5.3) e o conjunto unitrio cujo elemento o centro da
srie (Exemplo 5.4). Essas so as nicas possibilidades e resultam do
Teste da Razo ou da Raiz.
Teorema 5.1. (Teorema da convergncia para sries de potncias)
Para uma srie de potncias
possibilidades:

c ( x - a)
n =0

dada, existem apenas trs

i) Existe um nmero real R > 0 tal que a srie convergente quando x tal que | x a | < R e divergente quando | x a | > R .
ii) A srie convergente qualquer que seja x .
iii) A srie convergente apenas quando x = a .
Observao 5.3. Se | x a | = R , a srie pode ser convergente ou divergente. Ento, para x = a + R e x = a R , as sries devem ser checadas com algum teste.
O nmero R denominado raio de convergncia da srie de potncias. Por conveno, R = + no caso ii) e R = 0 no caso iii).
O conjunto de todos os valores de x para os quais a srie convergente o intervalo de convergncia.

269
Do item i) do Teorema 5.1 e da Observao 5.3 resultam que o intervalo de convergncia pode ser:
aberto: ]a R, a + R[ ,
semiaberto: ]a R, a + R] ou [a R, a + R[ ,
fechado: [a R, a + R ] .
ii) O conjunto dos nmeros reais pode ser representado por
] , +[ .
Nos exemplos acima temos,
Sries

Raio de convergncia

Intervalo de convergncia

R =1

] 1,1[

R=0

{0}

R = +

] , +[

R =1

[1, 3[

Raio de convergncia

Intervalo de convergncia

R =1

]a 1, a + 1[

R=0

{a}

R = +

] , +[

R =1

[a 1, a + 1[

(srie
geomtrica)

n =0

n! x

n =0

n =0 n !

( x - 2)

n
n =1

Da Observao 5.1, segue que


Sries

c( x - a)

n =0

(srie geomtrica)

n !( x - a)

n =0

( x - a)

n!
n =0

( x - a)

n
n =1

270

Resumo para encontrar o intervalo de convergncia


1 Passo. Aplique o Teste da Razo (ou da Raiz) para encontrar
o intervalo I em que a srie (absolutamente) convergente.
Obtemos, I =]a R, a + R[ ou I =] , +[ ou I = {a} .
2 Passo. Se o intervalo de convergncia I =] , +[ ou
I = {a} , ele est completamente determinado.
3 Passo. Se I =]a R, a + R[ , devemos verificar se a srie convergente ou divergente quando x um dos extremos do intervalo, ou seja, x = a + R e x = a R . Para isso usamos os outros
testes; da comparao, da integral ou da srie alternada.

Exerccio resolvido
1) Encontre o raio de convergncia e o intervalo de convergncia
das seguintes sries:
a)

(-1)

2 n-1

n -1

n =1

(-3) n x
b)
n +1
n =0

x
x3 x5 x 7
= x - + - + ...
2n - 1
3 5 7
n

Resoluo.
a) Seja an = (-1)

n -1

x 2 n -1
. Ento,
2n - 1

an +1
(-1)( n +1) -1 x 2( n +1) -1 2n - 1
(-1)(2n - 1) 2 2n - 1 2
x =
x , n 1.
=
=
n -1 2 n -1
an
2(n + 1) - 1 (-1) x
2n + 1
2n + 1
Logo, lim

an +1
2n - 1 2
x = x2 .
= lim
n

an
2n + 1

Pelo Teste da Razo, a srie absolutamente convergente quando


x 2 < 1e divergente quando x 2 > 1 .Note que x 2 < 1 vale se 1 < x < 1.
Assim, a srie absolutamente convergente, logo convergente, para
x tal que 1 < x < 1 e divergente para x > 1 ou x < -1 . Ento
podemos concluir que o raio de convergncia R = 1 . Para en-

271
contrar todo o intervalo de convergncia, precisamos verificar a
convergncia nos extremos do intervalo.
Para x = 1 e x = 1 , temos as seguintes sries:
(-1) n -1
e

n =1 2n - 1

(-1) n -1 (-1) 2 n -1 (-1) n


.
=

2n - 1
n =1
n =1 2n - 1
1
> 0 (uma srie
As duas so sries alternadas com bn =
2n - 1
oposta outra).

Note que, para todo n , 2(n + 1) 1 = 2n + 1 > 2n 1 , o que


implica
1
1
,
>
2n - 1 2(n + 1) - 1
ou seja, bn > bn +1 . Isso mostra que a sequncia (bn ) decrescente. Como
1
lim
=0,
n 2n - 1
pelo Teste da Srie Alternada as duas sries so convergentes.
Assim, a srie de potncias convergente para todo x tal que
1 x 1 (ou x [ 1,1]) .
b) Podemos escrever an =

(-3 x) n
(-1) n (3 x) n
como an =
. Assim,
n +1
n +1

an +1
n +1
n +1
(-1) n +1 (3 x) n +1
=
=
3x e
n
n
an
n + 1 + 1 (-1) (3 x)
n+2
lim

an +1
n +1
= lim
3 x =3 x .
n

an
n+2

Pelo Teste da Razo, a srie absolutamente convergente se


1
1
3 | x | < 1 , ou | x | < , e divergente se | x | > .
3
3

(-1) n
1
temos a srie numrica
que conver3
n +1
n =0
gente (veja Exerccio proposto 15) ou prove como no item a)).

Para x =

(-1) n (-1) n
1
1
=
Se x = - , a srie numrica
uma
3
n +1
n +1
n =0
n =0
1
p -srie com p = , logo divergente.
2

272
1 1
Concluso: a srie de potncias convergente quando x - ,
3 3
Pelo Teste da Razo, a srie absolutamente convergente se
1
1
3 | x | < 1 ou | x |< e divergente se | x |> . Logo o raio de con3
3
1
vergncia R = .
3

5.2.1 Exerccios
1) Encontre o raio e determine o intervalo de convergncia das
seguintes srie de potncias.
a)

nx n
n =1

b)

xn

n =1 n n

g)

( x - 1) n
c)
10n
n =0
n =1

e)

xn
n2 + 8

(2 x + 3) n

n2
n =1

(5 x - 1) n +1
h)
n
n =1

nn xn

n =0

d)

f)

(-1) n x n

n !
n =0

i)

(-1) n +1 ( x + 2) n

n 2n
n =1

j)

xn

n = 2 n ln n

5.3 Representao de funes como


sries de potncias

Na introduo do captulo vimos que a srie de potncias x n


n =0
1
no domnio de convergncia
convergente e sua soma f ( x) =
1- x
] 1,1[ e dizemos que a srie representa a funo f em ] 1,1[ .
Para cada x no domnio de convergncia, a srie de po

tncias

c ( x - a)
n =0

um

nmero

real

e,

assim,

n
correspondncia x cn ( x - a ) define uma funo real cujo don =0

mnio o intervalo de convergncia da srie.

273

Do mesmo modo que raras vezes encontramos a soma exata de uma


srie numrica convergente, determinamos o intervalo de convergncia de certas sries de potncias, mas no encontramos uma expresso para a funo soma. No caso particular da srie geomtrica,
temos uma expresso para a funo soma, ento representaremos
algumas funes como sries de potncias pela manipulao dessa
srie ou utilizando diferenciao e integrao de sries de potncias.
De outro lado vamos expressar funes conhecidas como polinmios infinitos. Isso til para, por exemplo:
1) Integrar funes que no possuem primitivas elementares.
2
Um exemplo clssico a funo f ( x) = e - x ;
2) Resolver equaes diferenciais;
3) Produzir aproximaes polinomiais. (As aproximaes simplificam as expresses e so eficazes, pois os erros so pequenos,
para representar funes em calculadoras e computadores).
Primeiros exemplos: Funes representadas como sries de potncias pela manipulao de sries geomtricas
1
1) A funo f ( x) =
, x ] - 1,1[ foi escrita como a seguinte s1- x
rie de potncias:

1
= 1 + x + x 2 + x3 ++ x n + = x n ,
1- x
n =0
no intervalo de convergncia ] 1,1[ .

Geometricamente
y=

1
1 x

S2
S1

1
0
Figura 5.1: Aproximaes de f ( x) =

S0
1
1
por S0 , S1 , S 2 , , no intervalo
1- x

274
Observe que
S0 = 1
S1 = 1 + x
2

1 3

S2 = 1 + x + x = x + +
2 4

2
3
S3 = 1 + x + x + x
2

2) Substituindo x por x na srie de potncias em 1), obtemos:

1
= 1 x + x 2 x3 + + (1) n x n + = (-1) n x n ,
1+ x
n =0
no intervalo de convergncia ] 1,1[ (note que | x | = | x | < 1 ).
3) Agora, substituindo x por x 2 na srie resultante em 2), obtemos:

1
2
4
6
n 2n
1

(
1)
(-1) n x 2 n , 0 x 2 < 1 ,
=
x
+
x
x
++
x
+
=

2
1+ x
n =0
o que implica 1 < x < 1 .
1
4) Para a > 0 , seja a funo f ( x) =
, x I , intervalo que dea+x
terminaremos abaixo.
De

1
1
1 1
=
=
,
a+x
x a 1+ x
a 1 +
a
a

substituindo x por

x
na igualdade em 2), obtemos
a
n

1
1
x
= (-1) n ,
a + x a n =0
a
x
< 1 ou x < a . Assim, no intervalo de convergncia
a

1
(-1) n
= n +1 x n .
a < x < a temos
a + x n =0 a

para

275

Diferenciao e integrao de sries de potncias


Sejam fi , i = 1, 2,3, , funes de x . Sabemos que se f1 e f 2 so funes diferenciveis (derivveis), ento a soma f1 + f 2 tambm diferencivel e vale
[ f1 + f 2 ]'(x) = f1 '( x) + f 2 '( x) .
De modo geral, a propriedade vale para um nmero finito n de funes diferenciveis e
[ f1 + f 2 + + f n ]'( x) = f1 '( x) + f 2 '( x) + + f n '( x) .
O mesmo acontece para as funes integrveis, ou seja, se f1 , f 2 , , f n
so funes integrveis, ento a soma integrvel e
n
n

f
(
x
)
dx
( f i ( x)dx) , para n finito.
=


i =1
i =1

Para uma soma infinita no muito simples. O caso geral poder


ser estudado em um curso mais avanado e nos restringiremos ao
caso particular e especial das funes potncias f n ( x) = cn ( x a ) n ,
n inteiro positivo. Assim, por falta de conceitos especficos (por
exemplo, convergncia uniforme), o teorema seguinte ser enunciado sem demonstrao.
Teorema 5.2. Derivao e integrao termo a termo.
Seja

c ( x - a)
n =0

uma

srie

de

potncias

lo de convergncia ]a R, a + R[ . Seja f

com

interva-

a funo definida por

f ( x) = cn ( x - a ) , para x ]a R, a + R[ . Ento,
n

n =0

i) A funo f diferencivel (e portanto contnua) em cada


x ]a R, a + R[ , e a derivada pode ser obtida por meio da derivao da srie inicial termo a termo



f '( x) = cn ( x - a) n = [cn ( x - a) n ] = ncn ( x - a) n -1 .
n =1
n =0
n =0
A srie das derivadas convergente para todo ponto interior
do mesmo intervalo de convergncia da srie inicial, ou seja, o
raio de convergncia o mesmo.

276
ii) A funo f tambm integrvel e temos

( x - a ) n +1

n
n
cn ( x - a ) dx = ( cn ( x - a ) dx) = cn
,
f ( x)dx =
n +1
n =0
n =0
n =0

para cada x no interior do intervalo de convergncia da srie


inicial, em que a srie resultante das integrais tambm convergente.

Observao 5.4. No teorema temos que tanto a srie derivada como


a integrada de uma srie de potncias so convergentes quando x
pertence ao intervalo (aberto) de convergncia da srie inicial. Se
x um dos extremos do intervalo, a srie pode ser convergente ou
divergente, portanto no teorema temos apenas a garantia de que o
raio de convergncia o mesmo.
Nota. Como mencionamos antes do enunciado do Teorema 5.2. acima, a derivao e integrao termo a termo pode no valer para

sen(n ! x)
sries de outros tipos de funes. Por exemplo, a srie

n2
n =1
convergente para todo x , mas se derivamos termo a termo obtemos

n !cos(n ! x)
a srie
que divergente.
n2
n =1
Exemplo 5.6. Represente cada uma das funes dadas como sries
de potncias, pela diferenciao ou integrao de srie de potncias dos primeiros exemplos acima, e determine seu raio de convergncia.
a) f ( x) =

1
(1 - x) 2

Observe que

1
= (1 - x) -2 e lembre-se que
2
(1 - x)

[(1 x) 1 ]' = 1(1 x) 2 ( 1) , ou seja,

1
1
=

.
(1 - x) 2 1 - x

1
Da srie geomtrica
= x n , para x tal que | x | < 1 segue
1 - x n =0
que,

1
n
(
)
'
=
x
=
nx n -1 , para x tal que | x | < 1 .

2
(1 - x)
n =0
n =1

(Note que se n = 0 , x 0 = 1 e a derivada da funo constante 1 a


funo nula).

277

Logo,

1
=
nx n -1 e o raio de convergncia R = 1 .

2
(1 - x)
n =1

b) g ( x) = ln 1 + x
1
Do captulo de integrais temos que
dx = ln |1 + x | + c, c ,
1+ x
e do Exemplo 2) acima,

1
= (-1) n x n , para x tal que | x | < 1 . Ento,
1 + x n =0
n +1

1

n n
n
n
n x
ln |1 + x | + c =
dx = (-1) x dx = (-1) ( x dx) = (-1)
,
1+ x
n +1
n =0
n =0
n =0

para x tal que | x | < 1 .


A igualdade tem que valer para todo x tal que 1 < x < 1 , em
particular para x = 0 . Assim,

0n +1
ln |1 + 0 | + c = (-1) n
=0,
n +1
n =0
o que implica c = 0 . Portanto,

ln |1 + x | = (-1) n
n =0

x n +1
xn
= (-1) n -1 ,
n + 1 n =1
n

e o raio de convergncia R = 1 .

Exerccio resolvido
2) Encontre uma representao da funo f ( x) = arctgx em srie
de potncias.
Resoluo. Lembre-se (ou veja numa tabela de derivadas) que
2 n +1

1

n 2n
n
2n
n x
dx
x
dx
x
dx
(
1)
(
1)
(
)
(
1)
=
=
=
+c,

n=0

1 + x2
2n + 1
n =0
n =0

1
(arctgx) ' =
.
1 + x2

1
(-1) n x 2 n , para x tal que| x | < 1 .
No Exemplo 3) acima,
=

2
1+ x
n =0

arctgx =

278

Ento, para x tal que | x | < 1 .

0n +1
Para x = 0 temos, arctg0 = (-1) n
+ c . Como
n +1
n =0
to c = 0 . Portanto,

arctgx = (-1) n
n =0

, en-

x 2 n +1
, para x tal que| x | < 1 .
2n + 1

5.3.1 Exerccios
2) Encontre uma representao em srie de potncias para cada
funo e determine o intervalo de convergncia.
1
1 - x3
1
b) f ( x) =
9 + x 2
x
c) f ( x) =

1- x
a) f ( x) =

d) f ( x) =

1
(1 + x)3

e) f ( x) = ln | 4 - x |
x
f) f ( x) = arctg
5

x2n
3) Seja f ( x) = (-1)
. Calcule a srie para f ''( x) e verifi(2n)!
n =0
que que f ''( x) = f ( x) .

(Isso deve ser verdade, porque


mais adiante).

f ( x) = cosx , como veremos

5.4 Srie de Taylor e srie de Maclaurin


1
, que pode ser escrita
1- x
como a srie geomtrica, no intervalo de convergncia, para escrever outras funes como srie de potncias. claro que essas outras funes esto relacionadas, de alguma forma, com a funo f
acima, seja por manipulao algbrica, diferenciao ou integrao.
Agora, nosso objetivo escrever mais funes como sries de potncias, utilizando uma tcnica mais geral para a construo das sries.
At agora utilizamos a funo f ( x) =

279

5.4.1 Definies
Inicialmente faremos duas consideraes baseadas no que estudamos na seo anterior.
1
como
x

uma srie de potncias do tipo cn x n , num intervalo aberto

1) Obviamente no podemos escrever a funo f ( x) =


n =0

contendo 0 (zero) porque a srie convergente se x = 0 , mas a


funo no nem definida em x = 0 .
1
, | x | < 1 . Ento se
1+ x
n =0
fizermos y = 1 + x temos x = y 1 e a igualdade acima toma a

1
forma (-1) n ( y - 1) n = , | y - 1| < 1 , ou seja,
y
n =0

Por outro lado, sabemos que

f ( x) =

(-1)

xn =

1
= (-1) n ( x - 1) n , para x tal que | x 1| < 1 .
x n =0

Isso significa que uma funo pode no ser representada como


uma srie de potncias com centro em a = 0 , mas pode ser escrita como uma srie de potncias com centro em a 0 .
2) O resultado do Teorema 5.2. que, se uma funo definida
por uma srie de potncias,

f ( x) = cn ( x - a ) n = c0 + c1 ( x a ) + c2 ( x a ) 2 + + cn ( x a) n + ,
n =0

para
x ]a R, a + R[ , ento a funo diferencivel e a derivada de f pode ser dada pela srie derivada

f '( x) = ncn ( x - a) n -1 = c1 + 2c2 ( x a) + 3c3 ( x a) 2 + + ncn ( x a) n -1 + ,


n =1

no mesmo intervalo de convergncia ]a R, a + R[ .


Aplicando o mesmo teorema funo derivada f ' , obtemos a
derivada segunda

280
a srie sendo convergente para cada x no interior do intervalo
]a R, a + R[ .
A derivada terceira

f '''( x) = n(n - 1)(n - 2)cn ( x - a ) n -3


n =3

= 3.2.1c3 + 4.3.2c4 ( x a ) ++ n(n 1)(n 2)cn ( x a ) n -3 +


para x ]a R, a + R[ .
Assim por diante, a funo f tem derivadas de todas as ordens e as
sries derivadas, obtidas por derivao termo a termo, so convergentes no mesmo intervalo de convergncia.
Note que a derivada de ordem n
, com
todas as demais parcelas contendo potncias de ( x a ) .
Para x = a , temos f (a ) = c0 , f (a ) = c1 , f ''(a ) = 2.1c2 = 2!c2 ,
f '''(a ) = 3.2.1c3 = 3!c3 , e em geral, f ( n ) (a ) = n !cn .
Se adotarmos 0! = 1 e f (0) = f , ento cn =
e mostramos o seguinte teorema.

f ( n ) (a)
, para n = 0,1, 2,3,
n!

Teorema 5.3. Se uma funo f tem representao (ou expanso) em srie de potncias ao redor de a (ou em a ) , isto , se

f ( x) = cn ( x - a ) n , | x - a | < R , ento os coeficientes da srie so da


n =0

f ( n ) (a)
. Assim, se uma funo f tiver uma expanso
n!
em srie de potncias em a , ento
forma cn =

f ( x) =
n =0

f ( n ) (a)
f '(a )
f ''(a )
f ( n ) (a)
( x - a) n = f (a ) +
( x - a) +
( x - a)2 + +
( x - a)n +
n!
1!
2!
n!

para | x a | < R .
Observao 5.5. No teorema temos uma condio necessria para
uma funo f ser representada por uma srie de potncias ao redor de a . Se f pode ser representada por uma srie de potncias
ao redor de a , s pode ser pela srie com coeficientes da forma
cn =

f ( n ) (a)
.
n!

281

(-1) n -1 n
x , para x tal que
n
n =1
(-1) n -1
, para n 1 e c0 = 0 .
| x | < 1 (Exemplo 5.6). Note que cn =
n

Exemplo 5.7. Sabemos que ln |1 + x | =

Escrevendo f ( x) = ln |1 + x | , temos f (0) = ln1 = 0 = c0 . De


f '( x) =

1
= (1 + x) -1
1+ x

f ''( x) = 1(1 + x) 2
f '''( x) = (-2)(-1)(1 + x) 3 = 2.1(1 + x) 3
f ''''( x) = (-3)2.1(1 + x) 4 = (-1)3.2.1(1 + x) 4
conclumos que f ( n ) ( x) = (-1) n 1 (n - 1)!(1 + x) n , para n 1 .
Para o centro da srie a = 0 , f ( n ) (0) = (-1) n 1 (n - 1)! . Ento,
f ( n ) (0) (-1) n -1 (n - 1)! (-1) n -1
cn =
=
=
,
n!
n!
n
que est de acordo com o teorema.
Tarefa. Faa o mesmo para

1
= x n , | x |< 1 .
1 - x n =0

A primeira condio para escrever uma funo arbitrria como uma


srie de potncias centrada em a que a funo tem que ter derivadas de todas as ordens nos nmeros de um intervalo I , centrado
em a .
Definio 5.2. Seja f : D , onde o domnio D um intervalo da
reta e a D . Se
, a funo f tem derivadas de todas as ordens
em algum intervalo aberto I contendo a , ento a srie

n =0

f ( n ) (a)
f '(a )
f ''(a )
f ( n ) (a)
( x - a)n = f (a) +
( x - a) +
( x - a)2 + +
( x - a)n +
n!
1!
2!
n!

denominada srie de Taylor gerada pela funo f em a (ou ao


redor de a ou centrada em a ).
Caso especial de srie de Taylor gerada por f em a = 0

282

n =0

f ( n ) (0) n
f '(0)
f "(0) 2
f ( n ) (0) n
x = f (0) +
x+
x + +
x +
n!
1!
2!
n!

denominada srie de Maclaurin gerada por f .


Observao 5.6. Definimos uma srie de potncias especial, com
os valores dos coeficientes determinados pelas derivadas de uma
funo f , num ponto a . Note que no igualamos, e nem podemos
igualar, a srie funo f porque no sabemos nem se a srie
convergente. Como para qualquer srie de potncias temos que determinar o intervalo de convergncia. No intervalo de convergncia,
nem sempre possvel encontrar a soma da srie convergente, como
j sabemos. Mesmo sendo gerada por uma funo f , a soma pode
ser diferente da funo f , como mostra o seguinte exemplo.
- x12

Exemplo 5.8. A funo f ( x) = e , se x 0 tem derivadas de to0 , se x = 0


das as ordens em .
De fato, para x 0 , f '( x) = e

1
x2

'

2 - x2
1
- 2 = 3 e .
x x

Vamos calcular f '(0) por definio de derivada:


-

f (0 + h) - f (0)
e h -0
e h
f '(0) = lim
= lim
= lim
h 0
h 0
h 0 h
h
h
(lembre-se que h 0 ).
Como

lim e

1
h2

h 0

= 0 , podemos aplicar a Regra de LHpital. Note

(logo acima) que a derivada de e

h2

fica mais complicada. Ento

escrevemos o quociente de outra maneira:


f '(0) = lim
h 0

1
h
1

eh

= lim
h 0

- h12
- h23 e

1
h2

= lim
h 0

h
1

2e h

h 1
= 00 = 0 .
1
h 0 2
2
eh

= lim

Assim, f '(0) = 0 .
(Outra maneira de calcular f '(0) fazendo a mudana de

283
1
1
1
= y e lembrando que lim+ = + , lim- = - ,
h

0
h

0
h
h
h
y
1
f + '(0) = lim y 2 = lim
2 = 0 e tambm f '(0) = 0 .).
y +
y +
e
2 ye y

varivel

Se x 0 ,

'
1
1
2 - x2 2 - x2
f ''( x) = 3 e + 3 e
x
x

'

1
1
1

6 - x2 2 2 - x2 4 6 - x2
.
=
+
=
e
e
e

x4
x3 x3
x4
x

2
h3

1
h2

-0

f '(0 + h) - f '(0)
2e
= lim
= lim 4 . Escre0
0
h
h

h
h
h
vendo o quociente de outra maneira e aplicando a Regra de LHpital
duas vezes conclumos que f ''(0) = 0 .

Assim, f ''(0) = lim

h2

h 0

1 - 2
De modo geral, f (0) = lim p e h , em que p um polinmio, e
h 0
h
(n)
mostramos que f (0) = 0 , para todo n . Ento a srie de Maclaurin
gerada por f em a = 0 escrita como
(n)

0 + 0 x + 0 x 2 + 0 x3 + = 0 + 0 + 0 +
que convergente para zero, qualquer que seja x . Logo, a soma
da srie de potncias a funo nula e como f ( x) 0 , para todo
x 0 , f no pode ser representada pela srie, ao redor do zero.
Concluso. A toda funo infinitamente num intervalo aberto I ,
podemos associar sempre uma srie de Taylor em a I . Mas isso
no suficiente para essa srie representar a funo numa vizinhana de a .

Exerccio resolvido
1
em a = 2 . Vex
rifique se a srie convergente e se a funo soma f em
algum intervalo.

3) Encontre a srie de Taylor gerada por f ( x) =

Resoluo. No incio deste item (5.4.1. Definies) representamos


essa funo, como srie de potncias ao redor de a = 1 , a partir da
srie geomtrica:
1
f ( x) = = (-1) n ( x - 1) n ,
x n =0

284

para x tal que | x 1| < 1 . Agora vamos usar a definio da srie de


Taylor.
a) A srie de Taylor gerada por f em a = 2

n =0

f ( n ) (2)
f '(2)
f "(2)
f ( n ) (2)
( x - 2) n = f (2) +
( x - 2) +
( x - 2) 2 + +
( x - 2) n +
n!
1!
2!
n!
1
= x -1 , temos
x
f '( x) = 1x 2

De f ( x) =

f ''( x) = (-1)(-2) x 3 = 2! x 3
f '''( x) = 2!(-3) x 4 = 3! x 4 .
E conclumos que f ( n ) ( x) = (-1) n n ! x ( n +1) ou
Assim, f (2) =

f ( n ) ( x) (-1) n
= n +1 .
n!
x

1 f '(2) -1 f ''(2) 1 f '''(2) -1


= 2,
= 4 , ...,
,
= 3,
2
1!
2
2!
2
3!
2

f ( n ) (2) (-1) n
= n +1 .
n!
2
Logo,

n =0

f ( n ) (2)
(-1) n
1 1
1
1
( x - 2) n = n +1 ( x - 2) n = - 2 ( x - 2) + 3 ( x - 2) 2 - 4 ( x - 2)3 +
n!
2 2
2
2
n =0 2

1
1
e r = - ( x - 2) .
2
2
1
Ento ela absolutamente convergente se - ( x - 2) < 1
2
ou | x 2 | < 2 e, para cada x nesse intervalo, a soma

b) Note que a srie geomtrica com a =

1
2

1 + ( x - 2)
1
2

1
1
= .
2+ x-2 x

c) Portanto, mostramos que a srie de Taylor gerada pela funo


1
f ( x) = em a = 2 convergente e
x
1 1 1
1
1
f ( x) = = - 2 ( x - 2) + 3 ( x - 2) 2 - 4 ( x - 2)3 + ,
x 2 2
2
2
para x tal que | x 2 | < 2 ou 0 < x < 4 .

285

5.4.2 Caso que a srie de Taylor


tem como soma a funo f (x)
diferencivel (derivvel) em a . Pela definio de def ( x) - f (a)
, segue que
em a , f '(a ) = lim
rivada de f
xa
x-a
f ( x) = f (a ) + f '(a )( x - a ) + Ra ( x) , onde o resto Ra ( x) satisR ( x)
faz lim a
= 0 . Denotamos por P1 ( x) = f (a ) + f '(a )( x - a ) , polixa x - a
Seja

nmio de grau um (aproximao linear de f em a ) e escrevemos


f ( x) = P1 ( x) + Ra ( x) . Nesse caso dizemos que P1 aproxima f , na vizinhana de a , a menos de um infinitsimo de ordem maior que um.
Vamos generalizar esse resultado e para isso lembremos que uma
funo f n-vezes diferencivel (derivvel) em x = a , se existe um
intervalo aberto I (contido no domnio de f ), a I , tal que f
(n - 1) - vezes diferencivel em I (ou seja, em cada x I ) e, alm disso, existe f ( n ) (a ) .
Definio 5.3. Seja f uma funo com derivadas de ordem n em
a I , I intervalo aberto da reta. O polinmio de Taylor de ordem
n gerado por f em a o polinmio
f '(a )
f "(a )
f ( n ) (a )
Pn ( x) = f (a ) +
( x - a) +
( x - a)2 + +
( x - a)n .
1!
2!
n!
Observao 5.7. Na definio escrevemos polinmio de Taylor de
ordem n em vez de grau n porque f ( n ) (a ) pode ser zero (veja definio de grau de polinmio).
Por exemplo, se f '(a ) = 0 , ento P1 ( x) = f (a ) , polinmio de primeira ordem e grau zero (ordem se refere ordem da derivada).
Exemplo 5.9. Seja f ( x) = e x , x . Sabemos que f tem derivadas
de todas as ordens em e que f ( n ) ( x) = e x , para todo x .
Para x = 0 , temos
f '(0) = f ''(0) = = f ( n ) (0) = 1 . Ento o polinmio de Taylor de ordem n gerado por f em a = 0
x2
xn
Pn ( x) = 1 + x + + + .
2!
n!

286
Para n = 1 , P1 ( x) = 1 + x
x2
x2
= 1+ x +
2!
2
2
3
x
x
x 2 x3
n = 3 , P3 ( x) = 1 + x + + = 1 + x + + , etc.
2! 3!
2 6

n = 2 , P2 ( x) = 1 + x +

Exemplo 5.10. Para f ( x) = cosx temos, para todo x


f '( x) = senx
f '''( x) = senx

f ''( x) = cosx
f '''' ( x) = cosx (e voltamos funo inicial).

Assim, f (2 k +1) ( x) = (-1) k +1 senx


Se

f (2 k ) ( x) = (-1) k cosx

x = 0 , ento sen0 = 0 e cos0 = 1 .

Assim,

f (2 k +1) (0) = 0 e

f (2 k ) (0) = (-1) k e os polinmios de Taylor de ordem n = 2k ou n = 2k + 1


x2 x4 x6
x2k .
so idnticos P2 k ( x) = P2 k +1 ( x) = 1 - + - + + (-1) k
2! 4! 6!
(2k )!
Teorema 5.4. Frmula de Taylor infinitesimal
Seja f uma funo n vezes derivvel em a I , I intervalo aberto.
Ento, para todo x prximo de a ( x = a + h , para h pequeno), tem-se
f '(a )
f "(a )
f ( n ) (a )
f ( x) = f (a ) +
( x - a) +
( x - a)2 + +
( x - a ) n + Rn ,a ( x) ,
1!
2!
n!
Rn ,a ( x)
= 0.
sendo que o resto Rn ,a ( x) satisfaz lim
xa ( x - a) n
Alm disso, o polinmio de Taylor

o nico polinmio de ordem menor ou igual a n ( n) tal que


R ( x)
f ( x) = Pn ( x) + Rn ,a ( x) com lim n ,a n = 0 .
xa ( x - a)
Nota. Omitimos a prova do teorema no porque difcil (utiliza o
Teorema de Rolle), mas porque muito extensa.
Observao 5.8. Do teorema segue que uma funo f derivvel
at a ordem n em x = a , pode ser aproximada pelo polinmio de
Taylor Pn ( x) ( f ( x) = Pn ( x) + Rn ,a ( x) ou f ( x) - Pn ( x) = Rn ,a ( x)) , numa
vizinhana de a , a menos de um infinitsimo de ordem superior a
R ( x)

n , em relao a x lim n ,a n = 0 .
xa ( x - a)

287
Exemplo 5.11. Seja p um polinmio de grau n . Dado a , a frmula de Taylor infinitesimal nos d
p '(a )
p "(a )
p ( n ) (a)
2
p( x) = p(a) +
( x - a) +
( x - a) + +
( x - a) n + Rn ,a ( x) .
1!
2!
n!
Note que Rn ,a ( x) um polinmio de ordem n e todas as suas
derivadas, desde a ordem zero at n , se anulam em a , ento
Rn ,a ( x) = 0 para todo x e temos
p '(a )
p "(a )
p ( n ) (a)
2
p( x) = p(a) +
( x - a) +
( x - a) + +
( x - a)n .
1!
2!
n!
Uma maneira de estimar o resto Rn ,a ( x) dado pelo seguinte
teorema.
Teorema 5.5. Frmula de Taylor com resto de Lagrange
Se f uma funo derivvel at a ordem n + 1 em um intervalo aberto I , a I , ento para todo x I existe um nmero c entre x e a
tal que
f ( x) = f (a ) +

f '(a )
f "(a )
f ( n ) (a )
f ( n +1) (c)
( x - a) +
( x - a)2 + +
( x - a)n +
( x - a ) n +1 ,
1!
2!
n!
(n + 1)!

ou seja, Rn ,a ( x) =

f ( n +1) (c)
( x - a ) n +1 .
(n + 1)!

Demonstrao. Consideremos x > a . Para x < a , a prova anloga.


Por hiptese, as funes derivadas, at a ordem n + 1 , so definidas
em[a, x] , para cada x I . Definimos
: [a, x] pondo
(t ) = f ( x) - f (t ) -

f '(t )
f "(t )
f ( n ) (t )
K
(x - t) ( x - t )2 - ( x - t )n ( x - t ) n +1
1!
2!
n!
(n + 1)!

em que K uma constante escolhida de modo que (a ) = 0 .


Como f derivvel at a ordem n + 1 , ento contnua em [a, x]
e derivvel em ]a, x[ . Note que ( x) = 0 . Assim, pelo Teorema de
Rolle (Clculo I), existe c ]a, x[ tal que (c) = 0 .

288
Calculando a derivada
f ''(t )
f '(t ) f "'(t )
f ''(t )
(x - t) +
( x - t )2 +
2( x - t ) -
1!
1!
2!
2!
f ( n +1) (t )
f ( n ) (t )
K
n
(x - t) +
n( x - t ) n -1 +
( x - t )n =
n!
n!
(n)!
'(t ) = 0 - f '(t ) -

f ( n +1) (t )
K
K - f ( n +1) (t )
n
n
(x - t) + (x - t) =
( x - t )n .
=n!
n!
n!
K - f ( n +1) (c)
( x - c) n = 0 . Portanto, K = f ( n +1) (c) e proSegue que
n!
vamos o teorema.

Nota. Se
x

Rn ,a ( x) =
a

f ( n +1) integrvel em [a, x] ou


[ x, a ] , ento
( n +1)
f
(t )
( x - t ) n dt (Frmula de Taylor com resto integral).
n!

Estimativa do Resto
Se existe uma constante M n > 0 tal que f ( n +1) (t) M n , para todo t
entre a e x , inclusive, ento o resto Rn ,a ( x) na Frmula de Taylor
satisfaz a desigualdade
Mn
n +1
Rn ,a ( x)
x-a .
(n + 1)!
Finalmente, se f uma funo que tem derivadas de todas as ordens em um intervalo aberto I , a I , ento podemos escrever para
todo n ,
f ( n +1) (cx )
f ( x) = Pn ( x) + Rn ,a ( x) onde Rn ,a ( x) =
( x - a ) n +1 , cx entre a e x
(n + 1)!
(para cada n aplicamos o Teorema 5.5 e para cada x , existe c ).
O valor absoluto Rn ,a ( x) = f ( x) Pn ( x) denominado o erro associado aproximao.
Se as condies para a estimativa do resto, acima, forem vlidas

f ( n ) (a)
para todo n , ento lim Rn ,a ( x) = 0 e f ( x) =
( x - a ) n , ou
n
n!
n =0

289

seja, a funo f igual soma da srie de Taylor gerada por ela em


a , para x numa vizinhana de a .
(No Teorema 5.4, o n fixo e o limite quando x a . Aqui, x fixo
e o limite quando n + .)
Exemplo 5.12. No Exemplo 5.9 encontramos o polinmio de Taylor de
x2
xn
ordem n , gerado por f ( x) = e x em a = 0 : Pn ( x) = 1 + x + + + .
2!
n!
A funo f tem derivadas de todas as ordens em e f ( n ) ( x) = e x ,
qualquer n .
De f ( n ) (0) = 1 para todo n , a srie de Taylor gerada por f em a = 0
(ou srie de Maclaurin gerada por f )
f '(0)
f "(0) 2
f ( n ) (0) n
x+
x + +
x +
1!
2!
n!
1
1
= 1 + x + x2 + + xn +
2!
n!

1
= xn
n =0 n !
f (0) +

No Exemplo 5.3 mostramos que a srie convergente, qualquer


que seja x . Resta mostrar que para cada x a soma da srie
f ( x) = e x .
De fato, como a funo satisfaz as condies do Teorema 5.5 para
todo n e em todo , ento para cada x existe um nmero cx
entre x e a = 0 tal que
1
1
e cx
x n +1 , para
e x = 1 + x + x 2 + + x n + Rn ,0 ( x) , onde Rn ,0 ( x) =
(n + 1)!
2!
n!
todo n . Para cada n e para cada x existe um cx . (No usamos cn , x
para no carregar mais na notao.) Agora vamos encontrar uma
estimativa para o resto e para isso usamos o fato de que f ( n +1) (t ) = et
uma funo crescente.

290

Fixemos x , que pode ser maior que zero ( x > 0) ou menor ( x < 0) .
1
n +1
i) Se x < 0 , ento x < cx < 0 e, assim, | Rn ,0 ( x) |<
x , para
(n + 1)!
todo n .
ex
n +1
x , para todo n .
ii) Se x > 0 , x > cx > 0 e temos, | Rn ,0 ( x) |<
(n + 1)!
Seja M o maior entre 1 e e x . Ento M > 0 (neste exemplo M no deM
n +1
pende de n , somente de x ) e 0 | Rn ,0 ( x) | <
x para todo n .
(n + 1)!

x n +1
1
= 0 , pois a srie x n convergente (Exemplo
Como lim
n ( n + 1)!
n =0 n !
5.3. Tambm pode ser calculado o limite.), para todo x , pelo Teorema do Confronto lim Rn ,a ( x) = 0 .
n

Portanto, a soma da srie e x , ou seja,

1
1
1
e x = x n = 1 + x + x 2 + + x n + , para todo x .
2!
n!
n =0 n !
Clculo de e
1
, que j vimos
n =0 n !
no Exerccio resolvido 4.8, e denominamos srie e . Agora vamos
calcular o valor de e correto at a quinta casa decimal. Para isso usamos a aproximao pelo polinmio de Taylor
1
1
ec
, para algum c
e = 1 + 1 + + + + Rn ,0 (1) , onde Rn ,0 (1) =
(n + 1)!
2!
n!
entre 0 e 1.

Para x = 1 na expresso de e x acima temos e =

Sabemos que 1 < ec < e e que e < 3 (veja o Exerccio resolvido 4.8).
Ento
1
3
e queremos que o erro seja menor que 10 5 .
< Rn ,0 (1) <
(n + 1)!
(n + 1)!
Precisamos descobrir o n de (n + 1)! que satisfaa 3[(n + 1)!]1 < 10 5 .
Note

que

10! = 3628800 ,

3[9!]1 < 10 5 , mas

9! = 362880

8! = 40320 ,

assim,

3[8!]1 > 10 5 . Portanto, (n + 1) deve ser no m-

nimo 9 ou n no mnimo 8. Logo, com erro menor que 10 5


1
1
e 1 + 1 + + + 2, 71828 .
2!
8!

291

Exemplo 5.13. Os polinmios de Taylor de ordem 2k (n par) e


2k + 1 (n mpar) gerados por f ( x) = cosx em a = 0 so idnticos:
P2 k ( x) = P2 k +1 ( x) = 1 -

x2 x4 x6
x2k
+ - + + (-1) k
(Exemplo 5.10).
2! 4! 6!
(2k )!

A funo cosseno tem derivadas de todas as ordens em todo x e


f (2 k ) ( x) = (-1) k cosx ,

f (2 k +1) ( x) = (-1) k +1 senx . Como

f (2 k +1) (0) = 0 e

f (2 k ) (0) = (-1) k , ento a srie de Maclaurin (pois o centro zero) gerado por f ( x) = cosx
f '(0)
f "(0) 2 f "'(0) 3
f ( n ) (0) n
x+
x +
x + +
x +
1!
2!
3!
n!
1
1
(-1) k 2 k
x +
= 1 + 0 x - x 2 + 0 x3 + x 4 + + 0 x 2 k -1 +
2!
4!
(2k )!
f (0) +

(-1) 2 k 2 k
x
k = 0 (2k )!

Pelo Teorema 5.5, para cada x ,


1
1
1
(-1) k 2 k
cosx = 1 - x 2 + x 4 - x 6 + +
x + Rn ,0 ( x) ,
2!
4!
6!
(2k )!
onde Rn ,0 ( x) =

f ( n +1) (cx ) n +1
x , para algum cx entre x e 0.
(n + 1)!

Isso vale para cada n ( n par ou mpar). Como tanto sen(t ) 1


1
n +1
como cos(t ) 1 , qualquer t , ento 0 | Rn ,0 ( x) |
x
(n + 1)!
para todo n .
x n +1
= 0 (o mesmo limite do exemplo anterior) segue que
n ( n + 1)!
lim Rn ,a ( x) = 0 , para todo x . Portanto,

De lim
n

cosx = 1 -

1 2 1 4 1 6
(-1) k 2 k
(-1) k 2 k
x + x - x + +
x + =
x ou
2!
4!
6!
(2k )!
k = 0 (2k )!

(-1) n 2 n
x para todo x .

n = 0 (2n)!

292

Exerccio resolvido
Encontre a srie de Maclaurin gerada por f ( x) = senx e mostre que a
srie convergente para f ( x) = senx , para todo x .
Resoluo. Sabemos que f ( x) = senx tem derivadas de todas as ordens, para todo x .
f '( x) = cosx
f '''( x) = cosx

f (2 k +1) ( x) = (-1) k cosx

f ''( x) = senx
f ''''( x) = senx ( e voltamos funo inicial )

f (2 k ) ( x) = (-1) k senx

Assim, f (2 k +1) (0) = (-1) k e f (2 k ) (0) = 0 . Logo, a srie de Maclaurin


tem apenas termos de ordem mpar e

x3 x5 x 7
x 2 k +1
(-1) 2 k +1 2 k +1
x - + - + + (-1) k
+ =
x , x
3! 5! 7!
(2k + 1)!
k = 0 (2k + 1)!
(voc pode encontrar expresso com 2k 1 no lugar de 2k + 1 . Nesse
caso, os valores de k comeam com k = 1 em vez de k = 0 ).
Usando o mesmo argumento do exemplo anterior, para a funo
cosseno, conclumos que

x3 x5 x 7
(-1) 2 n +1 2 n +1
senx = x - + - + =
x
, para todo x .
3! 5! 7!
n = 0 (2n + 1)!
Outro mtodo de resoluo:
Como senx = (-cosx) ' , usamos a representao de f ( x) = cosx pela
sua srie de Maclaurin e a diferenciao (derivao) termo a termo
para obter:

(-1) n 2 n (-1) n +1
(-1) n +1 2 n -1
.
senx = -
(2n) x 2 n -1 =
x =
x
n =1 (2n - 1)!
n =0 (2n)!
n =1 (2n)!
Sefazemos n 1 = k , ento n = k + 1. Assim, 2n 1 = 2(k + 1) 1 = 2k + 1
e n + 1 = k + 2 . Tambm n = 1 implica k = 0 . (Reindexao, Captulo 4, 4.3.)
(-1) 2 n +1 2 n +1
x
, para todo x (aqui s mudamos a
n = 0 (2n + 1)!
letra k por n ).

Logo, senx =

293
Nota. Para simplificar os enunciados:
Srie de Taylor gerada por f em a significar apenas a srie.
Srie de Taylor para f em a significar que a srie representa
a funo f , numa vizinhana de a , ou seja, alm de a srie ser
gerada pela funo, a srie convergente para f ( x) , para cada
x numa vizinhana de a .

Exerccio resolvido
Encontre a srie de Taylor para f ( x) = e x em a = 2 .
Resoluo. Como j vimos no Exemplo 5.12, a derivada de ordem n
de f ( x) = e x f ( n ) ( x) = e x , para todo n e todo x .
Para x = 2 temos f ( n ) (2) = e 2 , para todo n , logo, a srie de Taylor de

e2
e2
n
f em a = 2 ( x - 2) n . O termo geral da srie an = ( x - 2 ) ,
n!
n =0 n !
x-2
an +1
a
e 2 ( x - 2) n +1
n!
ento
=
=
e lim n +1 = 0 , para
2
n
n

an
n +1
(n + 1)! e ( x - 2)
an
todo x .
Da, pelo Teste da Razo, a srie convergente para todo x , ou
seja, o raio de convergncia da srie R = + .
Para cada n ,
e2
e2
e2
e x = e 2 + ( x - 2) + ( x - 2) 2 + + ( x - 2) n + Rn ,2 ( x) , onde
1!
2!
n!
cx
e
n +1
Rn ,2 ( x) =
( x - 2 ) , para algum cx entre x e 2.
(n + 1)!
Como a funo exponencial crescente ecx est entre e x e e 2 . Seja M
o maior entre os dois (e x e e 2 ) e temos
n +1

x-2
= 0 (j usamos esse limite duas vezes, com pequena
n ( n + 1)!
variao), segue pelo Teorema do Confronto, que lim Rn ,2 = 0 , para
n
todo x .

e2
Portanto, e x = ( x - 2) n , para todo x .
n =0 n !
De lim

294
Observao 5.9. Representamos a funo f ( x) = e x de duas manei
xn
x
ras. A srie de Maclaurin e = , para todo x , e a srie de
n =0 n !
e2
( x - 2) n , para todo x . Para
n
!
n =0
x prximo de zero, a melhor aproximao de e x dada pela srie

Taylor ao redor de a = 2 e x =

de Maclaurin e, se x nmero prximo de a , pela srie de Taylor


ao redor de a .

Exerccio resolvido

.
3

3
Resoluo. O valor do seno em a f = sen =
. Utilizando as
3
3 2

1
derivadas calculadas no Exerccio resolvido 5.4, f ' = cos = ,
3
3 2
3
1

,
f '' = -sen = f ''' = - cos = - , ... ,
2
2
3
3
3
3
1
3

e f (2 k ) = (-1) k
, para todo k .
f (2 k +1) = (-1) k
2
2
3
3

Assim, a srie de Taylor gerada por f ao redor de a =


3
n
2k
2 k +1


1 ( n )
(-1) k 3
(-1) k 1


=
+
f
x
x
x


3 k =0 (2k )! 2
3
(2k + 1)! 2
3
3
n =0 n !

6) Encontre a srie de Taylor para f ( x) = senx em a =

(para cada valor de k temos duas parcelas consecutivas).


Na Frmula de Taylor (Teorema 5.5) o resto, nesse caso,
n +1
f ( n +1) (cx )

. Como
R ( x) =
x - , para algum cx entre x e

n,
3
(n + 1)!
3
3
f ( n +1) (cx ) 1 , qualquer que seja n (par ou mpar), pois
cos( x) 1 , qualquer que seja x , ento 0 R

n,

e nesse caso tambm, lim R


n

n,

3
2k

( x)
3

x(n + 1)!
3

e
n +1

= 0 . Assim,

2 k +1
(-1) k 3
(-1) k 1

.
senx =
x- +
x-
3
(2k + 1)! 2
3
k =0
(2k )! 2

Observao 5.10. No item 5.2 representamos algumas funes como


soma de sries de potncias, pela manipulao de sries geomtri-

295
cas, pela diferenciao ou integrao de sries obtidas antes. Pelo Teorema 5.3, se uma funo f tiver uma representao por sries de
f ( n ) (a)
potncias em a , ento seus coeficientes so da forma
. Logo,
n!
as sries de potncias obtidas por mtodos indiretos so realmente
as sries de Taylor (ou Maclaurin no caso a = 0 ) para tais funes.
O clculo das derivadas sucessivas f ( n ) ( x) na maioria das vezes
1
quase que
maante e para funes simples como f ( x) =
1 + x3
impraticvel. Por isso, costuma-se utilizar outros mtodos, entre os
quais os do item 5.2 e o que veremos abaixo, para obter a srie de
Taylor para uma dada funo.
Combinaes de sries de Taylor
Na interseo dos seus intervalos de convergncia;
1) A srie de Taylor para
a soma (diferena) da srie de Taylor para f ( x) e a srie de Taylor para
g ( x) .
2) Multiplicaes por constantes e por potncias de x de sries
de Taylor resultam em sries de Taylor.
3) Tambm podemos fazer substituies, por exemplo, x por x 2 ,
como fizemos nas sries geomtricas em 5.2.
Exemplo 5.14. A srie de Maclaurin para f ( x) = xsenx
, para todo x .
Exemplo 5.15. A srie de Maclaurin para f ( x) = cos(2 x)
, para todo x .
(-1) 2 n +1 2 n +1
y
para todo y , trocar y por
n = 0 (2n + 1)!

x - para obter a srie de Taylor de f ( x) = senx em a = , encon3


3
trada no exerccio resolvido 6 (use a frmula de adio de ngulos

Tarefa. Sendo seny =

para sen x - ).
3

296

5.4.1 Exerccios
1) Encontre o polinmio de Taylor de ordem n gerado por f em a .
n = 5.
a) f ( x) = lnx , a = 1 ,

b) f ( x) = senx , a = , n = 4 .
4
c) f ( x) = tgx , a = 0 , n = 3 .
2) Encontre a srie de Taylor gerada por f com centro em a
dado. Prove que a srie obtida representa a funo f , no intervalo de convergncia.
a) f ( x) = 1 + x + x 2 + x 3 , a = 2 .
b) f ( x) = e x , a qualquer.

c) f ( x) = senx , a = .
4
1
d) f ( x) = 2 , a = 1 .
x
3) Ache por qualquer mtodo a srie de Maclaurin para as funes.
a) f ( x) = x 2 e x
e x + e- x
b) f ( x) = coshx
2

e x - e- x
c) f ( x) = senhx =
2
d) f ( x) = xcosx + senx
e) f ( x) = sen 2 x (sugesto: sen 2 x = 1 [1 cos2 x] )
2

5.5 Aplicaes
5.5.1 Aplicaes de Polinmios de Taylor.
f ( n ) (a)
( x - a)n ,
n
!
n =0
x numa vizinhana de a . Se Pn o polinmio de Taylor de ordem n ,
ento f ( x) = Pn ( x) + Rn ,a ( x) com lim Rn ,a ( x) = 0 e, assim, Pn pode ser

Seja f ( x) igual soma de sua srie de Taylor; f ( x) =

usado como uma aproximao de f (denotamos por f ( x) Pn ( x) ).


A preciso da aproximao obtida com a estimativa do resto
M
n +1
se f ( n +1) ( x) M . Utilizando essa mesma
| Rn ,2 ( x) |
x-a
(n + 1)!
frmula de estimativa, possvel decidir a ordem do polinmio que
aproxima uma funo para um dado erro.

297

Exemplo 5.16. Encontramos a srie de Maclaurin para a funo


f ( x) = senx :

(-1) n x 2 n +1
x3 x5 x 7
senx =
= x - + - + ... , sendo x qualquer n3! 5! 7!
n = 0 (2n + 1)!
mero real.
Note que os coeficientes de ndices pares so iguais a zero. Enx3 x5
a mesma que
to a aproximao, por exemplo, senx x - +
3! 5!
3
5
x
x
senx x + 0 x 2 - + 0 x 4 + + 0 x 6 .
3!
5!
1 7
A melhor estimativa do erro dada por R6 ( x) x (veja Exerc7!
1
1
cio resolvido 5.4). Para x prximo de zero, por exemplo, - < x < ,
5
5
7
1
1 1
1
ou | x | <
.
temos, R6 ( x) =
7! 5 393750000
5
Podemos usar essa aproximao para calcular, por exemplo,
sen10 . Para isso primeiro precisamos converter 10 em radianos:
180 ______
10 ______

1
o que implica =
(que menor que ).
5
18
3

1
1
Ento, sen10 = sen - +
0,17364817 e com
18 18 6 18 120 18
preciso de seis casas decimais sen10 0,173648 .
Por outro lado, se queremos os valores de x para os quais a aproximao tem preciso de 0,00005, resolvemos a desigualdade
1
1
7
| x |7 =
x < 0, 00005 .
7!
5040
7

De x < 0, 252 segue que x < (0, 252) 7 0,82 .


x3 x5
tem preciso de 0,00005
Assim, a aproximao senx x - +
3! 5!
quando x < 0,82 .
Observao 5.11. Outra aplicao o clculo do nmero real e , visto depois do Exemplo 5.12.

298

5.5.2 Srie binominal


A expanso (a + b) m , onde o expoente m um nmero inteiro, era
conhecido pelos matemticos chineses muitos sculos antes da poca de Newton.
Teorema Binomial. Se a e b so nmeros reais quaisquer e m um
nmero inteiro positivo, ento
m(m - 1) m - 2 2
m(m - 1) (m - k + 1) m - k k
(a + b) m = a m + ma m -1b +
a b + +
a b + + mab m -1 + b m
2
k!
O

coeficiente

de

a m - k b k pode

ser

escrito

como

m
m!
m(m - 1)...(m - k + 1)
k = 1, 2, , m
.
,
=
=
k!
k k !(m - k )!
m(m - 1) (m - k + 1)
Observe que a frmula
no funciona para k = 0 .
k!
m
Mas como 0! = 1 , por definio, = 1 e, assim escrevemos,
0
m
m m-k k
m
(a + b) = a b .
k =0 k
m
m
No caso particular, a = 1 e b = x temos (1 + x) m = x k .
k =0 k
Voc deve conhecer essa expanso e a demonstrao por induo ou
uma consequncia do resultado mais geral que veremos a seguir.
Queremos encontrar uma frmula para (a + b) r , onde r um nmero real qualquer. Antes, algumas observaes elementares:
i) Para r = 0 , temos (a + b)0 = 1 e o problema est resolvido. Ento, vamos supor r 0 .
ii) A exponencial c x definida somente para c > 0 . Portanto, temos que supor a + b > 0 .
iii) Se a = b , ento (a + a ) r = 2r a r e o problema tambm est resolvido. Ento, vamos supor a b .
iv) Como a e b so dois nmeros reais qualquer, podemos supor
a > b (se a < b , basta mudar para b + a que igual a a + b ). Esb
b
crevendo a + b = a 1 + (note que a 0 ) e fazendo x = ,
a
a
obtemos a + b = a (1 + x) .

299
b
b
b
1 implica 1 ou -1 .
a
a
a
Da b a ou b a , que contradizem a > b , acima, e a + b > 0 (ii).

Note que x < 1 , pois x 1 , isto ,

Concluso: Basta achar uma frmula para (1 + x) r , r 0 e x < 1 .


Como comentamos na introduo deste captulo, Newton foi o primeiro a estender a expanso de (1 + x) r , para o caso onde o expoente r no inteiro positivo. Nesse caso, a expresso no mais uma
soma finita, e sim uma srie de potncias centrada em zero. Pelo Teorema 5.3, a srie de Maclaurin de f ( x) = (1 + x) r que vamos construir calculando as derivadas em x = 0 . Note que f (0) = 1 .
f '( x) = r (1 + x) r -1

f '(0) = r

f ''( x) = r (r - 1)(1 + x) r - 2

f ''(0) = r (r - 1)

f '''( x) = r (r - 1)(r - 2)(1 + x) r - 3

f '''(0) = r (r - 1)(r - 2)

f ( n ) ( x) = r (r - 1) (r - n + 1)(1 + x) r - n

f ( n ) (0) = r (r - 1) (r - n + 1)

Portanto, a srie de Maclaurin gerada por f ( x) = (1 + x) r

n =0

f ( n ) (0) n
r (r - 1) 2 r (r - 1)(r - 2) 3
r (r - 1) (r - n + 1) n
x = 1 + rx +
x +
x + = 1+
x .
n!
2!
3!
n!
n =1

Aqui cabe uma observao: para r = m , inteiro positivo, sabem(m - 1)...(m - n + 1)


m!
, mas se r
mos que o quociente
=
n!
n !(m - n)!
no inteiro positivo o fatorial r ! no definido. Por outro lado,
r (r - 1) (r - n + 1)
tem significado para qualquer nmero real r e
n!
qualquer inteiro n 1 . Ento, para simplificar a notao podemos
r r (r - 1) (r - n + 1)
, qualquer nmero real r e qualquer
definir =
n!
n
inteiro n 1 .
1 1 ( 1 - 1)( 12 - 2) 1 1 1 3
3
1
Por exemplo, 2 = 2 2
=
= 4.
- - =
3
3!
3! 2 2 2 3!2
2
3

300

r
Observe que = r , e definimos
1

f ( n ) (0) n r n
x = x .

n!
n =0
n =0 n

r
= 1 . Assim,
0

Para verificar a convergncia da srie, seja an =

r (r - 1) (r - n + 1) n
x e
n!

r-n x
an +1
r (r - 1) (r - n)
n!
=
=
.
an
r (r - 1) (r - n + 1)
n +1
(n + 1)!
r
-1
r-n
x = lim n 1 x = x , pelo Teste da Razo, a sn n + 1
n 1 +
n
rie convergente quando x < 1 e divergente quando x > 1 . Fal-

Como lim

ta mostrar que o resto Rn ,0 ( x) tende a zero quando n tende


a infinito. Isso possvel, mas um pouco difcil, por isso apresentamos uma prova alternativa de que a funo soma
f ( x) = (1 + x) r , x < 1 . Para isso, como a srie convergente, seja

r
g ( x) = x n , para x < 1 . Temos que mostrar que g ( x) = (1 + x) r .
n =0 n
Consideremos h( x) = (1 + x) r g ( x) e primeiro vamos mostrar que
h '( x) = 0 .

r
Derivando a srie termo a termo, obtemos g '( x) = nx n -1 .
n =1 n
Para reindexar a srie fazemos n 1 = k e assim,

r
k
g '( x) =
(k + 1) x .
+
k
1
k =0

Note que
r
r (r - 1)(r - 2)...(r - (k - 1))(r - k )
(k + 1)

(k + 1) =
(k + 1)!
k + 1
=

r
r (r - 1)...(r - k + 1)
(r - k ) = (r - k ).
k!
k

301
Logo,

r
r
r
g '( x) = (r - k ) x k = rx k - kx k
k =0 k
k =0 k
k =0 k

r
= rg ( x) - x kx k -1
k =1 k

= rg ( x) xg '( x) ou (1 + x) g '( x) = rg ( x).


Usando essa igualdade na derivada
h '( x) = r (1 + x) r

g ( x) + (1 + x) r g '( x)

obtemos h '( x) = -(1 + x) r 1 (1 + x) g '( x) + (1 + x) r g '( x) = 0 . Ento,


h( x) = c , c constante, para x < 1 . Em particular, h(0) = c e como
h(0) = g (0) = 1 , conclumos que h( x) = 1 e portanto, g ( x) = (1 + x) r ,
como queramos provar.
A Srie Binominal: Para qualquer nmero real r e x < 1 , temos

r
r r (r - 1) (r - n + 1)
r
, n 1.
(1 + x) r = x n , onde = 1 e =
n!
n =0 n
n
0
Exemplo 5.19. Usando a srie binominal vamos representar a fun1
o f ( x) =
como uma srie de potncias.
(1 + x) 2
1
Note que
= (1 + x) 2 , r = 2 na frmula da srie binominal
(1 + x) 2

-2 n
1
=
e, assim, quando x < 1 , temos

x .
2
(1 + x)
n =0 n
Como
-2 (-2)(-2 - 1)(-2 - 2) (-2 - n + 1)
=
n!
n
(-1) n 2.3.4 (n + 1)
=
n!
n
(-1) (n + 1)!
=
n!
nn
= ((--1)
1) ((nn++1)
1)
ento

1
(-1) n (n + 1)x n , x < 1 .
=

2
(1 + x)
n =0

Exemplo 5.20. A srie de Maclaurin para a funo f ( x) = 1 + x


pode ser obtida usando a srie binominal, pois
sim, para x < 1 ,

1
2

1 + x = (1 + x) . As-

302

1
1
1
1
1
1 + x = 2 x n = 1 + 2 x + 2 x 2 + 2 x 3 + 2 x 4 +
n =0 n
1
2
3
4
1 1
( - 1) 2 12 ( 12 - 1)( 12 - 2) 3 12 ( 12 - 1)( 12 - 2)( 12 - 3) 4
1
= 1+ x + 2 2
x +
x +
x +
2
2!
3!
4!
1
1
(- 1 )
(- 1 )(- 32 ) 3 12 (- 12 )(- 23 )(- 52 ) 4
1
= 1 + x + 2 2 x2 + 2 2
x +
x +
2
2
3 2
4 3 2

1
1
1
5
x - 3 x 2 + 4 x3 - 7 x 4 + , x < 1 .
2
2
2
2
1
Note que 1 + x 1 + x , aproximao linear.
2
1
1
1 + x 1 + x - 3 x 2 , aproximao quadrtica.
2
2

Logo,

1+ x = 1+

Observao 5.11. A substituio de x d outras aproximaes:


1
1
1 - x 2 1 - x 2 - 3 x 4 , x 2 < 1 ou x < 1 .
2
2
1
1
1
1
1- 1- 3 2 , < 1 ou x > 1 .
x
2x 2 x x

5.5.3 Clculo de integrais aproximadas.


Algumas funes so muito difceis de integrar pelas tcnicas estudadas, mas podemos usar a ideia de Isaac Newton e representar
funes como sries de potncias para depois integrar a srie termo
a termo.
1
um exemplo de funo
1 + x7
difcil de integrar. Ento vamos represent-la como uma srie

1
de potncias substituindo x por x 7 na srie
= (-1) n x n ,
1 + x n =0
x < 1 (srie geomtrica. Primeiros exemplos de 5.2):

1
(-1) n x 7 n = 1 - x 7 + x14 - x 21 + , x < 1 . Integrando a srie
=

7
1+ x
n =0
Exemplo 5.21. A funo f ( x) =

termo a termo, obtemos a integral indefinida da funo dada como


srie de potncias,

303

1
(-1) n x 7 n dx
dx
=
1 + x7

n =0

= (-1) n x 7 n dx
n =0

(-1) n x 7 n +1
7n + 1
n =0

=c+

=c+x-

x8 x15 x 22
+
+
8 15 22

( c constante de integrao), para x 7 < 1 ou x < 1 .

Exerccio resolvido
a) Encontre uma srie de potncias para a integral indefinida e - x dx .
2

b) Calcule a integral definida e - x dx com preciso de 10 3 (= 0, 001) .


2

Resoluo.
2

a) Obtemos a srie de Maclaurin de f ( x) = e - x pela substituio

xn
x
2
x na srie de Maclaurin e = , para todo x :
n =0 n !

(- x 2 )n
(-1) n x 2 n
x2 x4 x6
=
= 1 - + - + , para
n!
n!
1! 2! 3!
n =0
n =0

f ( x) = e - x =
2

todo x .

Ento integramos termo a termo a srie resultante:


x2 x4 x6

(-1) n x 2 n
- x2
=
+
+
+
+ ... dx
e
dx
1

1! 2! 3!
n!

x3
x5
x7
(-1) n x 2 n +1
=c+x+
+ +
+
1!.3 2!.5 3!.7
n !(2n + 1)
(-1) n x 2 n +1
n ! 2n + 1
n =0

=c+

para todo x ( c constante de integrao).


b) Para calcular a integral definida, usamos uma funo primitiva
(funo F tal que F ' = f ), ento podemos tomar c = 0 .
1

-x
e dx = x 2

= 1-

x3
x5
x7
(-1) n x 2 n +1
+
+ +
+
1! 3 2! 5 3! 7
n !(2n + 1)

1
1
1
(-1) n
+
+ +
+
1! 3 2! 5 3! 7
n !(2n + 1)

1
0

304

Note que a srie alternada, ento o erro da aproximao da srie pela


1
1
soma parcial S n menor que 0,001 se n satisfaz
< 3
n !(2n + 1) 10
(Estimativa de sries alternadas 4.5.1).
Ento, n !(2n + 1) > 103 e calculando, por exemplo,
4! 9 = 216, 5!11 = 1320 ,
conclumos que devemos ter n = 4 (n + 1 = 5) , no mnimo. Assim,
1
1 1 1
1
- x2
e
0 dx 1 - 3 + 10 - 42 + 216 0, 7475 (com preciso de 0, 001 ).

5.5.1 Exerccios
1) Seja f ( x) =

.
1 - x2
a) Encontre a linearizao (polinmio de Taylor de ordem 1) de
f em x = 0 .

b) A aproximao quadrtica (ordem 2) de f em x = 0 .


2) Determine a preciso da aproximao linear senx x quando
x < 10-3 .
3) Ache uma srie para cada uma das seguintes funes e determine o intervalo (aberto) de convergncia.
x
x2
b) f ( x) = 1 + t10 dt
a) f ( x) =
1+ x
0
d) f ( x) = (2 + x 2 ) r , r 0 .

c) f ( x) = 3 2 + x
4)

a) Encontre uma srie de potncias para sen( x 2 ) dx .


1

b) Calcule a integral definida


10 3 (= 0, 001) .

sen( x
0

Respostas dos exerccios


5.2.1 Exerccios
1) a) R = 1 , I = ] 1, 1[ ;

) dx com preciso de

305
b) R = 1 , I = [1, 1] ;
c) R = 10 , I =] 9, 11[ ;
d) R = 0 , x = 0 ;
e) R = + , I =] , + [ ;
f) R = 1 , I = [1, 1[ ;
1
, I = [2, 1] ;
2
1
2
h) R = , I = 0, ;
5
5
g) R =

i) R = 2 , I =] 4, 0] ;
j) R = 1 , I = [1, 1[ .

5.3.1 Exerccios
1) a)
b)

3n

I =] 1, 1[ ;

n =0

(-1) n x 2 n
, I =] 3, 3[ ;

9n +1
n =0

n =0

n =1

c) -1 + x n = x n , I =] 1, 1[ ;
(n + 1)(n + 2) n
x , I =] 1, 1[ ;
d) (-1) n
2
n =0

xn
e) 2ln2 - 2 n , I = [4, 4[ ;
n =1 n 2

(-1) n x
f)

n = 0 2n + 1 5

2 n +1

, I = ] 5, 5] .

5.4.1 Exerccios
1
1
1
1
1) a) P5 ( x) = ( x - 1) ( x - 1) 2 + ( x - 1)3 - ( x - 1) 4 + ( x - 1)5
2
3
4
5
b)

2
2

P4 ( x) =
+
x- x- x- +
x-
2
2
4 4
4 12
4 48
4

306

1
c) P3 ( x) = x + x 3 .
3
2) a) f ( x) =15 + 17( x - 2) + 7( x - 2) 2 +( x - 2)3 , para todo x .
ea
( x - a ) n , para todo x .
n =0 n !

b) e x =

2n
2 n +1
1
2
1

n
(
1)
+
x
x

, para todo x .
2 n =0
4
(2n + 1)!
4
(2n)!

c) senx =
d)

1
(-1) n (n + 1)( x - 1) n , R = 1 .
=

2
x
n =0

(-1) n n + 2
x
, para todo x ;
n!
n =0

3) a) x 2 e - x =

x2n
, para todo x ;
n = 0 (2n)!

b) coshx =

x 2 n +1
c) senhx
, para todo x ;
n = 0 (2n + 1)!

d) f ( x) = (-1) n
n =0

2n + 2 2 n +1
x
, para todo x ;
(2n + 1)!

(-1) n +1 22 n -1 2 n
x , para todo x .
(2n)!
n =1

e) sen 2 x =

5.5.1 Exerccios
1) a) L( x) = 1 ;
x2
b) Q( x) = 1 +
2
(0,1)9
2) a) erro <
;
6
(0,1)3
b) erro <
< 1, 67 40-4 .
6
3) a)

- 12 n + 2
x , em ] 1, 1[ ;

n =0 n

12
1
b)
x10 n +1 , em ] 1, 1[ ;
n = 0 n (10n + 1)

307

c)

13 x
2 , em ] 2, 2[ ;
n =0 n 2

r x2
d) 2 , em ] - 2, 2[ .
n =0 n 2
r

(-1) n x 4 n +3
;
n = 0 (2n + 1)!(4n + 3)

4) a) c +

b) 0,310 .

Bibliografia bsica
STEWART, J. Clculo. v. 2. 4. ed. So Paulo: Pioneira Thomson
Learning, 2002.
THOMAS, G.B. et all. Clculo. v. 2. 10. ed. So Paulo: Addison
Wesley, 2003.

Bibliografia complementar
LIMA, E.L. Curso de Anlise. v. 1. 6. ed. Rio de Janeiro: Instituto
de Matemtica Pura e Aplicada, CNPq, 1989.
KONGUETSOF, L. Clculo Diferencial e Integral. So Paulo:
Editora McGraw-Hill do Brasil, 1974.

Você também pode gostar